Download as pdf or txt
Download as pdf or txt
You are on page 1of 154

i

TECHNOLOGICAL UNIVERSITY OF THE PHILIPPINES VISAYAS


Capt. Sabi St., City of Talisay, Negros Occidental

College of Engineering
Office of the Program Coordinator

LEARNING MODULE

MATH 213ME:
DIFFERENTIAL
EQUATIONS

DEPARTMENT: MECHANICAL ENGINEERING

COMPILED BY:

GENESIS D. DUMAICOS

2020

This module is a property of Technological University of the Philippines Visayas and intended
for EDUCATIONAL PURPOSES ONLY and is NOT FOR SALE NOR FOR REPRODUCTION.
ii

VISION

The Technological University of the Philippines shall be the premier state university
with recognized excellence in engineering and technology at par with leading universities in
the ASEAN region.

MISSION

The University shall provide higher and advanced vocational, technical, industrial,
technological and professional education and training in industries and technology, and in
practical arts leading to certificates, diplomas and degrees.
It shall provide progressive leadership in applied research, developmental studies in
technical, industrial, and technological fields and production using indigenous materials; effect
technology transfer in the countryside; and assist in the development of small-and-medium
scale industries in identified growth center. (Reference: P.D. No. 1518, Section 2)

QUALITY POLICY

The Technological University of the Philippines shall commit to provide quality higher
and advanced technological education; conduct relevant research and extension projects;
continually improve its value to customers through enhancement of personnel competence and
effective quality management system compliant to statutory and regulatory requirements; and
adhere to its core values.

CORE VALUES

T - Transparent and participatory governance


U - Unity in the pursuit of TUP mission, goals, and objectives
P - Professionalism in the discharge of quality service
I - Integrity and commitment to maintain the good name of the University
A - Accountability for individual and organizational quality performance
N - Nationalism through tangible contribution to the rapid economic growth of the
country
S - Shared responsibility, hard work, and resourcefulness in compliance to the mandates
of the university

This module is a property of Technological University of the Philippines Visayas and intended
for EDUCATIONAL PURPOSES ONLY and is NOT FOR SALE NOR FOR REPRODUCTION.
iii

TABLE OF CONTENTS
Page Numbers
TUP Vision, Mission, Quality Policy, and Core Values…………………..…… ii
Table of Contents……………………………………………………………..….. iii
Course Description……………………………………………………...…. vi
Learning Outcomes……………………………………………………...… vi
General Guidelines/Class Rules……………………………………...……. vi
Grading System………………………………………………………...….. vi
Learning Guide (Week No. 1) …………………………………………...... 1
Topic/s…………………………………………………………...… 1
Expected Competencies…………………………………………..... 1
Content/Technical Information…………………………………….. 1
Definitions of Basic Terms…………………………………. 1
Elimination of Arbitrary Constant……………………...….. 4
Families of Curves…………………………………………. 6
Progress Check…… ………………………………………………. 10
References…………………………………………………………. 14
Learning Guide (Week No. 2) …………………………………………..… 15
Topic/s……………………………………………………………... 15
Expected Competencies……………………………………………. 15
Content/Technical Information ……………………………………. 15
Methods for Solving Differential Equations of Order One… 16
Separable Equations……………………………….. 16
Homogeneous Functions…………………………… 21
Homogeneous Equations………………………….. 24
Linear Coefficients in Two Variables…………....… 29
Progress Check…… …………………………………………......... 32
References………………………………………………………...... 35
Learning Guide (Week No. 3&4) …………………………………………. 36
Topic/s………………………………………………………….….. 36
Expected Competencies…………………………………………..... 36
Content/Technical Information…………………………………….. 36
Exact Equations……………………………………………. 37
Integrating Factor by Inspection………………………...… 42
Integrating Factor by Formula………………………….… 45
Progress Check week 3………………………………………...….. 48
Linear Equations………………………………………..… 50
Bernoulli Equations………….……………………………. 53
Substitution Method……………………………………….. 56
Progress Check week 4……………………………………………. 59
References……………………………………………………...….. 61
Learning Guide (Week No. 6)………………….…………………………. 62
Topic/s………………………………………………………….…. 62
Expected Competencies………………………………………….... 62
This module is a property of Technological University of the Philippines Visayas and intended
for EDUCATIONAL PURPOSES ONLY and is NOT FOR SALE NOR FOR REPRODUCTION.
iv

Content/Technical Information………………………………….... 62
Applications of Differential Equations of Order One…….. 62
Newton’s Law of Cooling…………………………. 62
Growth and Decay……………………………..…. 66
Progress Check ……………………………………………………. 70
References……………………………………………………...…. 72
Learning Guide (Week No. 7&8)………………….……………..………. 73
Topic/s………………………………………………………….… 73
Expected Competencies…………………………………………... 73
Content/Technical Information…………………………………... 73
Differential Operators……………………………………. 73
Solutions of General Linear Homogeneous Equations…... 75
Distinct Real Roots of Auxiliary Equations………. 76
Repeated Real Roots of Auxiliary Equations……... 79
Distinct Imaginary Roots of Auxiliary Equations… 82
Repeated Imaginary Roots of Auxiliary Equations.. 84
Progress Check …………………………………………………….. 85
References……………………………………………………...….. 87
Learning Guide (Week No. 10)………………….……………..………. 88
Topic/s………………………………………………………….… 88
Expected Competencies…………………………………………... 88
Content/Technical Information…………………………………... 88
Non-homogeneous Linear Equations……………………. 88
Solutions of Equations having Constant Right Member….. 92
Progress Check …………………………………………………….. 95
References……………………………………………………...….. 97
Learning Guide (Week No. 11)………………….……………..………. 98
Topic/s………………………………………………………….… 98
Expected Competencies…………………………………………... 98
Content/Technical Information…………………………………... 98
Order Reduction……………………….…………………. 98
Variation of Parameters……………….…………………. 102
Inverse Differential Operator…………………………….. 104
Progress Check …………………………………………………….. 107
References……………………………………………………...….. 109
Learning Guide (Week No. 12)………………….……………..………. 110
Topic/s………………………………………………………….… 110
Expected Competencies…………………………………………... 110
Content/Technical Information…………………………………... 110
Applications of 2nd Order D.E………….…………………. 110
Simple Harmonic Motion……………….…………………. 110
Damped Vibrations………………….…………………….. 116
Overdamped Vibration ……………………………. 116
This module is a property of Technological University of the Philippines Visayas and intended
for EDUCATIONAL PURPOSES ONLY and is NOT FOR SALE NOR FOR REPRODUCTION.
v

Critically Damped Vibration………………………. 118


Undamped Vibration………………………………. 119
Forced Vibrations…………………………………………. 122
RCL Series Circuit………………………………………… 124
Progress Check …………………………………………………….. 128
References……………………………………………………...….. 130
Learning Guide (Week No. 13)………………….……………..………. 131
Topic/s………………………………………………………….… 131
Expected Competencies…………………………………………... 131
Content/Technical Information…………………………………... 131
The Laplace Transform………………….……………….. 131
Inverse Laplace Transform……………….………………. 138
Partial Fractions………………………………………….. 141
Progress Check …………………………………………………….. 144
References……………………………………………………...….. 145
List of References………………………………………………………………… 146
About the Compiler……………………………………………………………… 147

This module is a property of Technological University of the Philippines Visayas and intended
for EDUCATIONAL PURPOSES ONLY and is NOT FOR SALE NOR FOR REPRODUCTION.
vi

COURSE DESCRIPTION

This course is intended for all engineering students to have a firm foundation on differential
equations in preparation for their degree-specific advanced mathematic courses. It covers first
order differential equations, nth order linear differential equations and system of first order
linear differential equations. It also introduces the concept of Laplace Transforms in solving
differential equations. The students are expected to be able to recognize different kinds of
differential equations, determine the existence and uniqueness of solution, select the
appropriate methods of solution and interpret the obtained solution. Students are also expected
to relate differential equations to various practical engineering and scientific problem as well
as employ computer technology in solving and verifying solutions.

COURSE OUTCOMES

At the end of the fourteen-week course, the student must be able to:
1. Recognize different kinds of differential equations.
2. Use the appropriate methods of solving differential equations of order one and interpret the
obtained solution.
3. Use the appropriate methods of solving linear differential equations of nth order and interpret
the obtained solution.
4. Solve various practical engineering and scientific problems in applications of differential
equations of 1st and 2nd order.
5. Solve Laplace Transformations of functions.

GENERAL GUIDELINES/CLASS RULES

1. Written Exercises
a. Seatwork (one per week)
b. Quiz (one per week)
c. Exam (Prelim, Mid-Term, End-Term)
2. Problem Solving Activity
a. Board-work
b. Group
3. Policy on the attendance as stipulated in the student handbook will be observed in
case of late/absence.

GRADING SYSTEM

The student will be graded according to the following:

Average of examinations - 50%


Average of assessments - 50%

Prelim Grade : [(Prelim Exam x 0.50) + (Assessment x 0.50)]


Midterm Grade : [(Midterm Exam x 0.50) + (Assessment x 0.50)]
End term Grade : [(Final Exam x 0.50) + (Assessment x 0.50)]

This module is a property of Technological University of the Philippines Visayas and intended
for EDUCATIONAL PURPOSES ONLY and is NOT FOR SALE NOR FOR REPRODUCTION.
vii

Final Grade : (Prelim Grade x 0.30) + (Mid-term Grade x 0.30) + (End term
Grade x 0.40)]

The passing grade for this course is 5.0.

This module is a property of Technological University of the Philippines Visayas and intended
for EDUCATIONAL PURPOSES ONLY and is NOT FOR SALE NOR FOR REPRODUCTION.
1

LEARNING GUIDE

Week No.: __1__

TOPIC/S

I. Basic Concept
a. Definition of Basic Terms
II. Elimination of Arbitrary Constants
III. Family of Curves

EXPECTED COMPETENCIES
At the end of the week after studying the module, the student should be able to:
1. define differential equation;
2. identify the kind, order, degree and linearity of a differential equation; and
3. obtain the differential equation of the family of curves described

CONTENT/TECHNICAL INFORMATION

Introduction
Many physical applications involve the case of differential equations, dominantly in the
field of engineering, physics, and chemistry. Mathematical models are increasingly built up in
these areas especially in biology, physiology and economics.

Definitions of Basic Terms

A differential equation is an equation involving an unknown function and its


derivatives. The following are differential equations involving the unknown function y.

dy
i. = 5x + 3
dx

d2y dy
ii. x 2
+2 =1
dx dx

2 5
d3y d4y
iii. 4 3  + sin x 4  + 5 xy = 5 x + 3
 dx   dx 

This module is a property of Technological University of the Philippines Visayas and intended
for EDUCATIONAL PURPOSES ONLY and is NOT FOR SALE NOR FOR REPRODUCTION.
2

4 2
d3y d5y
7
 dy 
iv.  3  + 3 5  + y 3   = 6
 dx   dx   dx 

2z 2z
v. − =0
y 2 x 2

vi. 4𝑦 ′′′ − 5𝑦 ′′ − 6𝑦 = 0

Two Kinds of Differential Equations

• Ordinary Differential Equation – is a DE whose unknown function (dependent


variable) depends on only one independent variable.

• Partial Differential Equation – is a DE whose unknown function depends on two or


more independent variables.

Equations (i) through (iv) and (vi) are examples of ordinary differential equations, since
the unknown function 𝑦 depends only on a single variable 𝑥.

Order and Degree

The order of a differential equation is the order of highest derivative appearing in the
equation.
Equation (i) is of order 1; (ii) is of order 2; (iii) is of order 4; (iv) is of order 5, (v) is
of order 2; and (vi) is of order 3.

The degree of differential equation that can be written as polynomial in the unknown function
is the power to which the highest-order derivative is raised.
Equations (i), (ii), (v) and (vi) is of degree 1; (iii) is of degree 5; (iv) is of degree 2.

Not every differential equation has a degree. Example:


d2y dy
vii. ey 2
+2 =1
dx dx
dy
viii. + sin y = 0
dx
1
 dy  2
ix.   +y=x
 dx 

This module is a property of Technological University of the Philippines Visayas and intended
for EDUCATIONAL PURPOSES ONLY and is NOT FOR SALE NOR FOR REPRODUCTION.
3

Equation (vii) has no degree, as it cannot be written as polynomial in the unknown


function 𝑦 because of the expression 𝑒 𝑦 . Equation (viii) has no degree because of sin 𝑦; and
1
(ix) because of the function 2.
Linear Differential Equations

An nth order differential equation in the unknown function y and the independent
variable x is linear if it can be written in the form
dny d n −1 y
bn (x ) + ... + b1 (x ) + ybo (x ) = g (x )
dy
n
+ bn −1 n −1
dx dx dx
Differential equations that cannot be written in this form are called nonlinear.

Example 1. For the following differential equations, determine (a) order, (b) degree, (c)
linearity, (d) unknown function, (e) independent variable
1. ( y ) − 3 yy  + xy = 0
2

(a) 1
(b) 2
(c) nonlinear
(d) y
(e) x
2. x 4 y  + xy  = e x
(a) 3
(b) 1
(c) linear
(d) y
(e) x
3 2
 d 2r   d 2r  dr
3.  2  +  2  + y =0
 dt   dt  dy
(a) 2
(b) 3
(c) nonlinear
(d) r
(e) t, y

This module is a property of Technological University of the Philippines Visayas and intended
for EDUCATIONAL PURPOSES ONLY and is NOT FOR SALE NOR FOR REPRODUCTION.
4

ELIMINATION OF ARBITRARY CONSTANT

There are several ways of arriving at a differential equation. One way is by the method
called the elimination of arbitrary constants. This means that, in this section, we start with
the given function and find the corresponding equation, i.e. the function serves as the solution
for the differential equation. So, the solution is given first, and we need to find the problem
being the differential equation.

As a rule, for finding the differential equation, we simply differentiate the given
function or relation as many times depending on the number of constants present in the
function.

Example 2. Find the differential equation whose solution is given by

a. y 2 = 4ax
b. y = cx + c
c. x2 + y2 = r 2
d. y = ax 2 + bx + c
e. y = c1e − x + c 2 e 2 x

Solutions to Example 2.
a. y 2 = 4ax
There is only one constant 𝑎, so we need to differentiate the given equation only
once; i.e.
1
2 yy  = 4a , or a = yy 
2
Substituting this value of 𝑎 in the given equation gives
 yy  
y 2 = 4  x or y = 2 xy 
 2 
The result, being the last equation, may also be written in other forms such as:
dy
y = 2 x , or ydx = 2 xdy , or ydx − 2 xdy = 0
dx
This means the differential equation ydx − 2 xdy = 0 has the solution y 2 = 4ax .

b. y = cx + c
Differentiating, we have y  = c , substituting in the given, gives
y = y x + y  , or y = y (x + 1)

This module is a property of Technological University of the Philippines Visayas and intended
for EDUCATIONAL PURPOSES ONLY and is NOT FOR SALE NOR FOR REPRODUCTION.
5

c. x2 + y2 = r 2
By differentiating, we have 2 xdx + 2 ydy = 0 , or xdx + ydy = 0

d. y = ax 2 + bx + c
The given equation has 3 constants, thus the differentiation is done thrice, i.e.
y  = 2ax + b
y  = 2 a
y  = 0 , the required equation.

e. y = c1e − x + c 2 e 2 x (1)
The differentiation is done twice, i.e.
y  = −c1e − x + 2c2 e 2 x (2)
y  = c1e − x + 4c 2 e 2 x (3)

There are then 3 equations in unknowns 𝑐1and 𝑐2 . Eliminating 𝑐1 in (1) and (2) by
addition gives
y + y  = 3c 2 e 2 x (4)

Eliminating 𝑐1in (2) and (3), by addition, we obtain


y  + y  = 6c 2 e 2 x (5)

Eliminate 𝑐2 in equations (4) and (5) by multiplying (4) by 2, we have


2 y + 2 y  = 6c 2 e 2 x
y  + y  = 6c 2 e 2 x

By subtraction we have the desired equation


2 y + 2 y  − y  − y  = 0 , or 2 y + y  − y  = 0 , or y  = 2 y + y 

This module is a property of Technological University of the Philippines Visayas and intended
for EDUCATIONAL PURPOSES ONLY and is NOT FOR SALE NOR FOR REPRODUCTION.
6

FAMILIES OF CURVES
If an equation involves a parameter, then it represents a family of curves, with one
curve for each particular value of the parameter. The following equations illustrate family of
curves.

Family of lines
1. 𝑦 = 𝑐𝑥 passing through the
(one parameter) origin

Figure 1.1

Family of
2. 𝑦 = 𝑎𝑥 2
parabolas with axis
(one parameter)
as 𝑦

Figure 1.2

Family of
2 2 2
3. 𝑥 + 𝑦 = 𝑟 concentric circles
(one parameter) with centers at the
origin

Figure 1.3

This module is a property of Technological University of the Philippines Visayas and intended
for EDUCATIONAL PURPOSES ONLY and is NOT FOR SALE NOR FOR REPRODUCTION.
7

4. 𝑥 2 + (𝑦 − 𝑘)2 = 𝑟 2 Family of circles


(2 parameters) whose centers on
𝑦-axis

Figure 1.4

Family of circles
5. (𝑥 − ℎ)2 + 𝑦 2 = 𝑟 2
whose centers of
(two parameters)
the 𝑥-axis

Figure 1.5

Family of circles
6. (𝑥 − 𝑐)2 + (𝑦 − 𝑐)2 = with center on the
2𝑐 2 line 𝑦 = 𝑥 and
(one parameter) passing through the
origin
Figure 1.6

𝑥 𝑦
7. 𝑥 + 𝑦 = 𝑎, or + = Family of lines
𝑎 𝑎
1 𝑎=3 with equal
(one parameter) 𝑎=2 intercepts
𝑎=1
Figure 1.7 𝑎 = −1

This module is a property of Technological University of the Philippines Visayas and intended
for EDUCATIONAL PURPOSES ONLY and is NOT FOR SALE NOR FOR REPRODUCTION.
8

𝑚=3
𝑚=4 𝑚=2

𝑚=1

Family of lines
8. 𝑦 = 𝑎𝑥 + 𝑎
with slope and 𝑦
(one parameter)
intercept equal

Figure 1.8

Family of lines
through (ℎ, 𝑘), ℎ
𝑦−ℎ
9. =𝑚 and 𝑘 are not
𝑥−ℎ
parameters but
fixed constants

Figure 1.9

Family of circles
2 2
10. (𝑥 − ℎ) + (𝑦 − 𝑘) = with center at
𝑘2 (ℎ, 𝑘) and tangent
(two parameters) of the 𝑥-axis i.e.,
𝑘 = radius

Example 3. Obtain the differential equation of the family of lines through the origin.

Solution to Example 3.
The required equation satisfied by the given condition of the problem is
𝑦 = 𝑐𝑥 (1)
Upon differentiation, we have
𝑦 ′ = 𝑐 (2)
Substituting equation (2) to (1),
𝑦 = (𝑦 ′ )𝑥
This preceding equation may also be written as
𝒅𝒚
𝒚= 𝒙 𝑜𝑟 𝒚 𝒅𝒙 = 𝒙 𝒅𝒚 𝑜𝑟 𝒚𝒅𝒙 − 𝒙𝒅𝒚 = 𝟎
𝒅𝒙

This module is a property of Technological University of the Philippines Visayas and intended
for EDUCATIONAL PURPOSES ONLY and is NOT FOR SALE NOR FOR REPRODUCTION.
9

Example 4. Obtain the differential equation of the family of circles having their centers on
the 𝑥-axis.

Solution to Example 4.
If (ℎ, 𝑘) represents the center of each circle, then 𝑘 = 0. Since every center is on the
𝑥-axis, then the required equation is
(𝑥 − ℎ)2 + 𝑦 2 = 𝑟 2
where ℎ and 𝑟 vary, i.e., being parameters. Since there are two parameters, then
differentiation must be done twice,
1st differentiation: 2(𝑥 − ℎ) + 2𝑦𝑦 ′ = 0
ℎ = 𝑦𝑦 ′ + 𝑥

2nd differentiation: 0 = 𝑦𝑦" + 𝑦′𝑦′ + 1


Thus, the required equation is
𝒚𝒚" + (𝒚′)𝟐 + 𝟏 = 𝟎

Figure 1.10

This module is a property of Technological University of the Philippines Visayas and intended
for EDUCATIONAL PURPOSES ONLY and is NOT FOR SALE NOR FOR REPRODUCTION.
10

PROGRESS CHECK
Activity Sheets
EXERCISE 1
Review on Definitions of a D.E., Elimination of Arbitrary Constants and Family of
Curves

Name: _____________________________________________Year & Section: __________


Subject: ____________________________________________Date Submitted: __________
Professor: __________________________________________Score: __________________

For numbers 1 to 9, for the following differential equations, determine (a) order, (b)
degree, (c) linearity, (d) unknown function, and (e) independent variables

𝑑3 𝑦 𝑑𝑦
1. 𝑥 +7 − 7𝑦 = 6
𝑑𝑥 3 𝑑𝑥
(a)
(b)
(c)
(d)
(e)

2. 𝑥 2 𝑦 ′′ − 𝑥𝑦 ′ = 1 + cos 𝑦
(a)
(b)
(c)
(d)
(e)

3 2
𝑑2 𝑞 𝑑2 𝑞 𝑑𝑞
3. ( 2
) +( 2
) +𝑦 =3
𝑑𝑟 𝑑𝑟 𝑑𝑦
(a)
(b)
(c)
(d)
(e)

𝑑𝑦 4
4. ( ) − 3𝑦 2 = 𝑒 𝑦
𝑑𝑥
(a)
(b)
(c)
(d)
(e)

This module is a property of Technological University of the Philippines Visayas and intended
for EDUCATIONAL PURPOSES ONLY and is NOT FOR SALE NOR FOR REPRODUCTION.
11

𝑑2 𝑥 𝑑𝑥 3
5. 2
− 2𝑥𝑡 = ( )
𝑑𝑡 𝑑𝑡
(a)
(b)
(c)
(d)
(e)

𝑑3𝑣 𝑑𝑣
6. 𝑧 2 −𝑧 = 1 − cos 𝑧
𝑑𝑧 3 𝑑𝑡
(a)
(b)
(c)
(d)
(e)

𝑑𝑏 7
7. ( ) = 3𝑝
𝑑𝑝
(a)
(b)
(c)
(d)
(e)

8. 𝑥𝑦 ′′′ − 𝑥 2 𝑦 ′′ − 𝑥 3 𝑦 ′ − 𝑦 = 𝑥
(a)
(b)
(c)
(d)
(e)

𝑑𝑛 𝑥
9. = 𝑦2 + 1
𝑑𝑦 𝑛
(a)
(b)
(c)
(d)
(e)

This module is a property of Technological University of the Philippines Visayas and intended
for EDUCATIONAL PURPOSES ONLY and is NOT FOR SALE NOR FOR REPRODUCTION.
12

For numbers10 to 15, Elimination of Arbitrary Constants: Find the differential equation
whose solutions correspond to the following equation.

10. 3𝑥 2 − 𝑥𝑦 2 = 𝑐 13. (𝑥 − 𝑟)2 + 𝑦 2 = 𝑟 2

11. 𝑦 = ∅ cos(𝑥 + 𝜃) 14. 𝐴𝑥 3 + 𝐵𝑥 2 − 𝑦 4 = 0

12. 𝑦 = 𝑐1 𝑒 −𝑥 + 𝑐2 𝑒 −2𝑥 15. (𝑥 − ℎ)2 + 𝑦 2 = 𝑟 2

This module is a property of Technological University of the Philippines Visayas and intended
for EDUCATIONAL PURPOSES ONLY and is NOT FOR SALE NOR FOR REPRODUCTION.
13

For numbers 16 to 20, obtain the differential equation of the family of curves described
and sketch some members of the family.

16. Circles with center at the origin.

17. Circles with centers on 𝑦 = 𝑥 and passing through the origin.

18. Parabolas with vertex and focus on the 𝑥-axis opening in the right.

19. Parabolas with vertex and focus on the 𝑦-axis opening upward.

20. Circles tangent to the 𝑦-axis.

This module is a property of Technological University of the Philippines Visayas and intended
for EDUCATIONAL PURPOSES ONLY and is NOT FOR SALE NOR FOR REPRODUCTION.
14

REFERENCES
Sergio E. Ymas Jr. Differential Equations (Simplified Approach). Sampaloc, Manila. Pablo.
L. Bustamante III Press, 1997.

Louis Leithold. The Calculus with Analytic Geometry. 6th ed. Harper & Row Publishers Inc.,
1990.

Clyde E. Love, Earl D. Rainville. Differential and Integral Calculus. 6th ed. The Macmillan
Co., 1961.

Ricardo C. Asin. Elementary Differential Equations Reviewer. Manila, Philippines, Merriam


& Webster Bookstore, Inc., 1991.

This module is a property of Technological University of the Philippines Visayas and intended
for EDUCATIONAL PURPOSES ONLY and is NOT FOR SALE NOR FOR REPRODUCTION.
15

LEARNING GUIDE

Week No.: __2__

TOPIC/S

I. Methods for Solving Differential Equations of Order One


a. Separable Differential Equations
b. Homogeneous Equations
c. Linear Coefficients in Two Variables

EXPECTED COMPETENCIES
At the end of the week after studying the module, the student should be able to:
1. solve solutions of first order differential equations using the method Separation of
Variables;
2. determine the homogeneity of the functions and state the degree of homogeneity;
3. solve the general solution of the function and the particular solution by determining
the constants of integration when initial conditions are indicated;
4. solve solutions of first order differential equations using Homogenous Equation
method; and
5. solve solutions of first order differential equations using Linear Coefficients in Two
Variables method.

CONTENT/TECHNICAL INFORMATION
Prerequisite
Before going through the lesson, recall the following Integration Techniques:
1. Indefinite Integrals
a. Powers
b. Logarithms
c. Exponential Functions
d. Trigonometric Functions
e. Transformation by Trigonometric Formulas
f. Inverse Trigonometric Functions
g. Integration by Parts
h. Algebraic Substitution
i. Trigonometric Substitution
j. Distinct Linear Factors
k. Repeated Linear Factors
l. Quadratic Factors

This module is a property of Technological University of the Philippines Visayas and intended
for EDUCATIONAL PURPOSES ONLY and is NOT FOR SALE NOR FOR REPRODUCTION.
16

Introduction

In the previous lesson, we are given the relation of function suitable as the solution for
a certain differential equation to be found. This lesson gives the reverse process. Here, the
differential equation is given, and we need to find the corresponding solution in terms of an
equation of a function or relation.

METHODS FOR SOLVING DIFFERENTIAL EQUATIONS OF ORDER ONE

A solution of a differential equation in the unknown function 𝑦 and the independent


variable 𝑥 is a function 𝑦 (𝑥) that satisfies the differential equation.

One main objective of differential equation is to solve equations i.e. to find the solution
in terms of a function or relation. Different methods may be employed in solving equations.

Three methods of solving first-order differential equations:


• Separation of Variables
• Homogeneous Equations
• Linear Coefficients in Two Variables

Any differential equation of order one may be written in any of the following forms:

𝑴 (𝒙, 𝒚)𝒅𝒙 + 𝑵(𝒙, 𝒚)𝒅𝒚 = 𝟎, or

𝑴 + 𝑵 𝒚′ = 𝟎, or

𝑴 (𝒙, 𝒚)
𝒚′ + =𝟎
𝑵 (𝒙, 𝒚)

Separable Differential Equations

There are differential equations 𝑴 (𝒙, 𝒚)𝒅𝒙 + 𝑵 (𝒙, 𝒚)𝒅𝒚 with variables separable.
The first-order separable differential equation is of the form

𝑨(𝒙)𝒅𝒙 + 𝑩(𝒚)𝒅𝒚 = 𝟎

This is called the separable equation.

The solution of this equation is given by ∫ 𝑨(𝒙)𝒅𝒙 + ∫ 𝑩(𝒚)𝒅𝒚 = 𝑪.

This module is a property of Technological University of the Philippines Visayas and intended
for EDUCATIONAL PURPOSES ONLY and is NOT FOR SALE NOR FOR REPRODUCTION.
17

Example 1. Solve for 𝒚’ = 𝒚𝒙𝟑

Solution to Example 1.
We first rewrite this equation in differential form.
𝑑𝑦
= 𝑦𝑥 3
𝑑𝑥
Check whether the equation is separable or not. Separate the variable by rearranging
the variables to their respective functions. So, it shows
𝑑𝑦
𝑥 3 𝑑𝑥 − =0
𝑦

The solution of this separable equation is


𝑑𝑦
∫ 𝑥 3 𝑑𝑥 − ∫ =𝐶
𝑦
Integrating, we obtain
𝑥4
− ln 𝑦 = ln 𝐶
4

Take Note: The constant 𝐶 may take any form such as ln 𝐶, to give a better form of a
solution.

Simplifying the above equation gives

𝑥 4 = 4(ln 𝑦 + ln 𝐶)
𝒙𝟒 = 𝟒 𝐥𝐧 𝑪𝒚 general solution

Since the constant 𝐶 may take any form, we could have many possible general
solutions but will only have one particular solution under conditions. Take example number
2.

Example 2. Solve 𝑥𝑦𝑦 ′ − 𝑦 2 = 1, when 𝑥 = 2, 𝑦 = 1.

Solution to Example 2.
First rewrite the equation in differential form
𝑑𝑦
𝑥𝑦 − 𝑦2 = 1
𝑑𝑥
Separate the variables,
𝑥𝑦 𝑑𝑦 − 𝑦 2 𝑑𝑥 = 𝑑𝑥
𝑥𝑦 𝑑𝑦 = (𝑦 2 + 1) 𝑑𝑥
𝑦 𝑑𝑦 𝑑𝑥
=
(𝑦 2 + 1) 𝑥
This module is a property of Technological University of the Philippines Visayas and intended
for EDUCATIONAL PURPOSES ONLY and is NOT FOR SALE NOR FOR REPRODUCTION.
18

The solution of this separable equation is


𝑦 𝑑𝑦 𝑑𝑥
∫ 2 = ∫
(𝑦 + 1) 𝑥
Integrating, we obtain
1 Take note we use +C as
ln(𝑦 2 + 1) = ln 𝑥 + 𝐶
2 C may take any form
ln(𝑦 2 + 1) − 2 ln 𝑥 = 2𝐶
ln(𝑦 2 + 1) − ln 𝑥 2 = 2𝐶
𝑦2 + 1
ln = 2𝐶
𝑥2
𝑦2 + 1
2
= 𝑒 2𝐶
𝑥
𝒚𝟐 + 𝟏 = 𝒙𝟐 𝒆𝟐𝑪 General solution (1)
using +C form
This time, using ln 𝐶
1
ln(𝑦 2 + 1) = ln 𝑥 + ln 𝐶
2 Take note we use lnC
ln(𝑦 2 + 1) = 2 ln 𝑥 + 2 ln 𝐶 as C may take any form
ln(𝑦 2 + 1) = ln 𝑥 2 + ln 𝐶 2
ln(𝑦 2 + 1) = ln 𝑥 2 𝐶 2 General solution (2)
𝒚𝟐 + 𝟏 = 𝒙 𝟐 𝑪 𝟐 using lnC form

Notice the difference in the forms of general solutions (1) and (2). Now, using the conditions
that when 𝑥 = 2, 𝑦 = 1 get the particular solution.

Using general solution (1)


𝑦 2 + 1 = 𝑥 2 𝑒 2𝐶
Substitute the values of 𝑥 and 𝑦 to the equation
(1)2 + 1 = (2)2 𝑒 2𝐶
Getting the value of 𝐶,
2 = 4𝑒 2𝐶
1
= 𝑒 2𝐶
2
1
ln = ln 𝑒 2𝐶
2
1
ln = 2𝐶 ln 𝑒 ; ln 𝑒 = 1
2
1 1 Substitute the value of C to
ln = 𝐶
2 2 the general solution (1) for
the particular solution
Substitute C to the general solution (1)
1 1
𝑦 2 + 1 = 𝑥 2 𝑒 2(2 ln2)
1
𝑦 2 + 1 = 𝑥 2 𝑒 ln2
𝑦2 + 1 1

2
= 𝑒 ln2
𝑥
This module is a property of Technological University of the Philippines Visayas and intended
for EDUCATIONAL PURPOSES ONLY and is NOT FOR SALE NOR FOR REPRODUCTION.
19

𝑦2 + 1 ln
1
ln = ln 𝑒 2
𝑥2
𝑦2 + 1 1
ln 2
= ln
𝑥 2
Particular Solution
𝟐 𝟐
𝟐(𝒚 + 𝟏) = 𝒙

Using general solution (2)


𝑦2 + 1 = 𝑥2𝐶 2
(1)2 + 1 = (2)2 𝐶 2
1
= 𝐶2
2
Notice the difference of the
1
√ =𝐶 values of C in general
2 solution (1)

Substituting 𝐶 to general solution (2)


𝑦2 + 1 = 𝑥2𝐶 2
1
𝑦2 + 1 = 𝑥2 ( )
2 Particular Solution
𝟐(𝒚 + 𝟏) = 𝒙𝟐
𝟐

As stated earlier, there will have many forms of general solution but there could only be one
particular solution under given conditions.

Example 3. Solve 𝒆𝒚 𝒅𝒙 − 𝒙𝟐 𝒅𝒚 = 𝟎

Solution to Example 3.
By separation of variables, the solution to this differential equation is given by

𝑑𝑥 𝑑𝑦
∫ 2
− ∫ 𝑦 =0
𝑥 𝑒
Rewriting the equation,

∫ 𝑥 −2 𝑑𝑥 − ∫ 𝑒 −𝑦 𝑑𝑦 = 0
Integrating, we obtain
−𝑥 −1 + 𝑒 −𝑦 = 𝐶
Or
1
+ 𝑒 −𝑦 = 𝐶
𝑥
Applying 𝑙𝑛 on both sides, we have
1
−𝑦 ln 𝑒 = ln (𝐶 − )
𝑥
𝟏
𝒚 = − 𝐥𝐧 (𝑪 − 𝒙) general solution
This module is a property of Technological University of the Philippines Visayas and intended
for EDUCATIONAL PURPOSES ONLY and is NOT FOR SALE NOR FOR REPRODUCTION.
20

Example 4. Solve 𝐭𝐚𝐧𝟐 𝒙 𝐜𝐬𝐜 𝟑 𝒚 𝒅𝒙 = 𝒅𝒚

Solution to Example 4.
By separation of variables, we get
tan2 𝑥 𝑑𝑥 = sin3 𝑦 𝑑𝑦
By integrating both sides of the equation,
∫ 𝑡𝑎𝑛2 𝑥 𝑑𝑥 = ∫ sin3 𝑦 𝑑𝑦
Rewriting the equation, using trigonometric identities,
∫(sec 2 𝑥 − 1)𝑑𝑥 = ∫ sin 𝑦 (1 − cos 2 𝑦)𝑑𝑦

tan 𝑥 − 𝑥 = ∫ sin 𝑦 𝑑𝑦 − ∫ cos2 𝑦 sin 𝑦 𝑑𝑦


𝐜𝐨𝐬 𝟑 𝒚
𝐭𝐚𝐧 𝒙 − 𝒙 = − 𝐜𝐨𝐬 𝒚 + +𝑪 general solution
𝟑

Example 5. Solve 𝒙𝒚𝟑 𝒅𝒙 + (𝒚 + 𝟏)𝒆−𝒙 𝒅𝒚 = 𝟎

Solution to Example 5.
Separating the variables, we have,
𝑥 𝑑𝑥 (𝑦 + 1) 𝑑𝑦
+ =0
𝑒 −𝑥 𝑦3
𝑦 1
∫ 𝑥𝑒 𝑥 𝑑𝑥 + ∫ ( 3 + 3 ) 𝑑𝑦 = 0
𝑦 𝑦
∫ 𝑥𝑒 𝑥 𝑑𝑥 + ∫ 𝑦 −2 𝑑𝑦 + ∫ 𝑦 −3 𝑑𝑦 = 0
To integrate 𝑥𝑒 𝑥 , we use integration by parts:
𝑢=𝑥 ∫ 𝑑𝑣 = ∫ 𝑒 𝑥 𝑑𝑥
𝑑𝑢 = 𝑑𝑥 𝑣 = 𝑒𝑥

𝑥𝑒 𝑥 𝑑𝑥 = 𝑥𝑒 𝑥 − ∫ 𝑒 𝑥 𝑑𝑥
= 𝑥𝑒 𝑥 − 𝑒 𝑥 + 𝐶
Thus, the required solution is
𝟏 𝟏
𝒙𝒆𝒙 − 𝒆𝒙 − − 𝟐=𝑪
𝒚 𝟐𝒚

This module is a property of Technological University of the Philippines Visayas and intended
for EDUCATIONAL PURPOSES ONLY and is NOT FOR SALE NOR FOR REPRODUCTION.
21

Homogeneous Functions

Homogeneous polynomials is defined as those polynomials all of whose terms are of


the same degree. The constant exponent that appear the same in every term of variables 𝑥 and
𝑦 is said to be the degree of homogeneous equations.

The following illustrate homogeneous polynomials:

𝑥3 + 𝑥2𝑦 + 𝑦3 degree 3

𝑥 2 + 𝑥𝑦 + 𝑦 2 degree 2

𝑥 4 + 𝑥 3 𝑦 + 𝑥𝑦 3 + 𝑦 4 degree 4

𝑥6 + 𝑥5𝑦 + 𝑥3𝑦3 degree 6

There are other functions which do not appear so obvious in their terms as
homogeneous.
𝑦 𝑦
𝑓 (𝑥, 𝑦) = 𝑥 arctan 𝑥 − 𝑦 ln 𝑥 degree 1

𝑦 𝑥3
𝑓(𝑥, 𝑦) = 2𝑦 2 exp (𝑥 ) − degree 2
𝑥+𝑦

The degree of a homogeneous function need not be a positive

1
f ( x, y) = x + y − x degree 2
x2 1
f ( x, y ) = degree 2
x3 + y3
x
f ( x, y ) = degree 0
x + y2
2

1 1
f ( x, y ) = degree − 2
x+ y

The function 𝑓 (𝑥, 𝑦) is said to be homogeneous of degree 𝑘 in 𝑥 and 𝑦 if it can be


written in the form

𝑓 (𝜆𝑥, 𝜆𝑦) = 𝜆𝑘 𝑓(𝑥, 𝑦)

This module is a property of Technological University of the Philippines Visayas and intended
for EDUCATIONAL PURPOSES ONLY and is NOT FOR SALE NOR FOR REPRODUCTION.
22

Example 6. Determine the homogeneity of the following functions. If it is homogeneous,


state the degree of the function.

x
a. e. e x
x+ y
x
y f. x 3 − xy + y 3
b. tan + e y
x
x+ y g. ln(x − y )
c.
x− y

d. ln x − ln y

Solutions to Example 6.
𝑥
a. Let 𝑓(𝑥, 𝑦) =
√𝑥−𝑦
Substituting 𝜆𝑥 and 𝜆𝑦 in 𝑥 and 𝑦 we have the following:
𝜆𝑥 𝜆𝑥
𝑓 (𝜆𝑥, 𝜆𝑦) = = 1
√𝜆𝑥 − 𝜆𝑦 𝜆2 √𝑥 − 𝑦
1 𝑥
= 𝜆2 ( )
√𝑥 − 𝑦
1
= 𝜆2 𝑓(𝑥, 𝑦)

Thus, the function is homogeneous of degree ½

𝑥
𝑦
b. 𝑓(𝑥, 𝑦) = tan 𝑥 + 𝑒 𝑦
𝜆𝑦 𝜆𝑥
𝑓(𝜆𝑥, 𝜆𝑦) = tan + 𝑒 𝜆𝑦
𝜆𝑥
0
𝑦 𝑥
= 𝜆 (tan +𝑒 )𝑦
𝑥
= 𝜆0 𝑓(𝑥, 𝑦)

Thus, the function is homogeneous of degree 0.

𝑥+𝑦
c. 𝑓(𝑥, 𝑦) = 𝑥−𝑦
𝜆𝑥 + 𝜆𝑦
𝑓(𝜆𝑥, 𝜆𝑦) =
𝜆𝑥 − 𝜆𝑦
𝜆(𝑥 + 𝑦)
=
𝜆(𝑥 − 𝑦)
𝑥+𝑦
= 𝜆0 ( )
𝑥−𝑦
This module is a property of Technological University of the Philippines Visayas and intended
for EDUCATIONAL PURPOSES ONLY and is NOT FOR SALE NOR FOR REPRODUCTION.
23

= 𝜆0 𝑓(𝑥, 𝑦)

Thus, the function is homogeneous of degree 0.

𝑥
d. 𝑓(𝑥, 𝑦) = ln 𝑥 − ln 𝑦 = ln 𝑦
𝜆𝑥 𝑥 𝑥
𝑓(𝜆𝑥, 𝜆𝑦) = ln = ln ( ) = 𝜆0 ln
𝜆𝑦 𝑦 𝑦

Thus, the function is homogeneous of degree 0.

e. The function is not homogeneous, since we cannot take or do anything with 𝜆 in


𝑒 𝜆𝑥 .
f. The polynomial is not homogeneous, since it has no constant degree in each term.
g. The function is not also homogeneous, since 𝜆 cannot be taken as there is no
certain value of 𝑘 to satisfy by the definition of homogeneity.

To determine the homogeneity of a differential equation

𝑴(𝒙, 𝒚)𝒅𝒙 + 𝑵 (𝒙, 𝒚) 𝒅𝒚 = 𝟎


set
𝑓 (𝑥, 𝑦) = 𝑀 (𝑥, 𝑦) + 𝑁 (𝑥, 𝑦)

and substitute 𝜆𝑥 for 𝑥 and 𝜆𝑦 for 𝑦.

Example 7. Determine the homogeneity of the following differential equations:


a. (𝑦 2 − 𝑥𝑦)𝑑𝑥 − 𝑥 2 𝑑𝑦 = 0
b. (𝑥 − 𝑦 ln 𝑦 + 𝑦 ln 𝑥)𝑑𝑥 + 𝑥(ln 𝑦 − ln 𝑥)𝑑𝑦 = 0
𝑦
c. 𝑥𝑑𝑥 − sin2 (𝑥 ) (𝑦𝑑𝑥 − 𝑥𝑑𝑦) = 0

Solutions to Example 7.
a. Set 𝑓(𝑥, 𝑦) = 𝑦 2 − 𝑥𝑦 − 𝑥 2 . Substituting 𝜆𝑥 for 𝑥 and 𝜆𝑦 for 𝑦:
𝑓(𝜆𝑥, 𝜆𝑦) = (𝜆𝑦)2 − (𝜆𝑥)(𝜆𝑦) − (𝜆𝑥)2
= 𝜆2 𝑦 2 − 𝜆2 𝑥𝑦 − 𝜆2 𝑥 2
= 𝜆2 (𝑦 2 − 𝑥𝑦 − 𝑥 2 )
= 𝜆2 𝑓(𝑥, 𝑦)
Thus, the differential equation is homogeneous of degree 2.

𝑦 𝑦
b. Set 𝑓(𝑥, 𝑦) = 𝑥 − 𝑦(ln 𝑦 − ln 𝑥) + 𝑥(ln 𝑦 − ln 𝑥) or 𝑥 − 𝑦 ln 𝑥 + 𝑥 ln 𝑥 .

This module is a property of Technological University of the Philippines Visayas and intended
for EDUCATIONAL PURPOSES ONLY and is NOT FOR SALE NOR FOR REPRODUCTION.
24

𝜆𝑦 𝜆𝑦
𝑓(𝜆𝑥, 𝜆𝑦) = 𝜆𝑥 − 𝜆𝑦 ln
+ 𝜆𝑥 ln
𝜆𝑥 𝜆𝑥
𝑦 𝑦
= 𝜆 (𝑥 − 𝑦 ln + 𝑥 ln )
𝑥 𝑥
= 𝜆 𝑓(𝑥, 𝑦)
Thus, the differential equation is homogeneous of degree one.

𝑦
c. Set 𝑓(𝑥, 𝑦) = 𝑥 sin2 (𝑥 ) (𝑦 − 𝑥), and substitute 𝜆𝑥 for 𝑥 and 𝜆𝑦 for 𝑦:
𝜆𝑦
𝑓(𝜆𝑥, 𝜆𝑦) = 𝜆𝑥 − sin2 ( ) (𝜆𝑦 − 𝜆𝑥)
𝜆𝑥
𝑦
= 𝜆 (𝑥 − sin2 ( ) (𝑦 − 𝑥))
𝑥
= 𝜆 𝑓(𝑥, 𝑦)
Thus, the differential equation is homogeneous of degree one.

Homogeneous Equations

There are other equations whose variables are not separable. For example, in (𝑥 +
𝑦)𝑑𝑥 + 𝑥𝑑𝑦 = 0, the presence of 𝑥 + 𝑦 indicates that the variables 𝑥 and 𝑦 are not
separable.

Consider a differential equation of order one,

𝑀 (𝑥, 𝑦)𝑑𝑥 + 𝑁 (𝑥, 𝑦)𝑑𝑦 = 0

where both 𝑀 and 𝑁 are homogeneous functions of the same degree in each term
involving 𝑥 and 𝑦.

Let 𝒙 = 𝒗𝒚, or 𝒚 = 𝒗𝒙 depending on the convenience of the solution. Upon


substituting 𝑥 = 𝑣𝑦 and its derivative 𝑑𝑥 = 𝑣𝑑𝑦 + 𝑦𝑑𝑣, the resulting differential equation
will be in terms of the variables 𝑣 and 𝑦. After simplifying, the variables are separable. It is
very important to note that the solution of any homogeneous equation will usually lead to
the separation of variables.

Example 8. Solve the differential equation



𝑥2 + 𝑦2
𝑦 =
𝑥𝑦
Solution to Example 8.
This may also be written as
(𝑥 2 + 𝑦 2 )𝑑𝑥 − 𝑥𝑦𝑑𝑦 = 0

This module is a property of Technological University of the Philippines Visayas and intended
for EDUCATIONAL PURPOSES ONLY and is NOT FOR SALE NOR FOR REPRODUCTION.
25

You can use to let either 𝑦 = 𝑣𝑥 or 𝑥 = 𝑣𝑦. Since 𝑑𝑦 has lesser number of terms,
we let 𝑦 = 𝑣𝑥 and 𝑑𝑦 = 𝑣𝑑𝑥 + 𝑥𝑑𝑣 in 𝑦 and 𝑑𝑦, respectively gives the following
computations:
(𝑥 2 + 𝑣 2 𝑥 2 )𝑑𝑥 − 𝑥 (𝑣𝑥)(𝑣𝑑𝑥 + 𝑥𝑑𝑣) = 0,

𝑥 2 𝑑𝑥 + 𝑥 2 𝑣 2 𝑑𝑥 − 𝑥 2 𝑣 2 𝑑𝑥 − 𝑥 3 𝑣𝑑𝑣 = 0,

𝑥 2 𝑑𝑥 − 𝑥 3 𝑣𝑑𝑣 = 0,

After simplifying, the variables are separable

𝑑𝑥
= 𝑣𝑑𝑣
𝑥
Integrating, we get

𝑑𝑥
∫ = ∫ 𝑣𝑑𝑣
𝑥

𝑣2
ln 𝑥 = + ln 𝐶
2
𝑦
Since 𝑣 = ,
𝑥
𝑦2
2ln 𝑥 + ln 𝐶 = 2
𝑥

(𝑥 2 ) ln 𝑥 2 𝐶 = 𝑦 2

𝒚𝟐 = (𝒙𝟐 ) 𝐥𝐧 𝒙𝟐 𝑪 general solution

Reminder: The constant 𝐶 may take any form.

Alternate Solution to Example 8.


Solving the differential equation by using 𝑥 = 𝑣𝑦 and 𝑑𝑥 = 𝑣𝑑𝑦 + 𝑦𝑑𝑣, and try to
compare the process to the first solution.

(𝑥 2 + 𝑦 2 )𝑑𝑥 − 𝑥𝑦𝑑𝑦 = 0
((𝑣𝑦)2 + 𝑦 2 )(𝑣𝑑𝑦 + 𝑦𝑑𝑣) − (𝑣𝑦)𝑦𝑑𝑦 = 0
(𝑣 2 𝑦 2 + 𝑦 2 )(𝑣𝑑𝑦 + 𝑦𝑑𝑣) − 𝑣𝑦 2 𝑑𝑦 = 0
𝑣 3 𝑦 2 𝑑𝑦 + 𝑣𝑦 2 𝑑𝑦 + 𝑣 2 𝑦 3 𝑑𝑣 + 𝑦 3 𝑑𝑣 − 𝑣𝑦 2 𝑑𝑦 = 0
𝑣 3 𝑦 2 𝑑𝑦 + 𝑦 3 (𝑣 2 + 1)𝑑𝑣 = 0

This module is a property of Technological University of the Philippines Visayas and intended
for EDUCATIONAL PURPOSES ONLY and is NOT FOR SALE NOR FOR REPRODUCTION.
26

By separation of variables, we have


𝑑𝑦 (𝑣 2 + 1)𝑑𝑣
+ =0
𝑦 𝑣3
𝑑𝑦 1 1
∫ + ∫ ( + 3 ) 𝑑𝑣 = 0
𝑦 𝑣 𝑣
𝑑𝑦 𝑑𝑣
∫ +∫ + ∫ 𝑣 −3 𝑑𝑣 = 0
𝑦 𝑣
Integrating gives
1
ln 𝑦 + ln 𝑣 − = ln 𝐶
2𝑣 2
1
ln 𝑦 + ln 𝑣 + ln 𝐶 = 2
2𝑣
1
ln 𝑦𝑣𝐶 = 2
2𝑣
Substituting 𝑥/𝑦 for 𝑣, we have
𝑥 1
ln 𝑦 ( ) 𝐶 =
𝑦 𝑥 2
2 (𝑦)
𝑦2
2 ln 𝑥𝐶 =
𝑥2
𝒙 𝐥𝐧 𝒙 𝑪 = 𝒚𝟐
𝟐 𝟐

Notice when it comes to integrating this is harder compared to the first solution.

Example 9. Solve the equation


𝑦𝑑𝑥 = (𝑥 + √𝑦 2 − 𝑥 2 ) 𝑑𝑦
Solution to Example 9.
First determine whether the equation is homogeneous, set
𝑓(𝑥, 𝑦) = 𝑦 − (𝑥 + √𝑦 2 − 𝑥 2 )

𝑓(𝜆𝑥, 𝜆𝑦) = (𝜆𝑦) − ((𝜆𝑥) + √(𝜆𝑦)2 − (𝜆𝑥)2 )

= 𝜆 (𝑦 − 𝑥 − √𝑦 2 − 𝑥 2 )
= 𝜆 𝑓(𝑥, 𝑦)
Thus, the equation is homogeneous of degree one.
Since the coefficient of 𝑑𝑥 is less than 𝑑𝑦, then set 𝑥 = 𝑣𝑦 and 𝑑𝑥 = 𝑣𝑑𝑦 + 𝑦𝑑𝑣 and
substitute these in the given equation.
𝑦(𝑣𝑑𝑦 + 𝑦𝑑𝑣) = ((𝑣𝑦) + √𝑦 2 − (𝑣𝑦)2 ) 𝑑𝑦

𝑣𝑦𝑑𝑦 + 𝑦 2 𝑑𝑣 = 𝑣𝑦𝑑𝑦 + 𝑦√1 − 𝑣 2 𝑑𝑦


𝑦 2 𝑑𝑣 = 𝑦√1 − 𝑣 2 𝑑𝑦

This module is a property of Technological University of the Philippines Visayas and intended
for EDUCATIONAL PURPOSES ONLY and is NOT FOR SALE NOR FOR REPRODUCTION.
27

By separation of variables, we have


𝑑𝑣 𝑑𝑦
=
√1 − 𝑣 2 𝑦
Integrating, we find
𝑑𝑣 𝑑𝑦
∫ = ∫
√1 − 𝑣 2 𝑦
arcsin 𝑣 = ln 𝑦 + 𝐶
Substituting 𝑥/𝑦 for 𝑣 will lead to the result
𝒙
𝐚𝐫𝐜𝐬𝐢𝐧 = 𝐥𝐧 𝒚 + 𝑪
𝒚

Example 10. Solve the equation


𝑦 𝑦
(𝑥 − 𝑦 arctan ( )) 𝑑𝑥 + 𝑥 arctan ( ) 𝑑𝑦 = 0
𝑥 𝑥

Solution to Example 10.


The test of homogeneity will give the equation a degree one.
Let 𝑦 = 𝑣𝑥 and 𝑑𝑦 = 𝑣𝑑𝑥 + 𝑥𝑑𝑣

𝑣𝑥 𝑣𝑥
(𝑥 − (𝑣𝑥) arctan ( )) 𝑑𝑥 + 𝑥 arctan ( ) (𝑣𝑑𝑥 + 𝑥𝑑𝑣) = 0
𝑥 𝑥
𝑥𝑑𝑥 − 𝑣𝑥 arctan 𝑣 𝑑𝑥 + 𝑥𝑣 arctan 𝑣 𝑑𝑥 + 𝑥 2 arctan 𝑣 𝑑𝑣 = 0
𝑥𝑑𝑥 + 𝑥 2 arctan 𝑣 𝑑𝑣 = 0
𝑑𝑥
∫ + ∫ arctan 𝑣 𝑑𝑣 = 0
𝑥

To integrate arctan 𝑣 𝑑𝑣, use integration by parts:


𝑢 = arctan 𝑣 ∫ 𝑑𝑣 = ∫ 𝑑𝑣
𝑑𝑣
𝑑𝑢 = 𝑣=𝑣
1 + 𝑣2

𝑣𝑑𝑣
∫ arctan 𝑣 𝑑𝑣 = 𝑣 arctan 𝑣 − ∫
1 + 𝑣2
1
= 𝑣 arctan 𝑣 − ln(1 + 𝑣 2 )
2
Thus, the solution of the differential equation is
1
ln 𝑥 + 𝑣 arctan 𝑣 − ln(1 + 𝑣 2 ) = ln 𝐶
2
Substituting 𝑦/𝑥 for 𝑣, we find:
𝑦 𝑦 1 𝑦 2
ln 𝑥 + ( ) arctan ( ) − ln (1 + ( ) ) = ln 𝐶
𝑥 𝑥 2 𝑥
𝑦 𝑦 1 𝑥2 + 𝑦2
arctan = ln − ln 𝑥 2 + ln 𝐶
𝑥 𝑥 2 𝑥2
This module is a property of Technological University of the Philippines Visayas and intended
for EDUCATIONAL PURPOSES ONLY and is NOT FOR SALE NOR FOR REPRODUCTION.
28

2𝑦 𝑦 (𝑥 2 + 𝑦 2 ) 𝐶
arctan = ln
𝑥 𝑥 𝑥4
𝒚 (𝒙 + 𝒚𝟐 )𝑪
𝟐
𝟐𝒚 𝐚𝐫𝐜𝐭𝐚𝐧 = 𝒙 𝐥𝐧
𝒙 𝒙𝟒

Example 11. Solve the equation


(𝑥 − 𝑦 ln 𝑦 + 𝑦 ln 𝑥)𝑑𝑥 + 𝑥(ln 𝑦 − ln 𝑥)𝑑𝑦 = 0

Solution to Example 11.


The equation can also be written as
𝑦 𝑦
(𝑥 − 𝑦 ln ) 𝑑𝑥 + 𝑥 ln 𝑑𝑦 = 0
𝑥 𝑥

The equation is homogeneous of degree 1.


Let 𝑦 = 𝑣𝑥 and 𝑑𝑦 = 𝑣𝑑𝑥 + 𝑥𝑑𝑣, and substitute to the given equation:
(𝑣𝑥) (𝑣𝑥)
(𝑥 − (𝑣𝑥) ln ) 𝑑𝑥 + 𝑥 ln (𝑣𝑑𝑥 + 𝑥𝑑𝑣) = 0
𝑥 𝑥
𝑥𝑑𝑥 − 𝑣𝑥 ln 𝑣 𝑑𝑥 + 𝑣𝑥 ln 𝑑𝑥 + 𝑥 2 ln 𝑣 𝑑𝑣 = 0
𝑥𝑑𝑥 + 𝑥 2 ln 𝑣 𝑑𝑣 = 0
𝑑𝑥
∫ + ∫ ln 𝑣 𝑑𝑣 = 0
𝑥
To integrate ln 𝑣 𝑑𝑣, use integration by parts:
𝑢 = ln 𝑣 ∫ 𝑑𝑣 = ∫ 𝑑𝑣
𝑑𝑣
𝑑𝑢 = 𝑣=𝑣
𝑣

∫ ln 𝑣 𝑑𝑣 = 𝑣 ln 𝑣 − ∫ 𝑑𝑣
= 𝑣 ln 𝑣 − 𝑣
Thus, the solution is
ln 𝑥 + 𝑣 ln 𝑣 − 𝑣 = 𝐶
Substituting 𝑦/𝑥 for 𝑣, we find:
𝑦 𝑦 𝑦
ln 𝑥 + ln − = 𝐶
𝑥 𝑥 𝑥
𝑦
𝑥 ln 𝑥 + 𝑦 ln − 𝑦 = 𝐶𝑥
𝑥
𝑥 ln 𝑥 + 𝑦 ln 𝑦 − 𝑦 ln 𝑥 = 𝑦 + 𝐶𝑥
(𝒙 − 𝒚) 𝐥𝐧 𝒙 + 𝒚 𝐥𝐧 𝒚 = 𝒚 + 𝑪𝒙

This module is a property of Technological University of the Philippines Visayas and intended
for EDUCATIONAL PURPOSES ONLY and is NOT FOR SALE NOR FOR REPRODUCTION.
29

Linear Coefficients in Two Variables

Consider the two linear equations


𝑎1 𝑥 + 𝑏1 𝑦 + 𝑐1 = 0 (1)
𝑎2 𝑥 + 𝑏2 𝑦 + 𝑐2 = 0 (2)

Substituting the left-hand number of (1) in 𝑀 and that of (2) in 𝑁 of the differential
equation𝑀𝑑𝑥 + 𝑁𝑑𝑦 = 0 gives
(𝑎1 𝑥 + 𝑏1 𝑦 + 𝑐1 ) 𝑑𝑥 + (𝑎2 𝑥 + 𝑏2 𝑦 + 𝑐2 ) 𝑑𝑦 = 0

This is called the differential equation with linear coefficients in two variables.

If the two linear equations intersect, then let (ℎ, 𝑘) be the point of intersection.
Thus, the equations are satisfied by 𝑢 and 𝑣,
𝑎1 𝑢 + 𝑏1 𝑣 + 𝑐1 = 0
𝑎2 𝑢 + 𝑏2 𝑣 + 𝑐2 = 0

Define a transformation of the differential equation for 𝑥 and 𝑦 as follows:


𝑥 =𝑢+ℎ , 𝑑𝑥 = 𝑑𝑢
𝑦 =𝑣+𝑘 , 𝑑𝑦 = 𝑑𝑣

Substituting these equations in the differential equations gives the following:


[𝑎1 (𝑢 + ℎ) + 𝑏1 (𝑣 + 𝑘) + 𝑐1 ] 𝑑𝑥 + [𝑎2 (𝑢 + ℎ) + 𝑏2 (𝑣 + 𝑘) + 𝑐2 ] 𝑑𝑦 = 0
[(𝑎1 𝑢 + 𝑏1 𝑣) + (𝑎1 ℎ + 𝑏1 𝑘 + 𝑐1 )]𝑑𝑢 + [(𝑎2 𝑢 + 𝑏2 𝑣) + (𝑎2 ℎ + 𝑏2 𝑘 + 𝑐2 )]𝑑𝑣 = 0
(𝑎1 𝑢 + 𝑏1 𝑣) 𝑑𝑢 + (𝑎2 𝑢 + 𝑏2 𝑣) 𝑑𝑣 = 0

The transformed preceding equation is now homogeneous which we know how to solve.

Example 12. Solve the equation


(𝑥 − 2𝑦 + 3)𝑑𝑥 + (4𝑥 + 𝑦 + 3)𝑑𝑦 = 0

Solution to Example 12.


First, solve the point of intersection of the two linear equations

𝑥 − 2𝑦 + 3 = 0 𝑥 − 2𝑦 + 3 = 0
4𝑥 + 𝑦 + 3 = 0 8𝑥 + 2𝑦 + 6 = 0

Upon solving, we get 𝑥 = −1 and 𝑦 = 1. Set the transformation for 𝑥 and 𝑦 as


follows:
𝑥 =𝑢−1 𝑑𝑥 = 𝑑𝑢
𝑦 =𝑣+1 𝑑𝑦 = 𝑑𝑣

This module is a property of Technological University of the Philippines Visayas and intended
for EDUCATIONAL PURPOSES ONLY and is NOT FOR SALE NOR FOR REPRODUCTION.
30

Substituting the above equations in the given differential equation gives


(𝑥 − 2𝑦 + 3)𝑑𝑥 + (4𝑥 + 𝑦 + 3)𝑑𝑦 = 0
((𝑢 − 1) − 2(𝑣 + 1) + 3)𝑑𝑢 + (4(𝑢 − 1) + (𝑣 + 1) + 3)𝑑𝑣 = 0
(𝑢 − 2𝑣)𝑑𝑢 + (4𝑢 + 𝑣)𝑑𝑣 = 0

This leads to a homogeneous equations of degree one.


Let 𝑢 = 𝑚𝑣, and 𝑑𝑢 = 𝑚𝑑𝑣 + 𝑣𝑑𝑚 and substitute to the preceding D.E. as follows:

(𝑚𝑣 − 2𝑣)(𝑚𝑑𝑣 + 𝑣𝑑𝑚) + (4(𝑚𝑣) + 𝑣)𝑑𝑣 = 0


𝑚 𝑣𝑑𝑣 − 2𝑚𝑣𝑑𝑣 + 𝑚𝑣 2 𝑑𝑚 − 2𝑣 2 𝑑𝑚 + 4𝑚𝑣𝑑𝑣 + 𝑣𝑑𝑣 = 0
2

𝑣(𝑚2 + 2𝑚 + 1)𝑑𝑣 + 𝑣 2 (𝑚 − 2)𝑑𝑚 = 0

𝑑𝑣 𝑚−2
∫ +∫ 2 𝑑𝑚 = 0
𝑣 𝑚 + 2𝑚 + 1

By partial fractions, we can write the second term as


𝑚−2 𝐴 𝐵
= +
(𝑚 + 1)2 𝑚 + 1 (𝑚 + 1)2

𝑚 − 2 = 𝐴(𝑚 + 1) + 𝐵

Let 𝑚 = −1, this gives 𝐵 = −3. Substituting 𝑚 = 0 and 𝐵 = −3, in the preceding equation,
we get 𝐴 = 1. Thus, we have
𝑚−2 1 3
= −
(𝑚 + 1)2 𝑚 + 1 (𝑚 + 1)2

𝑑𝑣 𝑑𝑚 3 𝑑𝑚
∫ +∫ −∫ =0
𝑣 𝑚+1 (𝑚 + 1)2
Integrating, we obtain
3
ln 𝑣 + ln(𝑚 + 1) + = ln 𝐶
𝑚+1
Substituting 𝑚 = 𝑢/𝑣, we have
𝑢 3
ln 𝑣 + ln ( + 1) + 𝑢 = ln 𝐶
𝑣 +1 𝑣
𝑢+𝑣 3𝑣
ln 𝑣 + ln + = ln 𝐶
𝑣 𝑢+𝑣
𝑢+𝑣 3𝑣
ln 𝑣 ( ) + ln 𝐶 = −
𝑣 𝑢+𝑣
3𝑣
ln(𝑢 + 𝑣)𝐶 = −
𝑢+𝑣

This module is a property of Technological University of the Philippines Visayas and intended
for EDUCATIONAL PURPOSES ONLY and is NOT FOR SALE NOR FOR REPRODUCTION.
31

Substituting 𝑢 = 𝑥 + 1 and 𝑣 = 𝑦 − 1, we obtain


3(𝑦 − 1)
ln((𝑥 + 1) + (𝑦 − 1))𝐶 = −
(𝑥 + 1) + (𝑦 − 1)

(𝐱 + 𝐲)𝐥𝐧(𝒙 + 𝒚)𝑪 = −𝟑(𝒚 − 𝟏)

Example 13. Solve the equation (𝑥 + 𝑦 − 4)𝑑𝑥 − (𝑥 − 𝑦 + 2)𝑑𝑦 = 0

𝑥+𝑦−4=0
𝑥−𝑦+2=0
Upon solving for the point of intersection, we obtain 𝑥 = 1 and 𝑦 = 3. Then set the
transformations as follows:
𝑥 =𝑢+1 𝑑𝑥 = 𝑑𝑢
𝑦 =𝑣+3 𝑑𝑦 = 𝑑𝑣

Substituting the above equation in the given differential equation we have:


(𝑢 + 𝑣)𝑑𝑢 − (𝑢 − 𝑣)𝑑𝑣 = 0

The preceding equation if homogeneous. Let


𝑢 = 𝑣𝑚 𝑑𝑢 = 𝑣𝑑𝑚 + 𝑚𝑑𝑣

Then by substitution of 𝑢 and 𝑑𝑢, we obtain the following computations


(𝑢 + 𝑣)𝑑𝑢 − (𝑢 − 𝑣)𝑑𝑣 = 0
(𝑣𝑚 + 𝑣)(𝑣𝑑𝑚 + 𝑚𝑑𝑣) − (𝑣𝑚 − 𝑣)𝑑𝑣 = 0
𝑣 2 𝑚 𝑑𝑚 + 𝑣 2 𝑑𝑚 + 𝑣𝑚2 𝑑𝑣 + 𝑣𝑚𝑑𝑣 − 𝑣𝑚𝑑𝑣 + 𝑣𝑑𝑣 = 0
𝑣 2 (𝑚 + 1)𝑑𝑚 + 𝑣(𝑚2 + 1)𝑑𝑣 = 0
(𝑚 + 1)𝑑𝑚 𝑑𝑣
+ =0
(𝑚2 + 1) 𝑣
𝑚 𝑑𝑚 𝑑𝑚 𝑑𝑣
∫ 2 + ∫ 2 + ∫ =0
𝑚 +1 𝑚 +1 𝑣
1
ln(𝑚2 + 1) + arctan 𝑚 + ln 𝑣 = ln 𝐶
2
ln(𝑚2 + 1) + 2 arctan 𝑚 + 2 ln 𝑣 = ln 𝐶
ln(𝑚2 + 1)𝑣 2 𝐶 = −2 arctan 𝑚
where 𝑚 = 𝑢/𝑣;
𝑢 2 𝑢
ln (( ) + 1) 𝑣 2 𝐶 = −2 arctan
𝑣 𝑣
𝑢
ln(𝑢2 + 𝑣 2 )𝐶 = −2 arctan
𝑣
since 𝑢 = 𝑥 − 1 and 𝑣 = 𝑦 − 3, therefore;
𝒙−𝟏
𝐥𝐧((𝒙 − 𝟏)𝟐 + (𝒚 − 𝟑)𝟐 )𝑪 = −𝟐 𝐚𝐫𝐜𝐭𝐚𝐧
𝒚−𝟑

This module is a property of Technological University of the Philippines Visayas and intended
for EDUCATIONAL PURPOSES ONLY and is NOT FOR SALE NOR FOR REPRODUCTION.
32

PROGRESS CHECK
Activity Sheets
EXERCISE 2
Review on Methods for Solving Differential Equations of Order One

Name: _____________________________________________Year & Section: __________


Subject: ____________________________________________Date Submitted: __________
Professor: __________________________________________Score: __________________

For numbers 1 to 7, solve the following separable differential equations.

1. 𝑦 ′ = 𝑦 2 𝑥 3

𝑥+1
2. 𝑦 ′ =
𝑦 4 +1

3. (𝑥 − 1)𝑑𝑥 + 𝑥𝑦 2 𝑑𝑦 = 0

4. tan2 𝑦 𝑑𝑥 = sec 2 𝑥 𝑑𝑦

5. 𝑦 ′ = cos2 𝑥 cos 𝑦

6. 𝑑𝑟 = 𝑏(cos 𝜃 𝑑𝑟 + 𝑟 sin 𝜃 𝑑𝜃)

7. 𝑥𝑑𝑦 − 2𝑦𝑑𝑥 = 𝑦 2 𝑑𝑥

For numbers 8 to 10, solve the following separable differential equations and determine
the constant of integration when initial conditions are indicated.

8. 𝑥𝑦𝑦 ′ + (1 − 𝑦 2 ) = 0, when 𝑥 = 1, 𝑦=4

9. 𝑥 ln 𝑦 ln 𝑥 𝑑𝑦 + 𝑑𝑥 = 0 , when 𝑥 = 𝑒, 𝑦 = 1

10. (𝑦 2 − 𝑦 + 1)𝑦 ′ − 𝑦 = 𝑦 3 , when 𝑥 = 𝜋/4, 𝑦=1

This module is a property of Technological University of the Philippines Visayas and intended
for EDUCATIONAL PURPOSES ONLY and is NOT FOR SALE NOR FOR REPRODUCTION.
33

For numbers 11 to 25, determine the homogeneity of the functions. If it is homogenous,


state the degree the function.

11. 5 x + 3xy + 18 y
3 2 3

12. x + y + x

1
13.
y+x

14. ( x )e x y

x y
15. tan − arccos
y x

16. 5 x + 3x + 6 y

17. tan y + sin x + cos 3 y

x5
18. 4
x + y4

y x2
19. x ln + y ln 2
x y

20. (x 3 + y 3 )2
1

21. x 4 + xy 3 + x 2 y 2 − y 3

22. ln x (e x )

x− y
23.
x+ y

24. ln x + ln y

25. arctan x

This module is a property of Technological University of the Philippines Visayas and intended
for EDUCATIONAL PURPOSES ONLY and is NOT FOR SALE NOR FOR REPRODUCTION.
34

For numbers 26 to 31, Solve the following homogeneous equations. If initial conditions
are indicated, determine the value of the constant of integration and identify the
particular solution.
2𝑦+𝑥
26. 𝑦 ′ =
𝑥

27. 𝑥𝑦𝑑𝑦 = (𝑥 2 + 𝑦 2 )𝑑𝑥

28. (𝑥 2 + 2𝑥𝑦 − 4𝑦 2 )𝑑𝑥 − (𝑥 2 − 8𝑥𝑦 − 4𝑦 2 )𝑑𝑦 = 0

29. (𝑥 3 + 𝑦 3 ) 𝑑𝑥 + 3𝑥𝑦 2 𝑑𝑦 = 0, when 𝑥 = 1, 𝑦 = 1

30. (𝑥 2 + 𝑦 2 )𝑑𝑥 + 2𝑥𝑦𝑑𝑦 = 0, when 𝑥 = 2, 𝑦=1

31. (6𝑥 2 − 7𝑦 2 )𝑑𝑥 − 14𝑥𝑦𝑑𝑦 = 0, when 𝑥 = −1, 𝑦 = 3

For numbers 32 to 40, Linear Coefficients in Two Variables. Solve the following
equations.

32. (𝑥 − 𝑦)𝑑𝑥 − (𝑥 − 𝑦 − 1)𝑑𝑦 = 0

33. (𝑥 − 2𝑦 + 4)𝑑𝑥 + (2𝑥 − 𝑦 + 2)𝑑𝑦 = 0

34. (2𝑥 + 3𝑦)𝑑𝑥 + (𝑦 + 2)𝑑𝑦 = 0

35. (𝑥 − 4𝑦 − 9)𝑑𝑥 + (4𝑥 + 𝑦 − 2)𝑑𝑦 = 0

36. (2𝑥 + 𝑦)𝑑𝑥 − (3𝑥 + 2𝑦 − 2)𝑑𝑦 = 0

37. (2𝑥 + 3𝑦 − 1)𝑑𝑥 − 4(𝑥 + 1)𝑑𝑦 = 0

38. (3𝑥 − 𝑦)𝑑𝑥 − (𝑥 + 𝑦)𝑑𝑦 = 0

39. (4𝑥 + 3𝑦 − 7)𝑑𝑥 + (3𝑥 − 7𝑦 + 4)𝑑𝑦 = 0

40. (2𝑥 − 2𝑦)𝑑𝑥 + (𝑦 − 1)𝑑𝑦 = 0

This module is a property of Technological University of the Philippines Visayas and intended
for EDUCATIONAL PURPOSES ONLY and is NOT FOR SALE NOR FOR REPRODUCTION.
35

REFERENCES
Sergio E. Ymas Jr. Differential Equations (Simplified Approach). Sampaloc, Manila. Pablo.
L. Bustamante III Press, 1997.

Louis Leithold. The Calculus with Analytic Geometry. 6th ed. Harper & Row Publishers Inc.,
1990.

Clyde E. Love, Earl D. Rainville. Differential and Integral Calculus. 6th ed. The Macmillan
Co., 1961.

Ricardo C. Asin. Elementary Differential Equations Reviewer. Manila, Philippines, Merriam


& Webster Bookstore, Inc., 1991.

This module is a property of Technological University of the Philippines Visayas and intended
for EDUCATIONAL PURPOSES ONLY and is NOT FOR SALE NOR FOR REPRODUCTION.
36

LEARNING GUIDE

Week No.: __3&4__

TOPIC/S

I. Integrating Factor
a. Exact Equations
b. Integrating Factor by Inspection
c. Integrating Factor by Formula
d. Linear Equations
e. Bernoulli Equations
f. Substitution Method

EXPECTED COMPETENCIES
At the end of the week after studying the module, the student should be able to:
1. test the exactness of the differential equation;
2. determine if the given differential equation takes the linear form or Bernoulli form;
3. identify the integrating factor of the following linear equations; and
4. solve the following differential equations using integrating factor methods.

CONTENT/TECHNICAL INFORMATION

Introduction
This lesson shall continue the discussion on the other methods of solving differential
equations by using integrating factor. In learning guide week no. 2, we studied the three
methods of solving differential equations. This chapter will introduce six more methods as
follows:
• Exact Equations
• Integrating Factor by Inspection
• Integrating Factor by Formula
• Linear Equations
• Bernoulli Equations
• Substitution Method

While exact equations is a preliminary topic to integrating factor, the rest of the five
methods shall deal concretely with the concept of integrating factor.

This module is a property of Technological University of the Philippines Visayas and intended
for EDUCATIONAL PURPOSES ONLY and is NOT FOR SALE NOR FOR REPRODUCTION.
37

Exact Equations

There are differential equations whose methods in the previous chapters will not apply
such as the following examples:

(2𝑥𝑦 + 𝑥)𝑑𝑥 + (𝑥 2 + 𝑦)𝑑𝑦 = 0

3𝑥𝑦 2 𝑑𝑥 + (2𝑥 3 𝑦 + 4𝑦 3 )𝑑𝑦 = 0

(𝑥 + sin 𝑦)𝑑𝑥 + (𝑥 cos 𝑦 − 2𝑦)𝑑𝑦 = 0

However, for the next method of solving differential equations, which we call as
exact, the above equations are solvable.

Test for Exactness

If 𝑀(𝑥, 𝑦) and 𝑁(𝑥, 𝑦) are continuous functions and have continuous first partial
derivative, then a necessary and sufficient condition that the different equation

𝑀 𝑑𝑥 + 𝑁 𝑑𝑦 = 0
to be exact is that

𝜕𝑀 𝜕𝑁
=
𝜕𝑦 𝜕𝑥

For example, in the equation (2𝑥𝑦 + 𝑥)𝑑𝑥 + (𝑥 2 + 𝑦)𝑑𝑦 = 0, we have

𝑀 = 2𝑥𝑦 + 𝑥 𝑁 = 𝑥2 + 𝑦

𝜕𝑀 𝜕𝑁
= 2𝑥 = 2𝑥
𝜕𝑦 𝜕𝑥

Thus, this differential equation is said to be exact.

This module is a property of Technological University of the Philippines Visayas and intended
for EDUCATIONAL PURPOSES ONLY and is NOT FOR SALE NOR FOR REPRODUCTION.
38

Steps for Finding the solution of Exact Equations:

Let 𝐹(𝑥, 𝑦) + 𝑇(𝑦) = 𝐶 be the solution of an exact differential equation.

𝜕𝐹
1. Set 𝜕𝑥 = 𝑀, or ∫ 𝑑𝐹 = ∫ 𝑀 𝑑𝑥 + 𝑇(𝑦), 𝑇(𝑦) the constant of Integration

𝜕𝐹
2. Obtain 𝜕𝑦 = 𝑁, or ∫ 𝑑𝐹 = ∫ 𝑁 𝑑𝑦

𝑑𝑇
3. The preceding steps gives, 𝑇 ′ (𝑦) = , or ∫ 𝑑𝑇 = ∫ 𝑓(𝑦)𝑑𝑦
𝑑𝑦

4. Substitute the value of 𝑇 in 𝐹 + 𝑇 = 𝐶 to obtain the required solution.

Note: Another solution is possible by interchanging the roles of 𝑥 and 𝑦, i.e. start with

Let 𝐹(𝑥, 𝑦) + 𝑇(𝑥) = 𝐶 be the solution of an exact differential equation.

𝜕𝐹
1. Set 𝜕𝑦 = 𝑁, or ∫ 𝑑𝐹 = ∫ 𝑁 𝑑𝑦 + 𝑇(𝑥), 𝑇(𝑥) the constant of Integration

𝜕𝐹
2. Obtain 𝜕𝑥 = 𝑀, or ∫ 𝑑𝐹 = ∫ 𝑀 𝑑𝑥

𝑑𝑇
3. The preceding steps gives, 𝑇 ′ (𝑦) = , or ∫ 𝑑𝑇 = ∫ 𝑓(𝑥)𝑑𝑥
𝑑𝑥

4. Substitute the value of 𝑇 in 𝐹 + 𝑇 = 𝐶 to obtain the required solution.

This module is a property of Technological University of the Philippines Visayas and intended
for EDUCATIONAL PURPOSES ONLY and is NOT FOR SALE NOR FOR REPRODUCTION.
39

Example 1. Solve the following differential equations.


a. (2𝑥𝑦 + 𝑥)𝑑𝑥 + (𝑥 2 + 𝑦)𝑑𝑦 = 0
b. 3𝑥𝑦 2 𝑑𝑥 + (3𝑥 2 𝑦 + 4𝑦 3 )𝑑𝑦 = 0
c. (𝑥 + sin 𝑦)𝑑𝑥 + (𝑥 cos 𝑦 − 2𝑦)𝑑𝑦 = 0
d. (𝑥 3 + 𝑥𝑦 2 + 2𝑦)𝑑𝑥 + (𝑦 3 + 𝑥 2 𝑦 + 2𝑥)𝑑𝑦 = 0

**Technique for Solving solutions of DE: first check whether the variable is separable, if not, check the
homogeneity of the equation, if not, test the exactness.

Solutions to Example 1.
a. (2𝑥𝑦 + 𝑥) 𝑑𝑥 + (𝑥 2 + 𝑦)𝑑𝑦 = 0
Earlier, we tested the exactness of the equation and found out that the differential
equation is exact.

𝑀 = 2𝑥𝑦 + 𝑥 𝑁 = 𝑥2 + 𝑦

𝜕𝑀 𝜕𝑁
= 2𝑥 = 2𝑥
𝜕𝑦 𝜕𝑥

𝜕𝐹
Step 1. Set = 𝑀 = 2𝑥𝑦 + 𝑥
𝜕𝑥

∫ 𝜕𝐹 = ∫(2𝑥𝑦 + 𝑥)𝜕𝑥 + 𝑇(𝑦)


This gives This is supposedly the
2
𝑥 general solution, we just
𝐹 = 𝑥2𝑦 + + 𝑇(𝑦) need to identify the value
2 of 𝑇(𝑦) and add 𝐶.

𝜕𝐹 𝜕𝐹
Step 2. = 𝑥 2 + 𝑇 ′ (𝑦) ; =𝑁
𝜕𝑦 𝜕𝑦
𝑥 2 + 𝑇 ′ (𝑦) = 𝑥 2 + 𝑦
𝑇 ′ (𝑦) = 𝑦
𝑑𝑇
=𝑦
𝑑𝑦
Step 3. ∫ 𝑑𝑇 = ∫ 𝑦 𝑑𝑦
𝑦2
𝑇=
2
𝑦2
Step 4. Substitute in 𝑇(𝑦) to obtain the solution
2
𝑥2 𝑦2
𝐹 = 𝑥2𝑦 + + =𝐶
2 2
𝟐
𝒙𝟐 𝒚𝟐
𝒙 𝒚+ + =𝑪
𝟐 𝟐

This module is a property of Technological University of the Philippines Visayas and intended
for EDUCATIONAL PURPOSES ONLY and is NOT FOR SALE NOR FOR REPRODUCTION.
40

𝜕𝐹
Another Solution: Set 𝜕𝑦 = 𝑁

∫ 𝜕𝐹 = ∫(𝑥 2 + 𝑦) 𝜕𝑦
𝑦2
𝐹 = 𝑥2𝑦 + + 𝑇(𝑥)
2
𝜕𝐹
= 2𝑥𝑦 + 𝑇 ′ (𝑥) = 𝑀 = 2𝑥𝑦 + 𝑥
𝜕𝑥
∫ 𝑇 ′ (𝑥) = ∫ 𝑥
𝑥2
𝑇(𝑥) =
2
𝒚𝟐 𝒙𝟐
𝒙𝟐 𝒚 + + =𝑪
𝟐 𝟐

b. 3𝑥𝑦 2 𝑑𝑥 + (3𝑥 2 𝑦 + 4𝑦 3 )𝑑𝑦 = 0


Test the exactness of the equation:

𝑀 = 3𝑥𝑦 2 𝑁 = 3𝑥 2 𝑦 + 4𝑦 3
𝜕𝑀 𝜕𝑁
= 6𝑥𝑦 = 6𝑥𝑦
𝜕𝑦 𝜕𝑥
Thus, the equation is exact.

Using the Steps for Finding the Solution of Exact Equation:


𝜕𝐹
= 𝑀 = 3𝑥𝑦 2
𝜕𝑥
∫ 𝜕𝐹 = ∫(3𝑥𝑦 2 )𝜕𝑥 + 𝑇(𝑦)
3𝑥 2 𝑦 2
𝐹= + 𝑇(𝑦)
2
𝜕𝐹
= 3𝑥 2 𝑦 + 𝑇 ′ (𝑦) = 𝑁
𝜕𝑦
3𝑥 2 𝑦 + 𝑇 ′ (𝑦) = 3𝑥 2 𝑦 + 4𝑦 3
∫ 𝑇 ′ (𝑦) = ∫ 4𝑦 3
𝑇(𝑦) = 𝑦 4
𝟐 𝟐
𝟑𝒙 𝒚
+ 𝒚𝟒 = 𝑪
𝟐

c. (𝑥 + sin 𝑦)𝑑𝑥 + (𝑥 cos 𝑦 − 2𝑦)𝑑𝑦 = 0


Test the exactness of the equation:

𝑀 = 𝑥 + sin 𝑦 𝑁 = 𝑥 cos 𝑦 − 2𝑦
𝜕𝑀 𝜕𝑁
= cos 𝑦 = cos 𝑦
𝜕𝑦 𝜕𝑥
This module is a property of Technological University of the Philippines Visayas and intended
for EDUCATIONAL PURPOSES ONLY and is NOT FOR SALE NOR FOR REPRODUCTION.
41

Thus, the equation is exact.


𝜕𝐹
Say we set 𝜕𝑦 = 𝑁
𝜕𝐹
= 𝑁 = 𝑥 cos 𝑦 − 2𝑦
𝜕𝑦
∫ 𝜕𝐹 = ∫(𝑥 cos 𝑦 − 2𝑦)𝜕𝑦 + 𝑇(𝑥)
𝐹 = 𝑥 sin 𝑦 − 𝑦 2 + 𝑇(𝑥)
𝜕𝐹
= sin 𝑦 + 𝑇 ′ (𝑥) = 𝑀
𝜕𝑥
sin 𝑦 + 𝑇 ′ (𝑥) = 𝑥 + sin 𝑦
∫ 𝑇 ′ (𝑥) = ∫ 𝑥
𝑥2
𝑇(𝑦) =
2
𝒙𝟐 𝟐
𝒙 𝒔𝒊𝒏 𝒚 − 𝒚 + =𝑪
𝟐

d. (𝑥 3 + 𝑥𝑦 2 + 2𝑦)𝑑𝑥 + (𝑦 3 + 𝑥 2 𝑦 + 2𝑥)𝑑𝑦 = 0
Test the exactness of the equation:

𝑀 = 𝑥 3 + 𝑥𝑦 2 + 2𝑦 𝑁 = 𝑦 3 + 𝑥 2 𝑦 + 2𝑥
𝜕𝑀 𝜕𝑁
= 2𝑥𝑦 + 2 = 2𝑥𝑦 + 2
𝜕𝑦 𝜕𝑥
Thus, the equation is exact.

Using the Steps for Finding the Solution of Exact Equation:

𝜕𝐹
= 𝑀 = 𝑥 3 + 𝑥𝑦 2 + 2𝑦
𝜕𝑥
∫ 𝜕𝐹 = ∫(𝑥 3 + 𝑥𝑦 2 + 2𝑦)𝜕𝑥 + 𝑇(𝑦)

3
𝑥𝑦 3
𝐹 =𝑥 𝑦+ + 2𝑥𝑦 + 𝑇(𝑦)
3
𝜕𝐹
= 𝑥 3 + 𝑥𝑦 2 + 2𝑥 + 𝑇 ′ (𝑦) = 𝑁
𝜕𝑦
𝑥 3 + 𝑥𝑦 2 + 2𝑥 + 𝑇 ′ (𝑦) = 𝑦 3 + 𝑥 2𝑦 + 2𝑥
∫ 𝑇 ′ (𝑦) = ∫ 𝑦 3
𝑦4
𝑇(𝑦) =
4
𝒙𝒚𝟑 𝒚𝟒
𝒙𝟑 𝒚 + + 𝟐𝒙𝒚 + =𝑪
𝟑 𝟒

This module is a property of Technological University of the Philippines Visayas and intended
for EDUCATIONAL PURPOSES ONLY and is NOT FOR SALE NOR FOR REPRODUCTION.
42

Integrating Factor by Inspection

Recall the concept of exact differential equations which has bearings on the next
topics. The following are exact equations based on the test:

𝜕𝑀 𝜕𝑁
=
𝜕𝑦 𝜕𝑥

a. 𝑥 𝑑𝑦 + 𝑦 𝑑𝑥 = 0
1
b. 𝑥𝑦 𝑑𝑥 + 2 𝑥 2 𝑑𝑦 = 0
c. 𝑦 𝑒 𝑥𝑦 𝑑𝑥 + 𝑥 𝑒 𝑥𝑦 𝑑𝑦 = 0
1 𝑦
d. 𝑥 𝑑𝑦 − 𝑥 2 𝑑𝑥 = 0

The above are said to be exact equations, because we can associate each with an exact
differential, i.e.
a. 𝑥 𝑑𝑦 + 𝑦 𝑑𝑥 = 𝑑(𝑥𝑦)
1
b. 𝑥𝑦 𝑑𝑥 + 2 𝑥 2 𝑑𝑦 = 𝑑(𝑥 2 𝑦)
c. 𝑦 𝑒 𝑥𝑦 𝑑𝑥 + 𝑥 𝑒 𝑥𝑦 𝑑𝑦 = 𝑑(𝑒 𝑥𝑦 )
1 𝑦 𝑦
d. 𝑥 𝑑𝑦 − 𝑥 2 𝑑𝑥 = 𝑑 (𝑥 )
In general, the differential equation
𝑀 (𝑥, 𝑦) 𝑑𝑥 + 𝑁(𝑥, 𝑦) 𝑑𝑦 = 0

is not exact. Occasionally, however, it is possible to transform the non-exact differential


equation by a judicious multiplication which we call as integrating factor.

A function 𝐼 is an integrating factor on a differential equation with the equation

𝐼 [ 𝑀(𝑥, 𝑦) 𝑑𝑥 + 𝑁(𝑥, 𝑦) 𝑑𝑦 = 0 ]

is exact. As we have observed, an integrating factor is a solution to a certain partial


differential equation.

Finding an Integrating Factor

Integrating Factors are generally obtained by inspection. The success of the method
depends on the user’s ability to recognize in given what a particular group of term composes
an exact differential equation. The following table of integrating factors may prove helpful.

Group of Terms Integrating Factor 𝑰(𝒙, 𝒚) Exact Differential

1 𝑦𝑑𝑥 − 𝑥𝑑𝑦 𝑥
𝑦𝑑𝑥 − 𝑥𝑑𝑦 = 𝑑 ( )
𝑦2 𝑦2 𝑦

1 𝑥𝑑𝑦 − 𝑦𝑑𝑥 𝑥
𝑥𝑑𝑦 − 𝑦𝑑𝑥 = 𝑑 (− )
𝑦2 𝑦 2 𝑦
This module is a property of Technological University of the Philippines Visayas and intended
for EDUCATIONAL PURPOSES ONLY and is NOT FOR SALE NOR FOR REPRODUCTION.
43

1 𝑥𝑑𝑦 − 𝑦𝑑𝑥 𝑦
𝑥𝑑𝑦 − 𝑦𝑑𝑥 = 𝑑 ( )
𝑥2 𝑥2 𝑥
1 𝑥𝑑𝑦 − 𝑦𝑑𝑥 𝑦
𝑥𝑑𝑦 − 𝑦𝑑𝑥 = 𝑑 (arctan )
𝑥2 + 𝑦2 2
𝑥 +𝑦 2 𝑥

1 𝑥𝑑𝑦 + 𝑦𝑑𝑥
𝑥𝑑𝑦 + 𝑦𝑑𝑥 = 𝑑(ln 𝑥𝑦)
𝑥𝑦 𝑥𝑦

Example 2. Find the integrating factor of the following:


a. (𝑥 2 + 𝑦 2 )𝑑𝑥 + 𝑥𝑦 𝑑𝑦 = 0

b. 𝑥𝑦 𝑑𝑦 − (𝑦 2 + 3𝑥 2 )𝑑𝑥 = 0

c. 𝑦(𝑥 2 − 𝑦 2 + 1)𝑑𝑥 − 𝑥(𝑥 2 − 𝑦 2 − 1)𝑑𝑦 = 0

Solutions to Example 2.
a. (𝑥 2 + 𝑦 2 )𝑑𝑥 + 𝑥𝑦 𝑑𝑦 = 0
Proper grouping is important before determining an integrating factor. Notice, that
the first term must be treated independently because it is a pure function of 𝑥.
Here, the correct grouping must be done in the second and third terms, i.e.

𝑥 2 𝑑𝑥 + 𝑦 2 𝑑𝑥 + 𝑥𝑦𝑑𝑦 = 0
𝑥 2 𝑑𝑥 + 𝑦(𝑦𝑑𝑥 + 𝑥𝑑𝑦) = 0

Multiplying by 𝑥, we get

𝑥 3 𝑑𝑥 + 𝑥𝑦 (𝑦𝑑𝑥 + 𝑥𝑑𝑦) = 0

Notice that 𝑥 3 𝑑𝑥 is a pure function of 𝑥 alone, so it may stand without other


terms. So, by integration, we have

∫ 𝑥 3 𝑑𝑥 + ∫ 𝑥𝑦 (𝑦𝑑𝑥 + 𝑥𝑑𝑦) = 0
𝑥4
+ ∫ 𝑥𝑦 𝑑(𝑥𝑦) = 0
4
𝑥 4 (𝑥𝑦)2
+ =𝐶
4 2
𝑥 4 + 2(𝑥𝑦)2 = 𝐶
𝒙𝟒 + 𝟐𝒙𝟐 𝒚𝟐 = 𝑪 𝒐𝒓 𝒙𝟐 (𝒙𝟐 + 𝟐𝒚𝟐 ) = 𝑪

This module is a property of Technological University of the Philippines Visayas and intended
for EDUCATIONAL PURPOSES ONLY and is NOT FOR SALE NOR FOR REPRODUCTION.
44

b. 𝑥𝑦 𝑑𝑦 − (𝑦 2 + 3𝑥 2 ) 𝑑𝑥 = 0

𝑥𝑦 𝑑𝑦 − 𝑦 2 𝑑𝑥 − 3𝑥 2 𝑑𝑥 = 0
𝑦(𝑥𝑑𝑦 − 𝑦𝑑𝑥) − 3𝑥 2 𝑑𝑥 = 0
1
Multiplying by 𝑥 3 will make every term in appropriate grouping.
𝑦 𝑥𝑑𝑦 − 𝑦𝑑𝑥 3
( 2
) − 𝑑𝑥 = 0
𝑥 𝑥 𝑥

𝑦 𝑦 3
∫ 𝑑 ( ) − ∫ 𝑑𝑥 = 0
𝑥 𝑥 𝑥

1 𝑦 2
( ) − 3 ln 𝑥 = ln 𝐶
2 𝑥
𝑦2
= ln 𝑥 3 + ln 𝐶
2𝑥 2

𝒚𝟐 = 𝟐𝒙𝟐 𝐥𝐧 𝒙𝟑 𝑪

c. Again, sometimes an integrating factor becomes apparent if the terms of its


differential equations are strategically regrouped.

𝑦(𝑥 2 − 𝑦 2 + 1)𝑑𝑥 − 𝑥(𝑥 2 − 𝑦 2 − 1)𝑑𝑦 = 0

Observe the common factors of 𝑥 2 and 𝑦 2 for the grouping of terms.

𝑥 2 𝑦 𝑑𝑥 − 𝑦 3 𝑑𝑥 + 𝑦𝑑𝑥 − 𝑥 3 𝑑𝑦 + 𝑥𝑦 2 𝑑𝑦 + 𝑥𝑑𝑦 = 0
𝑥 2 (𝑦𝑑𝑥 − 𝑥𝑑𝑦) − 𝑦 2 (𝑦𝑑𝑥 − 𝑥𝑑𝑦) + (𝑥𝑑𝑦 + 𝑦𝑑𝑥) = 0

Dividing by 𝑥 2 𝑦 2 will give any group of terms an appropriate integrating factor,


i.e.
𝑦𝑑𝑥 − 𝑥𝑑𝑦 𝑦𝑑𝑥 − 𝑥𝑑𝑦
− + (𝑥𝑦)−2 (𝑥𝑑𝑦 + 𝑦𝑑𝑥) = 0
𝑦2 𝑥2

𝑥 𝑦
∫ 𝑑 ( ) − ∫ 𝑑 (− ) + ∫(𝑥𝑦)−2 𝑑(𝑥𝑦) = 0
𝑦 𝑥

𝑥 𝑦 (𝑥𝑦)−1
+ + =𝐶
𝑦 𝑥 −1

𝒙𝟐 + 𝒚𝟐 − 𝟏 = 𝑪𝒙𝒚, or
𝒙𝟐 + 𝑪𝒙𝒚 + 𝒚𝟐 = 𝟏

Take note that 𝐶 can assume any constants, so upon transposition of the term
𝐶𝑥𝑦, the sign will not change.

This module is a property of Technological University of the Philippines Visayas and intended
for EDUCATIONAL PURPOSES ONLY and is NOT FOR SALE NOR FOR REPRODUCTION.
45

Integrating Factor by Formula

In a way, integrating factor can easily be obtained by using formulas. These formulas
involve the test of exactness.
𝜕𝑀 𝜕𝑁
=
𝜕𝑦 𝜕𝑥
which should be divided either by 𝑀 or 𝑁 of the given differential equation depending on the
result which should lead to pure function.

Formulas for Determining the Integrating Factor

1. If
1 𝜕𝑀 𝜕𝑁
[ − ] = 𝑓(𝑥)
𝑁 𝜕𝑦 𝜕𝑥

a function of 𝑥 alone, then


𝐼(𝑥, 𝑦) = 𝑒 ∫ 𝑓(𝑥)𝑑𝑥

2. If
1 𝜕𝑁 𝜕𝑀
[ − ] = 𝑔(𝑦)
𝑀 𝜕𝑥 𝜕𝑦
a function of 𝑦 alone, then
𝐼(𝑥, 𝑦) = 𝑒 ∫ 𝑔(𝑦)𝑑𝑦

3. For homogeneous equations,


1
𝑀𝑥 + 𝑁𝑦
is the integrating factor.

Note: It should be noted at this point that if neither of the preceding criteria is satisfied, we
can say that the equation does not have an integrating factor that is a function of 𝑥 or
𝑦 alone.

Example 3. Solve the equation

(𝑥 2 + 𝑦 2 + 1)𝑑𝑥 + 𝑥(𝑥 − 2𝑦)𝑑𝑦 = 0

Solution to Example 3.

𝑀 = 𝑥2 + 𝑦2 + 1 𝑁 = 𝑥(𝑥 − 2𝑦)
𝜕𝑀 𝜕𝑁
= 2𝑦 = 2𝑥 − 2𝑦
𝜕𝑦 𝜕𝑥

Since the equation is not exact, we therefore use Integrating Factor by Formula

This module is a property of Technological University of the Philippines Visayas and intended
for EDUCATIONAL PURPOSES ONLY and is NOT FOR SALE NOR FOR REPRODUCTION.
46

Then by using the formula, we have

1 𝜕𝑀 𝜕𝑁 1
𝑓(𝑥) = [ − ]= [ 2𝑦 − (2𝑥 − 2𝑦) ]
𝑁 𝜕𝑦 𝜕𝑥 𝑥(𝑥 − 2𝑦)

4𝑦 − 2𝑥 2(2𝑦 − 𝑥)
𝑓(𝑥) = =
𝑥(𝑥 − 2𝑦) 𝑥(𝑥 − 2𝑦)

2
𝑓(𝑥) = −
𝑥

a function of 𝑥 alone. Thus, the integrating factor is given by


2
𝐼(𝑥, 𝑦) = 𝑒 ∫ 𝑓(𝑥)𝑑𝑥 = 𝑒 ∫ −𝑥𝑑𝑥 = 𝑒 −2 ln 𝑥

𝐼(𝑥, 𝑦) = 𝑥 −2

Multiplying the given equation by 𝑥 −2 and regrouping gives

(1 + 𝑥 −2 𝑦 2 + 𝑥 −2 )𝑑𝑥 + (1 − 2𝑥 −1 𝑦)𝑑𝑦 = 0
𝑑𝑥 + 𝑥 −2 𝑦 2 𝑑𝑥 + 𝑥 −2 𝑑𝑥 + 𝑑𝑦 − 2𝑥 −1 𝑦𝑑𝑦 = 0
𝑑𝑥 + 𝑑𝑦 + 𝑥 −2 𝑑𝑥 + (𝑥 −2 𝑦 2 𝑑𝑥 − 2𝑥 −1 𝑦 𝑑𝑦) = 0
∫ 𝑑𝑥 + ∫ 𝑑𝑦 + ∫ 𝑥 −2 𝑑𝑥 + ∫ 𝑑(−𝑥 −1 𝑦 2 ) = 0
1 𝑦2
𝑥+𝑦− − =𝐶
𝑥 𝑥
𝒙𝟐 + 𝒙𝒚 − 𝟏 − 𝒚𝟐 = 𝑪𝒙

Example 4. Solve the equation

2(2𝑦 2 + 5𝑥𝑦 − 2𝑦 + 4)𝑑𝑥 + 𝑥(2𝑥 + 2𝑦 − 1)𝑑𝑦 = 0

Solution to Example 4.

𝑀 = 2(2𝑦 2 + 5𝑥𝑦 − 2𝑦) 𝑁 = 𝑥(2𝑥 + 2𝑦 − 1)


𝜕𝑀 𝜕𝑁
= 8𝑦 + 10𝑥 − 4 = 4𝑥 + 2𝑦 − 1
𝜕𝑦 𝜕𝑥

No integrating factor is immediately apparent; however, with the use of the formula as
follows:

𝜕𝑀 𝜕𝑁
− = (8𝑦 + 10𝑥 − 4) − (4𝑥 + 2𝑦 − 1) = 6𝑥 + 6𝑦 − 3 = 3(2𝑥 + 2𝑦 − 1)
𝜕𝑦 𝜕𝑥

Upon dividing by 𝑁, we have

This module is a property of Technological University of the Philippines Visayas and intended
for EDUCATIONAL PURPOSES ONLY and is NOT FOR SALE NOR FOR REPRODUCTION.
47

1 𝜕𝑀 𝜕𝑁 3(2𝑥 + 2𝑦 − 1) 3
𝑓(𝑥) = [ − ]= =
𝑁 𝜕𝑦 𝜕𝑥 𝑥(2𝑥 + 2𝑦 − 1) 𝑥
Thus, the integrating factor is
𝑑𝑥
𝐼 (𝑥, 𝑦) = 𝑒 ∫ 𝑓(𝑥)𝑑𝑥 = 𝑒 3 ∫ 𝑥 = 𝑒 3 ln 𝑥 = 𝑥 3

Multiplying the given equation by 𝑥 3 and regrouping, we obtain the exact equation as
follows:
𝑥 3 [ 2(2𝑦 2 + 5𝑥𝑦 − 2𝑦 + 4)𝑑𝑥 + 𝑥(2𝑥 + 2𝑦 − 1)𝑑𝑦 = 0 ]
4𝑥 3 𝑦 2 𝑑𝑥 + 10𝑥 4 𝑦𝑑𝑥 − 4𝑥 3 𝑦𝑑𝑥 + 8𝑥 3 𝑑𝑥 + 2𝑥 5 𝑑𝑦 + 2𝑥 4 𝑦𝑑𝑦 − 𝑥 4 𝑑𝑦 = 0
(4𝑥 3 𝑦 2 𝑑𝑥 + 2𝑥 4 𝑦𝑑𝑦) + (10𝑥 4 𝑦𝑑𝑥 + 2𝑥 5 𝑑𝑦) − (4𝑥 3 𝑦𝑑𝑥 + 𝑥 4 𝑑𝑦) + 8𝑥 3 𝑑𝑥 = 0
∫ 𝑑(𝑥 4 𝑦 2 ) + ∫ 𝑑(2𝑥 5 𝑦) − ∫ 𝑑(𝑥 4 𝑦) + ∫ 8𝑥 3 𝑑𝑥 = 0
𝑥 4 𝑦 2 + 2𝑥 5 𝑦 − 𝑥 4 𝑦 + 2𝑥 4 = 𝐶
𝒙𝟒 (𝒚𝟐 + 𝟐𝒙𝒚 − 𝒚 + 𝟐) = 𝑪

Example 5. Solve the equation


𝑦(4𝑥 + 𝑦)𝑑𝑥 − 2(𝑥 2 − 𝑦)𝑑𝑦 = 0

Solution to Example 5.

𝑀 = 𝑦(4𝑥 + 𝑦) 𝑁 = −2(𝑥 2 − 𝑦)
𝜕𝑀 𝜕𝑁
= 4𝑥 + 2𝑦 = −4𝑥
𝜕𝑦 𝜕𝑥

1 𝜕𝑁 𝜕𝑀 1
𝑔(𝑦) = [ − ]= [−4𝑥 − (4𝑥 + 2𝑦)]
𝑀 𝜕𝑥 𝜕𝑦 𝑦(4𝑥 + 𝑦)

−8𝑥 − 2𝑦 −2 (4𝑥 + 𝑦) 2
𝑔(𝑦) = = = −
𝑦(4𝑥 + 𝑦) 𝑦(4𝑥 + 𝑦) 𝑦

Thus, the integrating factor is


𝑑𝑦
−2 ∫
𝐼(𝑥, 𝑦) = 𝑒 𝑦 = 𝑒 −2 ln 𝑦 = 𝑦 −2

Multiplying the given equation to 𝑦 −2 , we get

4𝑥𝑦 −1 𝑑𝑥 + 𝑑𝑥 − 2𝑥 2 𝑦 −2 𝑑𝑦 + 2𝑦 −1 𝑑𝑦 = 0
(4𝑥𝑦 −1 𝑑𝑥 − 2𝑥 2 𝑦 −2 𝑑𝑦) + 𝑑𝑥 + 2𝑦 −1 𝑑𝑦 = 0
𝑑𝑦
∫ 𝑑(2𝑥 2 𝑦 −1 ) + ∫ 𝑑𝑥 + 2 ∫ =0
𝑦
2𝑥 2 𝑦 −1 + 𝑥 + 2 ln 𝑦 = ln 𝐶
2
2𝑥
+ 𝑥 = − ln 𝑦 2 − ln 𝐶
𝑦
𝟐𝒙𝟐 + 𝒙𝒚 = −𝒚 𝐥𝐧 𝒚𝟐 𝑪
This module is a property of Technological University of the Philippines Visayas and intended
for EDUCATIONAL PURPOSES ONLY and is NOT FOR SALE NOR FOR REPRODUCTION.
48

PROGRESS CHECK
Activity Sheets
EXERCISE 3
Review on Integrating Factor (Exact Equations, Integrating Factor by Inspection,
Integrating Factor by Formula)

Name: _____________________________________________Year & Section: __________


Subject: ____________________________________________Date Submitted: __________
Professor: __________________________________________Score: __________________

For numbers 1 to 10, test the exactness of the following differential equations. If exact,
solve the equation using the Steps for Finding the solution of Exact Equations.

1. 2𝑥𝑦 𝑑𝑥 + (𝑦 2 + 𝑥 2 )𝑑𝑦 = 0

1
2. (𝑥𝑦 + 𝑥 2 )𝑑𝑥 + 2 𝑥 2 𝑑𝑦 = 0

3. (2𝑥𝑦 − 3𝑥 2 )𝑑𝑥 + (𝑥 2 + 2𝑦)𝑑𝑦 = 0

4. (2𝑥𝑦 − tan 𝑦)𝑑𝑥 + (𝑥 2 − 𝑥 sec 2 𝑦)𝑑𝑦 = 0

5. (cos 𝑥 cos 𝑦 − cot 𝑥)𝑑𝑥 − sin 𝑥 sin 𝑦 𝑑𝑦 = 0

6. (2𝑥𝑦 + 4𝑥 + 3)𝑑𝑥 + (𝑥 2 + 2𝑦 − 5)𝑑𝑦 = 0

7. (4𝑥 3 + 8𝑥 3 𝑦 3 )𝑑𝑥 + (10𝑦 4 + 6𝑥 4 𝑦 2 )𝑑𝑦 = 0

1 𝑦
8. 𝑑𝑦 − (𝑥 2 + 2 ln 𝑥 2 ) 𝑑𝑥 = 0
𝑥

9. (sin 𝑥 + 𝑦 )𝑑𝑥 + (𝑥 − 2 cos 𝑦)𝑑𝑦 = 0

1 𝑦 1 2𝑥
10. (𝑦 2 − 𝑥 2 ) 𝑑𝑥 + (𝑥 − 𝑦 3 − 2) 𝑑𝑦 = 0

This module is a property of Technological University of the Philippines Visayas and intended
for EDUCATIONAL PURPOSES ONLY and is NOT FOR SALE NOR FOR REPRODUCTION.
49

For numbers 11 to 20, solve the following differential equations using Integrating
Factor by Inspection.

11. (3𝑥 2 + 3𝑥𝑦 2 )𝑑𝑥 + (3𝑥 2 𝑦 − 3𝑦 2 + 2𝑦)𝑑𝑦 = 0

1 𝑥
12. 𝑦 𝑑𝑥 − 𝑦 2 𝑑𝑦 = 0

1
13. (𝑥 + 3)−1 cos 𝑦 𝑑𝑥 − [sin 𝑦 ln(5𝑥 + 15) − 𝑦] 𝑑𝑦 = 0

14. 𝛼 2 sec 2𝜃 tan 2𝜃 𝑑𝜃 + 𝛼(sec 2𝜃 + 2)𝑑𝛼 = 0

15. 𝑒 2𝑥 (𝑑𝑦 + 2𝑦 𝑑𝑥) = 𝑥 2 𝑑𝑥

16. 𝑥𝑑𝑦 − 𝑦 𝑑𝑥 = 𝑥 3 𝑦 2 𝑑𝑥

17. 𝑦 3 sin 2𝑥 𝑑𝑥 − 3𝑦 2 cos2 𝑥 𝑑𝑦 = 0

18. 𝑦 (𝑥 2 + 𝑦 2 − 1)𝑑𝑥 + 𝑥(𝑥 2 + 𝑦 2 + 1)𝑑𝑦 = 0

19. 𝑥𝑑𝑦 − 𝑦𝑑𝑥 = (𝑥𝑦) 𝑦 2 𝑑𝑦

20. 𝑥𝑑𝑦 − 𝑦𝑑𝑥 = (𝑥 2 + 𝑥𝑦 − 2𝑦 2 )𝑑𝑥

For numbers 21 to 29, solve the following differential equations using Integrating
Factor by Formula.

21. 𝑦 2 𝑑𝑥 + 𝑥𝑦 𝑑𝑦 = 0

22. 𝑥𝑦 2 𝑑𝑥 + 𝑥 2 𝑦𝑑𝑦 = 0

23. (𝑥 2 𝑦 2 − 𝑥)𝑑𝑥 + (𝑥 2 𝑦 4 − 𝑥)𝑑𝑦 = 0

24. 2𝑦(𝑥 2 − 𝑦 + 𝑥)𝑑𝑥 + (𝑥 2 − 2𝑦)𝑑𝑦 = 0

25. 𝑦(2𝑥 − 𝑦 + 1) 𝑑𝑥 + 𝑥(3𝑥 − 4𝑦 + 3)𝑑𝑦 = 0

26. (𝑥𝑦 + 1)𝑑𝑥 + 𝑥(𝑥 + 4𝑦 − 2)𝑑𝑦 = 0

27. (4𝑥𝑦 + 3𝑦 2 − 𝑥)𝑑𝑥 + 𝑥(𝑥 + 2𝑦)𝑑𝑦 = 0

28. (3𝑥𝑦 3 + 4𝑦)𝑑𝑥 + (3𝑥 2 𝑦 2 + 2𝑥)𝑑𝑦 = 0

29. (2𝑥𝑦 2 − 2𝑦)𝑑𝑥 + (3𝑥 2 𝑦 − 4𝑥)𝑑𝑦 = 0

This module is a property of Technological University of the Philippines Visayas and intended
for EDUCATIONAL PURPOSES ONLY and is NOT FOR SALE NOR FOR REPRODUCTION.
50

Linear Equations

The first order linear differential equation is defined as


𝑑𝑦 𝑑𝑥 Linear in 𝑥
+ 𝑃(𝑥) 𝑦 = 𝑄(𝑥) Linear in 𝑦 + 𝑃(𝑦) 𝑥 = 𝑄(𝑦)
𝑑𝑥 𝑑𝑦
𝑑𝑦
Notice that the left member is linear in both the dependent variable 𝑦 and its derivative 𝑑𝑥 .
The following are examples of linear equations:
𝑑𝑦
+ 3𝑥 2 𝑦 = 𝑥 2 sin 𝑥 𝑙𝑖𝑛𝑒𝑎𝑟 𝑖𝑛 𝑦
𝑑𝑥

𝑑𝑥
− 𝑥𝑦 2 = 𝑦 𝑙𝑖𝑛𝑒𝑎𝑟 𝑖𝑛 𝑥
𝑑𝑦

The following are examples of non-linear equations:


𝑑𝑦
+ 3𝑥𝑦 2 = 𝑥 2
𝑑𝑥

𝑑𝑥
+ 3𝑥 2 = 𝑦
𝑑𝑦

Theorem 3.1
The integrating factor of the linear equation is given by

𝐼(𝑥, 𝑦) = 𝑒 ∫ 𝑃(𝑥)𝑑𝑥
Proof: Supposed 𝐼(𝑥), a pure fraction of 𝑥 alone, be the integrating factor of the
linear equation (1). Then by multiplying (1) by 𝐼(𝑥) gives
𝑑𝑦
𝐼(𝑥) + 𝐼(𝑥)𝑃(𝑥)𝑦 = 𝐼(𝑥) 𝑄(𝑥)
𝑑𝑥
𝐼(𝑥)𝑑𝑦 + 𝐼(𝑥)𝑃(𝑥) 𝑦 𝑑𝑥 = 𝐼(𝑥)𝑄(𝑥)𝑑𝑥
[𝐼(𝑥)𝑃(𝑥)𝑦 − 𝐼(𝑥)𝑄(𝑥)] 𝑑𝑥 + 𝐼(𝑥)𝑑𝑦 = 0
where
𝑀𝑑𝑥 + 𝑁𝑑𝑦 = 0
𝑀 = 𝐼(𝑥)𝑃(𝑥)𝑦 − 𝐼(𝑥)𝑄(𝑥) , and 𝑁 = 𝐼(𝑥)
Taking the partials
𝜕𝑀 𝜕𝑁
=
𝜕𝑦 𝜕𝑥
𝑑𝐼
𝐼(𝑥)𝑃(𝑥) =
𝑑𝑥
𝑑𝐼
∫ 𝑃(𝑥)𝑑𝑥 = ∫
𝐼(𝑥)
ln 𝐼(𝑥) = ∫ 𝑃(𝑥)𝑑𝑥
𝑒 ln 𝐼(𝑥) = 𝑒 ∫ 𝑃(𝑥)𝑑𝑥
𝐼(𝑥) = 𝑒 ∫ 𝑃(𝑥)𝑑𝑥
This module is a property of Technological University of the Philippines Visayas and intended
for EDUCATIONAL PURPOSES ONLY and is NOT FOR SALE NOR FOR REPRODUCTION.
51

4
Example 6. Solve 𝑦 ′ + (𝑥) 𝑦 = 𝑥 4

Solution to Example 6.

Knowing that the equation is in linear form, then the integrating factor
𝑑𝑦
+ 𝑃(𝑥)𝑦 = 𝑄(𝑥)
𝑑𝑥
𝑑𝑦 4
+ ( ) 𝑦 = 𝑥4
𝑑𝑥 𝑥
4
𝑃(𝑥) =
𝑥
4
𝐼(𝑥) = 𝑒 ∫𝑥𝑑𝑥 = 𝑒 4 ln 𝑥 = 𝑥 4

Multiply the differential equation by 𝑥 4 , we find

𝑥 4 𝑑𝑦 + 4𝑥 3 𝑦𝑑𝑥 = 𝑥 8 𝑑𝑥
∫ 𝑑(𝑥 4 𝑦) = ∫ 𝑥 8 𝑑𝑥

𝟒
𝒙𝟗
𝒙 𝒚= +𝑪
𝟗

Example 7. Solve (𝑥 4 + 2𝑦)𝑑𝑥 − 𝑥𝑑𝑦 = 0

Solution to Example 7.
The given equation in linear form is:

𝑑𝑦
−𝑥 + 𝑥 4 + 2𝑦 = 0
𝑑𝑥
𝑑𝑦 2
− ( ) 𝑦 = 𝑥3
𝑑𝑥 𝑥

Thus, the integrating factor is


𝑑𝑥
𝐼(𝑥) = 𝑒 −2 ∫ 𝑥 = 𝑒 −2 ln 𝑥 = 𝑥 −2

Multiplying the equation in linear form by 𝑥 −2, we have

𝑑𝑦 2
𝑥 −2 [ − ( ) 𝑦 = 𝑥3 ]
𝑑𝑥 𝑥
𝑥 −2 𝑑𝑦 − 2𝑥 −3 𝑦 𝑑𝑥 = 𝑥𝑑𝑥
∫ 𝑑(𝑥 −2 𝑦) = ∫ 𝑥𝑑𝑥
𝑦 𝑥2
= +𝐶
𝑥2 2

𝟐𝒚 = 𝒙𝟐 (𝒙𝟐 + 𝑪)
This module is a property of Technological University of the Philippines Visayas and intended
for EDUCATIONAL PURPOSES ONLY and is NOT FOR SALE NOR FOR REPRODUCTION.
52

Example 8. Solve (𝑥𝑦 2 + 𝑥 2 )𝑑𝑦 + 𝑥𝑦 𝑑𝑥 = 0

Solution to Example 8.

𝑥𝑦 2 + 𝑥 2 𝑑𝑥
+ =0
𝑥𝑦 𝑑𝑦

The linear form of the equation is


𝑑𝑥 1
+ ( ) 𝑥 = −𝑦
𝑑𝑦 𝑦

The integrating factor is


𝑑𝑦
∫𝑦
𝐼 = 𝑒 ∫ 𝑃(𝑦)𝑑𝑦 = 𝑒 = 𝑒 ln 𝑦 = 𝑦

Multiplying the equation in linear form by 𝑦, we find

𝑦𝑑𝑥 + 𝑥𝑑𝑦 = −𝑦 2 𝑑𝑦
∫ 𝑑(𝑥𝑦) = − ∫ 𝑦 2 𝑑𝑦
𝒚𝟑
𝒙𝒚 = − +𝑪
𝟑

Example 9. Solve the equation 𝑦𝑑𝑥 − 𝑥𝑑𝑦 = 𝑦 2 𝑑𝑦

Solution to Example 9.

The linear form of the equation is


𝑑𝑥 𝑥
− =𝑦
𝑑𝑦 𝑦
The integrating factor is
𝑑𝑦
−∫
𝐼= 𝑒 ∫ 𝑃(𝑦)𝑑𝑦 =𝑒 𝑦 = 𝑒 − ln 𝑦 = 𝑦 −1

Multiplying the linear equation by 𝑦 −1 , we have

𝑦 −1 𝑑𝑥 − 𝑥𝑦 −2 𝑑𝑦 = 𝑑𝑦
∫ 𝑑(𝑥𝑦 −1 ) = ∫ 𝑑𝑦
𝑥
=𝑦+𝐶
𝑦
𝒙 − 𝒚𝟐 = 𝑪𝒚

This module is a property of Technological University of the Philippines Visayas and intended
for EDUCATIONAL PURPOSES ONLY and is NOT FOR SALE NOR FOR REPRODUCTION.
53

Example 10. Solve the equation 𝑦 ′ = 1 − 2𝑦 cot 2𝑥

Solution to Example 10.

The linear form of the equation is


𝑑𝑦
+ 2𝑦 cot 2𝑥 = 1
𝑑𝑥
The integrating factor is
𝐼 = 𝑒 ∫ 𝑃(𝑥)𝑑𝑥 = 𝑒 ∫ 2 cot 2𝑥 𝑑𝑥 = 𝑒 ln sin 2𝑥 = sin 2𝑥

Thus,
sin 2𝑥 𝑑𝑦 + 2𝑦 cos 2𝑥 𝑑𝑥 = sin 2𝑥 𝑑𝑥
∫ 𝑑(𝑦 sin 2𝑥 ) = ∫ sin 2𝑥 𝑑𝑥
1
𝑦 sin 2𝑥 = − cos 2𝑥 + 𝐶
2
𝟏
𝒚 = − 𝐜𝐨𝐭 𝟐𝒙 + 𝐂 𝐜𝐬𝐜 𝟐𝒙
𝟐

Bernoulli Equations

Bernoulli Equation is a generalized case of the linear form. Bernoulli contains a


certain expression in terms of the dependent variable which makes it distinct from the linear
form.

A Bernoulli Equation is of the form

𝑑𝑦
+ 𝑃(𝑥)𝑦 = 𝑦 𝑛 𝑄(𝑥)
𝑑𝑥

The expression 𝑦 𝑛 , in terms of the dependent variable, is what makes it distinct from
the linear form. So, multiplying both sides of the Bernoulli equation by 𝑦 −𝑛 gives

𝑑𝑦
𝑦 −𝑛 + 𝑃(𝑥) 𝑦 −𝑛+1 = 𝑄(𝑥) (1)
𝑑𝑥
Let
𝑦 −𝑛+1 = 𝑧
(−𝑛 + 1) 𝑦 −𝑛 𝑑𝑦 = 𝑑𝑧 𝑜𝑟
𝑑𝑧
𝑦 −𝑛 𝑑𝑦 =
1−𝑛

Substitute 𝑦 −𝑛 𝑑𝑦 in (1) gives

𝑑𝑧
+ 𝑃(𝑥) 𝑧 = 𝑄(𝑥) 𝑜𝑟
(1 − 𝑛)𝑑𝑥

This module is a property of Technological University of the Philippines Visayas and intended
for EDUCATIONAL PURPOSES ONLY and is NOT FOR SALE NOR FOR REPRODUCTION.
54

𝑑𝑧
+ (1 − 𝑛)𝑃(𝑥) 𝑧 = (1 − 𝑛) 𝑄(𝑥) Linear in 𝑧
𝑑𝑥

where, (1 − 𝑛) is constant

The preceding solution is now linear in 𝑧, therefore we can now use the process in
solving solutions using linear equation method.

Example 11. Solve the differential equation


𝑑𝑦
− 𝑦 = 𝑥𝑦 2
𝑑𝑥

Solution to Example 11.

It is important to determine the expression, in terms of the dependent variable that


makes it distinct from the linear form and that is 𝑦 2 . Multiplying the equation by 𝑦 −2 or
dividing the equation by 𝑦 2 gives

𝑑𝑦
𝑦 −2 − 𝑦 −1 = 𝑥
𝑑𝑥
Let
𝑦 −1 = 𝑧
−𝑦 −2 𝑑𝑦 = 𝑑𝑧 𝑜𝑟
𝑦 −2 𝑑𝑦 = −𝑑𝑧

By substitution, we get
𝑑𝑧 𝑑𝑧
− − 𝑧 = 𝑥 𝑜𝑟 + 𝑧 = −𝑥
𝑑𝑥 𝑑𝑥

which is now linear in z. The integrating factor is

𝐼 = 𝑒 ∫ 𝑃(𝑥)𝑑𝑥 = 𝑒 ∫ 𝑑𝑥 = 𝑒 𝑥

Multiplying the linear equation by 𝑒 𝑥

𝑒 𝑥 𝑑𝑧 + 𝑧 𝑒 𝑥 𝑑𝑥 = −𝑥𝑒 𝑥 𝑑𝑥
∫ 𝑑(𝑧𝑒 𝑥 ) = − ∫ 𝑥𝑒 𝑥 𝑑𝑥
To integrate ∫ 𝑥𝑒 𝑥 𝑑𝑥 use integration by parts

𝑢=𝑥 ∫ 𝑑𝑣 = ∫ 𝑒 𝑥 𝑑𝑥
𝑑𝑢 = 𝑑𝑥 𝑣 = 𝑒𝑥

∫ 𝑒 𝑥 𝑑𝑥 = 𝑥𝑒 𝑥 − ∫ 𝑒 𝑥 𝑑𝑥 = 𝑥𝑒 𝑥 − 𝑒 𝑥 + 𝐶

This module is a property of Technological University of the Philippines Visayas and intended
for EDUCATIONAL PURPOSES ONLY and is NOT FOR SALE NOR FOR REPRODUCTION.
55

Thus, we have
𝑧𝑒 𝑥 = −(𝑥𝑒 𝑥 − 𝑒 𝑥 ) + 𝐶
𝑧𝑒 𝑥 = −𝑥𝑒 𝑥 + 𝑒 𝑥 + 𝐶
Since 𝑧 = 𝑦 −1
𝑒𝑥
= −𝑥𝑒 𝑥 + 𝑒 𝑥 + 𝐶 𝑜𝑟
𝑦

𝒚−𝟏 = −𝒙 + 𝟏 + 𝑪𝒆−𝒙

Example 12. Solve (𝑥 2 + 𝑦 2 )𝑑𝑥 − 𝑥𝑦 𝑑𝑦 = 0

Solution to Example 12.

Rearranging the form of the given equation

𝑑𝑦 1 𝑥
− 𝑦=
𝑑𝑥 𝑥 𝑦

Notice that the equation in in Bernoulli form, therefore multiplying both sides by 𝑦
gives
𝑑𝑦 1 2
𝑦 − 𝑦 =𝑥
𝑑𝑥 𝑥
Let
𝑦2 = 𝑧
2𝑦𝑑𝑦 = 𝑑𝑧 𝑜𝑟
𝑑𝑧
𝑦𝑑𝑦 =
2
By substitution
𝑑𝑧 1 𝑑𝑧 2
− 𝑧=𝑥 𝑜𝑟 − 𝑧 = 2𝑥
2 𝑑𝑥 𝑥 𝑑𝑥 𝑥

The preceding equation is now linear in 𝑧. The integrating factor is


𝑑𝑥
𝐼 = 𝑒 −2 ∫ 𝑥 = 𝑒 −2 ln 𝑥 = 𝑥 −2

Multiplying the equation in linear form by 𝑥 −2 gives

𝑑𝑧
𝑥 −2 − 2𝑥 −3 𝑧 = 2𝑥 −1
𝑑𝑥
𝑥 −2 𝑑𝑧 − 2𝑥 −3 𝑧𝑑𝑥 = 2𝑥 −1 𝑑𝑥
𝑑𝑥
∫ 𝑑(𝑥 −2 𝑧) = 2 ∫
𝑥
𝑥 −2 𝑧 = 2ln 𝑥 + ln 𝐶
𝑥 −2 𝑧 = ln 𝑥 2 𝐶
Since 𝑧 = 𝑦 2 , therefore

This module is a property of Technological University of the Philippines Visayas and intended
for EDUCATIONAL PURPOSES ONLY and is NOT FOR SALE NOR FOR REPRODUCTION.
56

𝑦2
2
= ln 𝑥 2 𝐶 𝑜𝑟
𝑥
𝒚𝟐 = 𝒙𝟐 𝐥𝐧 𝒙𝟐 𝑪

It is interesting to observe that this was also given in homogeneous equation under
example 8.

Substitution Method

A differential equation 𝑀𝑑𝑥 + 𝑁𝑑𝑦 = 0 may not be possible to solve by any of the
previous methods, but solvable by substitution approach. This is done by change of variables
to transform the differential equation into a type which we know how to solve. The
substitution will be an appropriate method if we choose certain expression to represent as 𝑢
in the change of variables, then its differential 𝑑𝑢 must also be present in the given equation
and, thus, reducing the equation to a known method.

Illustrations:
a. (3 tan 𝑥 − 2 cos 𝑦) sec 2 𝑥 𝑑𝑥 + tan 𝑥 sin 𝑦 𝑑𝑦 = 0
𝑢 = tan 𝑥, 𝑑𝑢 = sec 2 𝑥 𝑑𝑥

b. tan 𝑥 sin 2𝑦 𝑑𝑦 + (cos2 𝑦 − cos2 𝑥) 𝑑𝑦 = 0


𝑢 = cos2 𝑦, 𝑑𝑢 = −2 sin 2𝑦 𝑑𝑦

c. (𝑘 𝑒 2𝑦 − 𝑥)𝑑𝑥 = 2𝑒 2𝑦 (𝑒 2𝑦 + 𝑘𝑥)𝑑𝑦
𝑢 = 𝑒 2𝑦 , 𝑑𝑢 = 2𝑒 2𝑦 𝑑𝑦

Example 13. Solve the equation


− sin 𝑦 (𝑥 + sin 𝑦)𝑑𝑥 + 2𝑥 2 cos 𝑦 𝑑𝑦 = 0

Solution to Example 13.

Let 𝑢 = sin 𝑦, since the differential 𝑑𝑢 = cos 𝑦 𝑑𝑦 is also present in the equation. By
substitution, we get

−𝑢 (𝑥 + 𝑢)𝑑𝑥 + 2𝑥 2 𝑑𝑢 = 0

This equation is solvable by homogeneous, by Bernoulli and by integrating factor.

Choosing the homogeneous method, we let

𝑢 = 𝑣𝑥 , 𝑑𝑢 = 𝑣𝑑𝑥 + 𝑥𝑑𝑣

By substitution,
−𝑣𝑥(𝑥 + 𝑣𝑥)𝑑𝑥 + 2𝑥 2 (𝑣𝑑𝑥 + 𝑥𝑑𝑣) = 0
This module is a property of Technological University of the Philippines Visayas and intended
for EDUCATIONAL PURPOSES ONLY and is NOT FOR SALE NOR FOR REPRODUCTION.
57

−𝑣𝑥 2 𝑑𝑥 − 𝑣 2 𝑥 2 𝑑𝑥 + 2𝑣𝑥 2 𝑑𝑥 + 2𝑥 3 𝑑𝑣 = 0
−𝑣 2 𝑥 2 𝑑𝑥 + 𝑣𝑥 2 𝑑𝑥 + 2𝑥 3 𝑑𝑣 = 0
𝑣𝑥 2 (1 − 𝑣)𝑑𝑥 + 2𝑥 3 𝑑𝑣 = 0
𝑑𝑥 2𝑑𝑣
∫ +∫ =0
𝑥 𝑣(1 − 𝑣)

we can solve the 2nd term by partial fraction:


2 𝐴 𝐵
= +
𝑣(1 − 𝑣) 𝑣 1 − 𝑣

2 = 𝐴(1 − 𝑣) + 𝐵𝑣
Let 𝑣 = 0, then 𝐴 = 2. If 𝑣 = 1, then 𝐵 = 2. Thus, we have

𝑑𝑥 2 2
∫ + ∫( + ) 𝑑𝑣 = 0
𝑥 𝑣 1−𝑣

𝑑𝑥 𝑑𝑣 𝑑𝑣
∫ + 2∫ + 2∫ =0
𝑥 𝑣 1−𝑣
ln 𝑥 + 2 ln 𝑣 − 2 ln(1 − 𝑣) = ln 𝐶
ln 𝑥 + ln 𝑣 2 − ln(1 − 𝑣)2 = ln 𝐶
𝑥𝑣 2
ln = ln 𝐶
(1 − 𝑣)2
𝑥𝑣 2 = (1 − 𝑣)2 𝐶
Since 𝑣 = 𝑢/𝑥 , then
𝑢 2 𝑢 2
𝑥 ( ) = (1 − ) 𝐶
𝑥 𝑥
𝑥𝑢2 (𝑥 − 𝑢)2
= 𝐶
𝑥2 𝑥2
2 2
𝑥𝑢 = (𝑥 − 𝑢) 𝐶
Since 𝑢 = sin 𝑦, then

𝒙 𝐬𝐢𝐧𝟐 𝒚 = (𝒙 − 𝐬𝐢𝐧 𝒚)𝟐 𝑪

Example 14. Solve 𝑦 (𝑥 tan 𝑥 + ln 𝑦) 𝑑𝑥 + tan 𝑥 𝑑𝑦 = 0

Solution to Example 14.

The given equation may be written as


𝑑𝑦
(𝑥 tan 𝑥 + ln 𝑦)𝑑𝑥 + tan 𝑥 =0
𝑦
𝑑𝑦
Let 𝑢 = ln 𝑦, since its differential 𝑑𝑢 = 𝑦 is also present in the equation. Thus, we
have
(𝑥 tan 𝑥 + 𝑢)𝑑𝑥 + tan 𝑥 𝑑𝑢 = 0

This module is a property of Technological University of the Philippines Visayas and intended
for EDUCATIONAL PURPOSES ONLY and is NOT FOR SALE NOR FOR REPRODUCTION.
58

This can be written in Bernoulli form

𝑑𝑢
+ 𝑢 cot 𝑥 = −𝑥
𝑑𝑥

The integrating factor is


𝐼 = 𝑒 ∫ cot 𝑥 𝑑𝑥 = 𝑒 ln sin 𝑥 = sin 𝑥

Multiplying the linear equation by sin 𝑥 gives

sin 𝑥 𝑑𝑢 + 𝑢 cos 𝑥 𝑑𝑥 = −𝑥 sin 𝑥 𝑑𝑥


∫ 𝑑(𝑢 sin 𝑥) = − ∫ 𝑥 sin 𝑥 𝑑𝑥
We can solve the right member of the equation by parts
Let
𝑢=𝑥 ∫ 𝑑𝑣 = ∫ sin 𝑥 𝑑𝑥
𝑑𝑢 = 𝑑𝑥 𝑣 = − cos 𝑥

∫ 𝑥 sin 𝑥 𝑑𝑥 = −𝑥 cos 𝑥 + ∫ cos 𝑥 𝑑𝑥


= −𝑥 cos 𝑥 + sin 𝑥 + 𝐶

Therefore,
𝑢 sin 𝑥 = −(−𝑥 cos 𝑥 + sin 𝑥) + 𝐶
𝑢 sin 𝑥 = 𝑥 cos 𝑥 − sin 𝑥 + 𝐶

Since 𝑢 = ln 𝑦, we have

𝐥𝐧 𝒚 𝐬𝐢𝐧 𝒙 = 𝒙 𝐜𝐨𝐬 𝒙 − 𝐬𝐢𝐧 𝒙 + 𝑪

This module is a property of Technological University of the Philippines Visayas and intended
for EDUCATIONAL PURPOSES ONLY and is NOT FOR SALE NOR FOR REPRODUCTION.
59

PROGRESS CHECK
Activity Sheets
EXERCISE 4
Review on Integrating Factor (Linear Equation, Bernoulli Equation, Substitution
Method)

Name: _____________________________________________Year & Section: __________


Subject: ____________________________________________Date Submitted: __________
Professor: __________________________________________Score: __________________

For numbers 1 to 10, determine if the equation takes the linear form, if so, find the
integrating factor and solve the solution of the given differential equation.

1. 𝑦 ′ + 𝑦 cot 𝑥 = 5𝑒 cos 𝑥

2. 𝑥𝑦 ′ = 𝑦 + 𝑥 3 + 3𝑥 2 − 2𝑥

3. 𝑦 ′ − 2𝑦 cot 2𝑥 = 1 − 2𝑥 cot 2𝑥 − 2 csc 2𝑥

4. 𝑑𝑦⁄𝑑𝑥 + 2𝑥𝑦 = 4𝑥

5. 𝑦 ′ + 𝑦 = sin 𝑥

6. 𝑦 ′ = 1 + 3𝑦 tan 𝑥

7. 2𝑥(𝑦 − 𝑥 2 )𝑑𝑥 + 𝑑𝑦 = 0

8. 𝑦 ′ = csc 𝑥 − 𝑦 cot 𝑥

9. 𝑥𝑑𝑦 − 2𝑦𝑑𝑥 = (𝑥 − 2)𝑒 𝑥 𝑑𝑥

10. 𝑦 ′ − 𝑥 + 2𝑦 cot 2𝑥 = 0

For numbers 11 to 20, solve the following Bernoulli Equations:

𝑑𝑦 𝑦
11. 𝑑𝑥 + 𝑥 = 𝑥𝑦 2

𝑑𝑦 𝑥
12. 𝑑𝑥 + 𝑦 = 𝑦

This module is a property of Technological University of the Philippines Visayas and intended
for EDUCATIONAL PURPOSES ONLY and is NOT FOR SALE NOR FOR REPRODUCTION.
60

𝑑𝑦 𝑦 𝑥
13. 𝑑𝑥 + 𝑥 = 𝑦

14. 𝑦 ′ + 𝑥𝑦 = 𝑥𝑦 2

1
3
15. 𝑦 ′ − 𝑥 𝑦 = 𝑥 4 𝑦 3

16. 𝑦 ′ + 𝑦 = 𝑦 2

𝑑𝑥
17. 𝑑𝑦 + 𝑦 2 𝑥 = 𝑥 2 𝑦 2

18. 6𝑦 2 𝑑𝑥 − 𝑥(2𝑥 3 + 𝑦)𝑑𝑦 = 0

19. 2𝑥 3 𝑦 ′ = 𝑦(𝑦 2 + 3𝑥 2 )

20. 𝑦(6𝑦 2 − 𝑥 −1 )𝑑𝑥 + 2𝑥𝑑𝑦 = 0

For numbers 21 to 25, solve the following differential equation by substitution.

21. cos 𝑦 (𝑥 + cos 𝑦)𝑑𝑥 + 2𝑥 2 sin 𝑦 𝑑𝑦 = 0

22. (3 tan 𝑥 − 2 cos 𝑦)𝑑𝑥 + cot 𝑥 𝑑𝑦 = 0

23. 𝑦(cot 𝑥 + ln 𝑦) 𝑑𝑥 + cot 𝑥 𝑑𝑦 = 0

24. (3 sin 𝑦 − 5𝑥)𝑑𝑥 + 2𝑥 2 cot 𝑦 𝑑𝑦 = 0

25. 2𝑦 𝑑𝑥 + 𝑥(𝑥 2 ln 𝑦 − 1)𝑑𝑦 = 0

This module is a property of Technological University of the Philippines Visayas and intended
for EDUCATIONAL PURPOSES ONLY and is NOT FOR SALE NOR FOR REPRODUCTION.
61

REFERENCES
Sergio E. Ymas Jr. Differential Equations (Simplified Approach). Sampaloc, Manila. Pablo.
L. Bustamante III Press, 1997.

Louis Leithold. The Calculus with Analytic Geometry. 6th ed. Harper & Row Publishers Inc.,
1990.

Clyde E. Love, Earl D. Rainville. Differential and Integral Calculus. 6th ed. The Macmillan
Co., 1961.

Ricardo C. Asin. Elementary Differential Equations Reviewer. Manila, Philippines, Merriam


& Webster Bookstore, Inc., 1991.

This module is a property of Technological University of the Philippines Visayas and intended
for EDUCATIONAL PURPOSES ONLY and is NOT FOR SALE NOR FOR REPRODUCTION.
62

LEARNING GUIDE

Week No.: __6__

TOPIC/S

I. Applications of Differential Equations of Order One


a. Newton’s Law of Cooling
b. Growth/Decay
c. others

EXPECTED COMPETENCIES
At the end of the week after studying the module, the student should be able to:
1. apply methods in solving solutions of first order differential equations in real life
problems.

CONTENT/TECHNICAL INFORMATION

Introduction

Many real-life problems involve differential equations of order one. Two physical
applications shall be considered in this topic – the Newton’s Law of Cooling and Decay,
Growth Problems.

Newton’s Law of Cooling

Newton’s Law of Cooling states that: The time rate of change of the temperature of a
body is proportional to the temperature difference between the body and its surrounding
medium.
𝑑𝑇
∝ (𝑇 − 𝑇𝑚 )
𝑑𝑡

Let 𝑻 denote the temperature of the body and let 𝑻𝒎 denote the temperature of the
surrounding medium. Then the time rate of change of the temperature of the body is 𝑑𝑇⁄𝑑𝑡,
and Newton’s Law of cooling can be formulated as 𝑑𝑇⁄𝑑𝑡 = −𝑘 (𝑇 − 𝑇𝑚 ), or as

𝑑𝑇
+ 𝑘𝑇 = 𝑘𝑇𝑚
𝑑𝑡
Where 𝑘 is a positive constant of proportionality. Once 𝑘 is chosen positive, the
minus sign is required in Newton’s Law to make 𝑑𝑇⁄𝑑𝑡 negative in a cooling process. Note
that in such a process, 𝑇 is greater than 𝑇𝑚 ; thus 𝑇 − 𝑇𝑚 is positive.

This module is a property of Technological University of the Philippines Visayas and intended
for EDUCATIONAL PURPOSES ONLY and is NOT FOR SALE NOR FOR REPRODUCTION.
63

Example 1.

A metal bar at a temperature of 100℉ is placed in a room at a constant temperature of


0℉. If after 20 minutes the temperature of the bar is 50℉, find (a) the time it will take the bar
to reach a temperature of the bar 25℉ and (b) the temperature of the bar after 10 minutes.

Solution to Example 1.

Given 𝑇𝑚 = 0; the medium here is the room which is being held at a constant temperature of
0℉. Thus, we have 𝑑𝑇⁄𝑑𝑡 + 𝑘𝑇 = 0, a linear equation whose solution is

𝑇 = 𝑐𝑒 −𝑘𝑡 (1)
Since 𝑇 = 100 at 𝑡 = 0 (the temperature of the bar is initially 100℉), it follows from
(1) that
100 = 𝑐𝑒 −𝑘(0)
100 = 𝑐
Substituting this value into (1), we obtain

𝑇 = 100𝑒 −𝑘𝑡 (2)

At 𝑡 = 20, we are given that 𝑇 = 50; hence, from (2)


50 = 100𝑒 −𝑘(20)
1 50
𝑘 = − ln = 0.035
20 100

Substituting this value into (2), we obtain the temperature of the bar at any time 𝑡 as
𝑇 = 100𝑒 −0.035 (3)

(a) We require 𝑡 when 𝑇 = 25. Substituting 𝑇 = 25 into (3) we have


25 = 100𝑒 −0.035𝑡

Solving, we find that 𝒕 = 𝟑𝟗. 𝟔 𝒎𝒊𝒏.

(b) We require 𝑇 when 𝑡 = 10. Substituting 𝑡 = 10 into (3) and then solving for 𝑇,
we find that
𝑇 = 100𝑒 (−0.035)(10) = 100 (0.705) = 𝟕𝟎. 𝟓℉

This module is a property of Technological University of the Philippines Visayas and intended
for EDUCATIONAL PURPOSES ONLY and is NOT FOR SALE NOR FOR REPRODUCTION.
64

Example 2.

𝑑𝜃
Assume that a body cools according to Newton’s Law 𝑑𝑡 = −𝑘𝜃 where 𝜃 is the
difference between the temperature 𝑇 of the body and that of the surrounding air. Find the
temperature 𝑇 at time 𝑡 of a boiler of water cooling in air at 0°𝐶 if the water is initially
boiling at 100℃ and the temperature dropped 10℃ during the first 20 min. Also, find the
time for the temperature of the water to drop from 90℃ to 80℃ and the temperature of the
water after 90 mins.

Solution to Example 2.

𝑑𝜃
= −𝑘𝜃 𝑤ℎ𝑒𝑟𝑒 𝜃 = 𝑇 − 𝑇𝑚
𝑑𝑡
𝑑𝜃
= −𝑘𝑑𝑡
𝜃
𝑑𝜃
∫ = ∫ −𝑘𝑑𝑡
𝜃
ln 𝜃 = −𝑘𝑡 + ln 𝐶
𝜃
ln = −𝑘𝑡
𝐶
𝜃 = 𝐶𝑒 −𝑘𝑡

𝑇 − 𝑇𝑚 = 𝐶𝑒 −𝑘𝑡

when 𝑡 = 0, 𝑇 = 100℃, 𝑇𝑚 = 0℃
substituting,
100℃ = 𝐶𝑒 −𝑘(0)
𝐶 = 100
thus,
𝑇 = 100𝑒 −𝑘𝑡

temperature dropped 10℃ during the first 20 min. hence,

𝑇 − 𝑇𝑚 = 100𝑒 −𝑘𝑡

𝑇 = 90℃, 𝑇𝑚 = 0℃,
90 = 100𝑒 −𝑘(20)

to simplify the equation, we define 𝑒 −𝑘


1
9 20
𝑒 −𝑘 =( )
10
therefore,
𝑡
𝑇 = 100(0.9)20 particular solution

This module is a property of Technological University of the Philippines Visayas and intended
for EDUCATIONAL PURPOSES ONLY and is NOT FOR SALE NOR FOR REPRODUCTION.
65

The time required for temperature to drop from 100℃ to 90℃ is 20 minutes.

The time required for the temperature to drop from 100℃ to 80℃ is:
𝑡
80 = 100(0.9)20
𝑡
0.8 = (0.9)20
𝑡
ln 0.8 = ln(0.9)20
𝑡
ln 0.8 = ln(0.9)
20
20 ln 0.8
𝑡= = 42.36 𝑚𝑖𝑛.
ln 0.9
Therefore, the time required for temperature to drop from 90 to 80℃ is

42.36 − 20 = 𝟐𝟐. 𝟑𝟔 𝒎𝒊𝒏.

The temperature of the water after 90 minutes is:


90
𝑇 = 100(0.9)20 = 𝟔𝟐. 𝟐℃

Example 3.

A body at an unknown temperature is placed in a room which is held at constant


temperature of 30 ℉. If after 10 minutes the temperature of the body is 0℉, and after 20
minutes the temperature of the body is 15 ℉, find the unknown initial temperature.

Solution to Example 3.

Given: 𝑇𝑚 = 30℉ Required: 𝑇 at 𝑡 = 0

𝑡 = 10 𝑚𝑖𝑛. , 𝑇 = 0℉

𝑡 = 20 𝑚𝑖𝑛. , 𝑇 = 15℉

Solving we obtain,
𝑇 = 𝐶𝑒 −𝑘𝑡 + 30 (1)
at 𝑡 = 10, 𝑇 = 0.
0 = 𝐶𝑒 −10𝑘 + 30 or 𝐶𝑒 −10𝑘 = −30 (2)

at 𝑡 = 20, 𝑇 = 15
15 = 𝐶𝑒 −20𝑘 + 30 or 𝐶𝑒 −20𝑘 = −15 (3)

Solving for 𝑘 and 𝐶 using (2) and (3), we find

15
(− ) 𝑒 −10𝑘 = −30
𝑒 −20𝑘
𝑒 10𝑘 = 2
This module is a property of Technological University of the Philippines Visayas and intended
for EDUCATIONAL PURPOSES ONLY and is NOT FOR SALE NOR FOR REPRODUCTION.
66

ln 𝑒 10𝑘 = ln 2
1
𝑘= ln 2 = 0.069
10
and
𝐶 = −30𝑒 10𝑘 = −30(2) = −60

Substituting these values into equation (1), we have for the temperature of the body at
any time 𝑡
𝑇 = (−60)𝑒 −0.069𝑡 + 30

Therefore, at initial time 𝑡 = 0 the temperature of the body is

𝑇 = (−60)𝑒 −0.069(0) + 30 = −𝟑𝟎℉

Growth and Decay

Let 𝑁(𝑡) denote the amount of substance (or population) that is either growing or
decaying. If assume that 𝑑𝑁⁄𝑑𝑡, the time rate of change of this amount of substance, is
proportional to the amount of substance present,
𝑑𝑁
∝𝑁
𝑑𝑡
then,
𝑑𝑁 𝑑𝑁
= 𝑘𝑁 𝑜𝑟 − 𝑘𝑁 = 0
𝑑𝑡 𝑑𝑡

where 𝑘 is the constant of proportionality.

Example 4.

If the population of a country doubles in 50 years, in how many years will it treble under
the assumption that the rate of increase is proportional to the number of inhabitants?

Solution to Example 4.
𝑑𝑁
= 𝑘𝑁
𝑑𝑡
𝑑𝑁
∫ = ∫ 𝑘𝑑𝑡
𝑁
ln 𝑁 = 𝑘𝑡 + ln 𝐶
𝑁
ln = 𝑘𝑡
𝐶
𝑵 = 𝑪𝒆𝒌𝒕

This module is a property of Technological University of the Philippines Visayas and intended
for EDUCATIONAL PURPOSES ONLY and is NOT FOR SALE NOR FOR REPRODUCTION.
67

at 𝑡 = 0, 𝑁 = 𝑁0 . where 𝑁0 is the original population


𝑁0 = 𝐶𝑒 𝑘(0)
𝐶 = 𝑁0
Substituting 𝐶 to the general solution, becomes
𝑁 = 𝑁0 𝑒 𝑘𝑡

Solving for 𝑘 when 𝑡 = 50, 𝑁 = 2𝑁𝑜


2𝑁0 = 𝑁0 𝑒 𝑘(50)
2 = 𝑒 50𝑘
ln 2 = ln 𝑒 50𝑘
1
ln 2 = 𝑘
50
Therefore, particular solution would be
1
𝑁 = 𝑁0 𝑒 50 ln 2𝑡 particular solution

the time it will take the population to treble (3𝑥) is

1
3𝑁0 = 𝑁0 𝑒 50 ln 2𝑡
1
3 = 𝑒 50 ln 2𝑡
1
ln 3 = ln 𝑒 50 ln 2𝑡
1
ln 3 = ln 2 𝑡
50
50 ln 3
𝑡= = 𝟕𝟗 𝒚𝒆𝒂𝒓𝒔
ln 2

Example 5.
A certain radioactive material is known to decay at a rate proportional to the amount
present. If initially there is 50 milligrams of the material present and after two hours it is
observed that the material has lost 10% of its original mass, find (a) an expression for the
mass of the material remaining at any time t, (b) the mass of the material after four hours; and
(c) the time at which the material has decayed to one half of its initial mass.

Solution to Example 5.
(a) Let 𝑁 denote the amount of materials present at time 𝑡.
𝑁 = 𝐶𝑒 𝑘𝑡 (1)

At 𝑡 = 0, we are given that 𝑁 = 50, therefore

50 = 𝐶𝑒 𝑘(0)
𝐶 = 50

This module is a property of Technological University of the Philippines Visayas and intended
for EDUCATIONAL PURPOSES ONLY and is NOT FOR SALE NOR FOR REPRODUCTION.
68

Substituting to (1)
𝑁 = 50𝑒 𝑘𝑡 (2)
At 𝑡 = 2, 10% of the original mass of 50 mg, or 5 mg, has decayed. Hence, at 𝑡 = 2,
𝑁 = 50 − 5 = 45 mg. Substituting these values to equation (2) in order to solve for
𝑘, we have
45 = 50𝑒 𝑘(2)
45
ln = ln 𝑒 2𝑘
50
1 45
ln = 𝑘 = −0.053
2 50
Substituting these values to (2), we obtain the amount of mass present at any time 𝑡 as

𝑵 = 𝟓𝟎𝒆−𝟎.𝟎𝟓𝟑𝒕 particular solution

(b) At 𝑡 = 4, 𝑁 is
𝑁 = 50𝑒 −0.053(4) = 50 (0.809) = 𝟒𝟎. 𝟓 mg

50
(c) the time at which the material has decayed to ½ of its initial mass, 𝑁 = = 25 mg,
2
is
25 = 50𝑒 −0.053(𝑡)
1 25
− ln = 𝑡 = 𝟏𝟑 𝒉𝒐𝒖𝒓𝒔
0.053 50

Example 6.
A bacteria culture is known to grow at a rate proportional to the amount present. After
one hour, 1000 strands of the bacteria are observed in the culture; and after four hours, 3000
strands. Find (a) an expression for the number of strands of the bacteria present in the culture
at any time t and (b) the number strands of the bacteria originally in the culture.

Solution to Example 6.
(a) The solution to this differential equation is
𝑁 = 𝑐𝑒 𝑘𝑡 (1)
At 𝑡 = 1, 𝑁 = 1000; hence,
1000 = 𝐶𝑒 𝑘 (2)
At 𝑡 = 4, 𝑁 = 3000; hence,
3000 = 𝐶𝑒 4𝑘 (3)
Solving (2) and (3) for 𝑘 and 𝐶, we find
𝑘 = 1⁄3 ln 3 = 0.366 and 𝐶 = 1000𝑒 0.366 = 694
Substituting these values of 𝑘 and 𝐶 into (1), we obtain

𝑵 = 𝟔𝟗𝟒𝒆𝟎.𝟑𝟔𝟔𝒕
as an expression for the amount of the bacteria present at any time 𝑡.
This module is a property of Technological University of the Philippines Visayas and intended
for EDUCATIONAL PURPOSES ONLY and is NOT FOR SALE NOR FOR REPRODUCTION.
69

(b) At 𝑡 = 0, 𝑁 is
𝑁 = 694𝑒 0.366(0) = 𝟔𝟗𝟒.

This module is a property of Technological University of the Philippines Visayas and intended
for EDUCATIONAL PURPOSES ONLY and is NOT FOR SALE NOR FOR REPRODUCTION.
70

PROGRESS CHECK
Activity Sheets
EXERCISE 5
Review on Applications of Differential Equations of Order One

Name: _____________________________________________Year & Section: __________


Subject: ____________________________________________Date Submitted: __________
Professor: __________________________________________Score: __________________

Solve the following problems.


Newton’s Law of Cooling
1. A body of unknown temperature is placed in a refrigerator at a constant temperature
of 0℉. If after 20 minutes the temperature of the body is 40℉ and after 40 minutes
the temperature of the body is 20℉, find the initial temperature of the body.

2. A thermometer reading 18℉ is brought into a room where the temperature is 70℉; 1
minute later the thermometer reading is 31℉. Determine the temperature reading as a
function of time and, in particular, find the temperature reading 5 minutes after the
thermometer is first brought into the room.

3. A thermometer reading 75℉ is taken out where the temperature is 20℉, the reading is
30℉ 4 minutes later. Find (a) the thermometer reading 7 minutes after the
thermometer was brought outside, and (b) the time taken for the reading to drop from
75℉ to within a half degree of the air temperature.
Growth and Decay
4. The population of a certain state is known to grow at a rate proportional to the number
of people presently living in the state. If after 10 years the population has trebled and
if after 20 years the population is 150, 000, find the number of people initially living
in the state.
5. In a culture of yeast, the amount of active ferment grows at a rate proportional to the
amount present. If the amount doubles in 1 hour, how many times the original amount
may be anticipated at the end of 2.75 hours.
6. A certain radioactive material is known to decay at a rate proportional to the amount
present. If initially there are 100 milligrams of the material present, and if after two
years it is observed that 5% of the original mass has decayed, find (a) an expression
for the mass at any time 𝑡 and (b) the time necessary for the 10% of the original mass
to have decayed.

This module is a property of Technological University of the Philippines Visayas and intended
for EDUCATIONAL PURPOSES ONLY and is NOT FOR SALE NOR FOR REPRODUCTION.
71

Other Applications of Order One Differential Equations


7. Assume that the rate of change of air pressure with altitude (distance above the earth)
is proportional to the air pressure. If the air pressure on the ground is 14.7 psi and if at
an altitude of 10, 000 ft it is 10 psi, find the air pressure at an altitude, and find the air
pressure at an altitude of 15, 000 ft.
Ans. 8.372 psi

8. Water leaks from a cylinder with axis vertical through a small orifice in its base at a
rate proportional to the square root of the volume remaining any time. If the cylinder
contains 64 gal. initially and 15 gal. leak out the first day, when will 25 gal. remains?
How much will remain at the end of 4 days?
Ans. 3 days, 16 gal.

9. If 𝐼 represents the intensity of light which has penetrated water to a depth of 𝑥 ft., then
4
the rate of change of 𝐼 is proportional to 𝐼. If 𝐼 at depth 30 ft is the intensity at the
9
surface, find the intensities at depths 60 ft and 180 ft.
256
Ans. 𝐼 = 6561 𝐼𝑜

10. A boat with its load weighs 382 lb. If the force exerted upon the boat by the motor in
the direction of motion is equivalent to a constant force of 15 lb., if the resistance (in
pounds) to motion is equal to twice the speed (in ft. per second) that is 2𝑣 lb., and if
the boat starts from rest, find the speed of (a) 𝑡 seconds; (b) after 10 seconds (c) when
𝑡 = ∞, that is, the limiting speed.

This module is a property of Technological University of the Philippines Visayas and intended
for EDUCATIONAL PURPOSES ONLY and is NOT FOR SALE NOR FOR REPRODUCTION.
72

REFERENCES
Sergio E. Ymas Jr. Differential Equations (Simplified Approach). Sampaloc, Manila. Pablo.
L. Bustamante III Press, 1997. Chapter 4.

Ricardo C. Asin. Elementary Differential Equations Reviewer. Manila, Philippines, Merriam


& Webster Bookstore, Inc., 1991. pg. 39 to 50.

This module is a property of Technological University of the Philippines Visayas and intended
for EDUCATIONAL PURPOSES ONLY and is NOT FOR SALE NOR FOR REPRODUCTION.
73

LEARNING GUIDE

Week No.: __7&8__

TOPIC/S

I. Methods of Solving Linear Differential Equations of nth Order


a. Differential Operators
II. Solutions of General Linear Homogeneous Equations
a. Distinct Real Roots of Auxiliary Equations
b. Repeated Real Roots of Auxiliary Equations
c. Distinct Imaginary Roots of Auxiliary Equations
d. Repeated Imaginary Roots of Auxiliary Equations

EXPECTED COMPETENCIES
At the end of the week after studying the module, the student should be able to:
1. use the exponential shifts to solve the differential equation;
2. solve solutions of general linear homogeneous differential equations using auxiliary
equation methods:
a. distinct real roots;
b. repeated real roots;
c. distinct imaginary roots; and
d. repeated imaginary roots.

CONTENT/TECHNICAL INFORMATION

Differential Operators

In mathematics, a differential operator is an operator defined as a function of the


differentiation operator. It is helpful, as a matter of notation first, to consider differentiation
as an abstract operation that accepts a function and returns another function.

𝑑𝑦
Aside from the notation 𝑦 ′ = , we shall define and introduce other notations for
𝑑𝑥
differentiation. We shall define 𝐷 as a differential operator, and
𝑑𝑦
𝐷𝑦 = , 1st differentiation with respect to 𝑥.
𝑑𝑥
𝑑2𝑦
𝐷2 𝑦 = , 2nd differentiation with respect to 𝑥.
𝑑𝑥 2

𝑑𝑘 𝑦
𝐷𝑘 𝑦 = , kth differentiation with respect to 𝑥.
𝑑𝑥 𝑘

This module is a property of Technological University of the Philippines Visayas and intended
for EDUCATIONAL PURPOSES ONLY and is NOT FOR SALE NOR FOR REPRODUCTION.
74

Theorem 7.1 Let 𝑓(𝐷) be a polynomial in 𝐷, then

𝑓(𝐷 + 𝑎)(𝑒 𝑎𝑥 )𝑦 = 𝑒 𝑎𝑥 𝑓(𝐷)𝑦

The above relation shows how to shift an exponential factor, from the left of the
differential operator to the right of the operator.

Example 1. Use the exponential shift to solve the differential equation


(𝐷 + 2)3 𝑦 = 0

Solution to Example 1.
Multiply the equation by 𝑒 2𝑥
𝑒 2𝑥 (𝐷 + 2)3 𝑦 = 0
Applying exponential shift leads to
𝐷3 (𝑒 2𝑥 𝑦) = 0
Integrating three times,
𝐷2 (𝑒 2𝑥 𝑦) = 𝑐1
𝐷(𝑒 2𝑥 𝑦) = 𝑐1 𝑥 + 𝑐2
𝑒 2𝑥 𝑦 = 𝑐1 𝑥 2 + 𝑐2 𝑥 + 𝑐3
Thus, the solution is given by
𝒚 = 𝒆−𝟐𝒙 (𝒄𝟏 𝒙𝟐 + 𝒄𝟐 𝒙 + 𝒄𝟑 ).

Example 2. Use the exponential shift to solve the differential equation


(𝐷 − 5)4 𝑦 = 0
Solution to Example 2.
Multiply the equation by 𝑒 −5𝑥
𝑒 −5𝑥 (𝐷 − 5)4 𝑦 = 0
Applying the exponential shift gives
𝐷4 (𝑒 −5𝑥 )𝑦 = 0
Integrating four times, we have the following:
𝐷3 (𝑒 −5𝑥 𝑦) = 𝑐1
𝐷2 (𝑒 −5𝑥 𝑦) = 𝑐1 𝑥 + 𝑐2
𝐷(𝑒 −5𝑥 𝑦) = 𝑐1 𝑥 2 + 𝑐2 𝑥 + 𝑐3
𝑒 −5𝑥 𝑦 = 𝑐1 𝑥 3 + 𝑐2 𝑥 2 + 𝑐3 𝑥 + 𝑐4
𝒚 = 𝒆𝟓𝒙 (𝒄𝟏 𝒙𝟑 + 𝒄𝟐 𝒙𝟐 + 𝒄𝟑 𝒙 + 𝒄𝟒 )

This module is a property of Technological University of the Philippines Visayas and intended
for EDUCATIONAL PURPOSES ONLY and is NOT FOR SALE NOR FOR REPRODUCTION.
75

SOLUTIONS OF GENERAL LINEAR HOMOGENEOUS EQUATIONS

The form of a general linear differential equation with constant coefficient is

𝑑𝑛 𝑦 𝑑𝑛−1 𝑦 𝑑𝑦
𝑎0 𝑛
+ 𝑎1 𝑛−1
+ ⋯ + 𝑎𝑛−1 + 𝑎𝑛 𝑦 = 0
𝑑𝑥 𝑑𝑥 𝑑𝑥

This form may also be written in a compact form as

𝑓(𝐷)𝑦 = 0

where 𝑓(𝐷)𝑦 is a linear differential operation.

Theorem 7.2 If 𝑚 is a root of the equation 𝑓(𝑚) = 0, then

𝑓(𝐷)𝑦 = 𝑓(𝐷)𝑒 𝑚𝑥 = 0.

The above theorem asserts that 𝑦 = 𝑒 𝑚𝑥 is a solution of 𝑓(𝐷)𝑦 = 0. In general

𝑦 = 𝑐𝑒 𝑚𝑥 , 𝑐 constant

We shall define 𝑓(𝑚) = 0 as the auxiliary equation.

The roots of auxiliary equation is classified into 4 cases:

Case 𝟏: Distinct Real Root


Case 𝟐: Repeated Real Root
Case 𝟑: Distinct Imaginary Root
Case 𝟒: Repeated Imaginary Root

This module is a property of Technological University of the Philippines Visayas and intended
for EDUCATIONAL PURPOSES ONLY and is NOT FOR SALE NOR FOR REPRODUCTION.
76

Case 𝟏: Distinct Real Roots of Auxiliary Equation


Let the roots of the auxiliary equation be 𝑚1 , 𝑚2 , … , 𝑚𝑘 . Then these will produce 𝑘
solutions as follows:
𝑦1 = 𝑒 𝑚1 𝑥 , 𝑦2 = 𝑒 𝑚2 𝑥 , … , 𝑦𝑘 = 𝑒 𝑚𝑘 𝑥
The general solution, denoted by 𝑦𝑐 , of the general linear homogeneous differential equation
with 𝑚1 , 𝑚2 , … , 𝑚𝑘 roots of the auxiliary operation is

𝒚𝒄 = 𝒄𝟏 𝒆𝒎𝟏𝒙 + 𝒄𝟐 𝒆𝒎𝟐𝒙 + ⋯ + 𝒄𝒌 𝒆𝒎𝒌 𝒙

Example 3. Solve the equation


𝑦 ′′′ − 𝑦 ′′ − 2𝑦 ′ = 0
Solution to Example 3.
The differential equation is translated in terms of differential operator 𝐷 as
𝐷3 𝑦 − 𝐷2 𝑦 − 2𝐷𝑦 = 0
(𝐷3 − 𝐷2 − 2𝐷)𝑦 = 0
The auxiliary equation is
𝑓(𝑚) = 𝑚3 − 𝑚2 − 2𝑚 = 0
The goal is to find the roots of 𝑚 by factoring,
𝑚(𝑚 − 2)(𝑚 + 1) = 0
𝑚 = 0, 𝑚 = 2, 𝑚 = −1
Therefore, the roots of auxiliary equation are 0, 2, −1. Since the roots are classified as distinct
real roots, thus the general solution fall on case 1

𝑦 = 𝑐1 𝑒 (0)𝑥 + 𝑐2 𝑒 2𝑥 + 𝑐3 𝑒 (−1)𝑥

𝒚 = 𝒄𝟏 + 𝒄𝟐 𝒆𝟐𝒙 + 𝒄𝟑 𝒆−𝒙

Note: The arrangement of roots of auxiliary equation doesn’t affect the particular solution.

This module is a property of Technological University of the Philippines Visayas and intended
for EDUCATIONAL PURPOSES ONLY and is NOT FOR SALE NOR FOR REPRODUCTION.
77

Example 4. Solve the equation


𝑑3 𝑦 𝑑𝑦
4 3
− 13 + 6𝑦 = 0
𝑑𝑥 𝑑𝑥
Solution to Example 2.
Express first the given differential equation in terms of differential operator 𝐷.
4𝐷3 𝑦 − 13𝐷𝑦 + 6𝑦 = 0
(4𝐷3 − 13𝐷 + 6)𝑦 = 0
The auxiliary equation is
𝑓(𝑚) = 4𝑚3 − 13𝑚 + 6 = 0
By synthetic division, we have:
4 0 − 13 6 ⌊−2
−8 16 −6
________________________
4 −8 3 0

Thus, 4𝑚3 − 13𝑚 + 6 = (𝑚 + 2)(4𝑚2 − 8𝑚 + 3) = 0


(𝑚 + 2)(2𝑚 − 1)(2𝑚 − 3) = 0
1 3
𝑚 = −2, 𝑚= , 𝑚=
2 2
1 3
The roots of the auxiliary equation are −2, , . The general solution is
2 2
𝟏 𝟑
𝒚 = 𝒄𝟏 𝒆−𝟐𝒙 + 𝒄𝟐 𝒆𝟐𝒙 + 𝒄𝟑 𝒆𝟐𝒙
Example 5. Solve the equation
(𝐷2 + 2𝐷 − 1)𝑦 = 0
Solution to Example 3.
The auxiliary equation is
𝑓(𝑚) = 𝑚2 + 𝑚 − 1 = 0
The roots of auxiliary equation are obtained by quadratic formula,
−2 ± √4 + 4
𝑚= = −1 ± √2
2
Thus, the solution is given by

𝒚 = 𝒄𝟏 𝒆(−𝟏+√𝟐)𝒙 + 𝒄𝟐 𝒆(−𝟏−√𝟐)𝒙

This module is a property of Technological University of the Philippines Visayas and intended
for EDUCATIONAL PURPOSES ONLY and is NOT FOR SALE NOR FOR REPRODUCTION.
78

Example 6. Solve the equation


(𝐷2 − 2𝐷 − 3)𝑦 = 0 when 𝑥 = 0, 𝑦 = 0, and 𝑦 ′ = 4
Solution to Example 6.
The auxiliary equation is
𝑓(𝑚) = 𝑚2 − 2𝑚 − 3 = 0
(𝑚 − 3)(𝑚 + 1) = 0
𝑚 = 3, 𝑚 = −1
The roots of auxiliary equation are 3 and −1. Thus, the general solution is
𝑦 = 𝑐1 𝑒 3𝑥 + 𝑐2 𝑒 −𝑥
Since condition is given, when 𝑥 = 0, 𝑦 = 0 and 𝑦 ′ = 4, we need to substitute these values in
order to obtain the particular solution.
First, derive the general solution to achieve 𝑦′
𝑦 = 𝑐1 𝑒 3𝑥 + 𝑐2 𝑒 −𝑥 (1)
𝑦′ = 3𝑐1 𝑒 3𝑥 − 𝑐2 𝑒 −𝑥 (2)
when 𝑥 = 0, 𝑦 = 0, substitute these values using eq. 1, to obtain values of 𝑐1 and/or 𝑐2

0 = 𝑐1 𝑒 3(0) + 𝑐2 𝑒 −(0)
0 = 𝑐1 + 𝑐2
𝑐1 = −𝑐2
When 𝑥 = 0, 𝑦 ′ = 4, substitute these values using eq. 2,

4 = 3𝑐1 𝑒 3(0) − 𝑐2 𝑒 −(0)


4 = 3𝑐1 − 𝑐2
Since 𝑐1 = −𝑐2,
4 = 3(−𝑐2 ) − 𝑐2
4 = −4𝑐2
𝑐2 = −1
therefore,
𝑐1 = −𝑐2 = −(−1) = 1
𝑐1 = 1
Substitute the values of 𝑐1 and 𝑐2 , to eq. 1, thus the particular solution is

𝒚 = 𝒆𝟑𝒙 − 𝒆−𝒙
This module is a property of Technological University of the Philippines Visayas and intended
for EDUCATIONAL PURPOSES ONLY and is NOT FOR SALE NOR FOR REPRODUCTION.
79

Case 𝟐: Repeated Real Roots of Auxiliary Equation


Suppose we are given the differential equation
(𝐷 − 𝑎)2 𝑦 = 0
then the roots of the auxiliary equation are 𝑎 and 𝑎 (repeated). Then using case 1, the solution
is given by
𝑦 = 𝑐1 𝑒 𝑎𝑥 + 𝑐2 𝑒 𝑎𝑥
𝑦 = 𝑒 𝑎𝑥 (𝑐1 + 𝑐2 )
𝑦 = 𝑐𝑒 𝑎𝑥
The last equation asserts that the auxiliary equation has only one root 𝑎, which leads to a
contradiction of repeated roots. This is now where the role of Theorem 7.1 becomes very
helpful. Multiplying both sides of the given differential equation by 𝑒 𝑎𝑥 gives
𝑒 −𝑎𝑥 (𝐷 − 𝑎)2 𝑦 = 0
By exponential shift, we have
𝐷2 (𝑒 −𝑎𝑥 𝑦) = 0
Integrating two lines, we obtain the general solution:
𝐷(𝑒 −𝑎𝑥 𝑦) = 𝑐2
𝑒 −𝑎𝑥 𝑦 = 𝑐1 + 𝑐2 𝑥

𝒚 = 𝒆𝒂𝒙 (𝒄𝟏 + 𝒄𝟐 𝒙)

Note: arrangement of 𝑐1 and 𝑐2 does not affect the particular solution.

As a consequence, for the differential equation


(𝐷 − 𝑎)3 𝑦 = 0
where the roots repeated 3 times, then the solution is
𝑦 = 𝑒 𝑎𝑥 (𝑐1 + 𝑐2 𝑥 + 𝑐3 𝑥 2 )
𝑦 = 𝑐1 𝑒 𝑎𝑥 + 𝑐2 𝑥𝑒 𝑎𝑥 + 𝑐3 𝑥 2 𝑒 𝑎𝑥

Example 7. Solve the equation


𝑑 3 𝑦 𝑑 2 𝑦 𝑑𝑦
− − +𝑦 =0
𝑑𝑥 3 𝑑𝑥 2 𝑑𝑥

This module is a property of Technological University of the Philippines Visayas and intended
for EDUCATIONAL PURPOSES ONLY and is NOT FOR SALE NOR FOR REPRODUCTION.
80

Solution to Example 7.
First, express the given differential equation in terms of differential operator 𝐷,
𝐷3 𝑦 − 𝐷2 𝑦 − 𝐷𝑦 + 𝑦 = 0
(𝐷3 − 𝐷2 − 𝐷 + 1)𝑦 = 0
The auxiliary equation is
𝑓(𝑚) = 𝑚3 − 𝑚2 − 𝑚 + 1 = 0
(𝑚 − 1)2 (𝑚 + 1) = 0
The roots of auxiliary equation are −1, 1, 1. Thus the general solution is a combination of
case 1 and case 2. For root −1 the solution fall on case 1, and for root 1 (repeated) the
equation falls on case 2. Thus, the general solution is,
𝒚 = 𝒄𝟏 𝒆−𝒙 + 𝒆𝒙 (𝒄𝟐 + 𝒄𝟑 𝒙)

Example 8. Solve the equation


(4𝐷4 − 4𝐷3 − 23𝐷2 + 12𝐷 + 36)𝑦 = 0
Solution to Example 8.
The auxiliary equation is
𝑓(𝑚) = 4𝑚4 − 4𝑚3 − 23𝑚2 + 12𝑚 + 36 = 0
To find the roots, we use synthetic division,
4 − 4 − 23 12 36 ⌊2
8 8 − 30 − 36
__________________________
4 4 − 15 − 18 0

Thus, 𝑚 = 2 is a root. Again, by synthetic division:


4 4 − 15 − 18 ⌊2
8 24 18
________________________
4 12 9 0
Thus,
4𝑚4 − 4𝑚3 − 23𝑚2 + 12𝑚 + 36 = 0
(𝑚 − 2)(𝑚 − 2)(4𝑚2 + 12𝑚 + 9) = 0
(𝑚 − 2)(𝑚 − 2)(2𝑚 + 3)2 = 0
3 3
Thus, the roots are 2, 2, − 2 , − 2. The solution is
𝟑
𝒚 = 𝒆𝟐𝒙 (𝒄𝟏 + 𝒄𝟐 𝒙) + 𝒆−𝟐𝒙 (𝒄𝟑 + 𝒄𝟒 𝒙)

This module is a property of Technological University of the Philippines Visayas and intended
for EDUCATIONAL PURPOSES ONLY and is NOT FOR SALE NOR FOR REPRODUCTION.
81

Example 9. Solve the equation


(𝐷2 + 4𝐷 + 4)𝑦 = 0 when 𝑥 = 0, 𝑦 = 1 and 𝑦 ′ = −1
Solution to Example 9.
The auxiliary equation is
𝑓(𝑚) = 𝑚2 + 4𝑚 + 4 = 0
(𝑚 + 2)(𝑚 + 2) = 0
The roots are −2, −2. Thus, the solution fall on case 2: repeated real roots
𝑦 = 𝑒 −2𝑥 (𝑐1 + 𝑐2 𝑥) (1)
Since condition is given, when 𝑥 = 0, 𝑦 = 1 and 𝑦 ′ = −1, we need to substitute these values
in order to obtain the particular solution.
First, derive the general solution to achieve 𝑦′
Notice the solution can be derive using 𝑑(𝑢𝑣) = 𝑢𝑑𝑣 + 𝑣𝑑𝑢, where 𝑢 = 𝑒 −2𝑥 and
𝑣 = (𝑐1 + 𝑐2 𝑥).
𝑦 ′ = 𝑒 −2𝑥 (𝑐2 ) + (𝑐1 + 𝑐2 𝑥)(−2𝑒 −2𝑥 ) (2)
when 𝑥 = 0, 𝑦 = 1, use eq. 1

1 = 𝑒 −2(0) (𝑐1 + 𝑐2 (0))

𝑐1 = 1
when 𝑥 = 0, 𝑦 ′ = −1, use eq. 2

−1 = 𝑒 −2(0) (𝑐2 ) + (𝑐1 + 𝑐2 (0))(−2𝑒 −2(0) )

−1 = 𝑐2 − 2𝑐1
since 𝑐1 = 1, therefore 𝑐2 = 1
Substituting the values of 𝑐1 and 𝑐2 to equation 1, the particular solution is

𝒚 = 𝒆−𝟐𝒙 (𝟏 + 𝒙)

Example 10. Solve the equation


(𝐷2 − 4)(𝐷 − 2)𝑦 = 0
Solution to Example 10.
The auxiliary equation is
𝑓(𝑚) = (𝑚 − 2)2 (𝑚 + 2) = 0
where roots are −2, 2, 2. The general solution is
𝒚 = 𝒄𝟏 𝒆−𝟐𝒙 + 𝒆𝟐𝒙 (𝒄𝟐 + 𝒄𝟑 𝒙)
This module is a property of Technological University of the Philippines Visayas and intended
for EDUCATIONAL PURPOSES ONLY and is NOT FOR SALE NOR FOR REPRODUCTION.
82

Case 𝟑: Imaginary Roots of Auxiliary Equation


Supposed the root of the auxiliary equation is imaginary 𝛼 ± 𝛽𝑖. Then, the solution
using case 1 is
𝑦 = 𝑐3 𝑒 𝑚1 𝑥 + 𝑐4 𝑒 𝑚2𝑥
𝑦 = 𝑐3 𝑒 (𝛼+𝛽𝑖)𝑥 + 𝑐4 𝑒 (𝛼−𝛽𝑖)𝑥
𝑦 = 𝑐3 𝑒 𝛼𝑥 ∙ 𝑒 𝛽𝑖𝑥 + 𝑐4 𝑒 𝛼𝑥 ∙ 𝑒 −𝛽𝑖𝑥
𝑦 = 𝑒 𝛼𝑥 (𝑐3 𝑒 𝛽𝑖𝑥 + 𝑐4 𝑒 −𝛽𝑖𝑥 )
From Calculus, 𝑒 𝑖𝜃 is defined by
𝑒 𝑖𝜃 = cos 𝜃 + 𝑖 sin 𝜃
The above equation is called the Euler Equation.
Considering the last equation if 𝜃 = 𝛽𝑥, then
𝑒 𝑖(𝛽𝑥) = cos(𝛽𝑥) + 𝑖 sin(𝛽𝑥)
and
𝑒 𝑖(−𝛽𝑥) = cos(−𝛽𝑥) + 𝑖 sin(−𝛽𝑥)
𝑒 𝑖(−𝛽𝑥) = cos(𝛽𝑥) − 𝑖 sin(𝛽𝑥)
therefore,
𝑦 = 𝑒 𝛼𝑥 [𝑐3 (cos 𝛽𝑥 + 𝑖 sin 𝛽𝑥) + 𝑐4 (cos 𝛽𝑥 − 𝑖 sin 𝛽𝑥)]
𝑦 = 𝑒 𝛼𝑥 [𝑐3 cos 𝛽𝑥 + 𝑖 𝑐3 sin 𝛽𝑥 + 𝑐4 cos 𝛽𝑥 − 𝑖𝑐4 sin 𝛽𝑥]
𝑦 = 𝑒 𝛼𝑥 [(𝑐3 + 𝑐4 ) cos 𝛽𝑥 + (𝑖𝑐3 − 𝑖𝑐4 ) sin 𝛽𝑥]

If we let (𝑐3 + 𝑐4 ) = 𝑐1 and (𝑖𝑐3 − 𝑖𝑐4 ) = 𝑐2 , where 𝑐1 and 𝑐2 are new arbitrary constants,
then the general solution for case 3: Distinct Imaginary Root is

𝒚 = 𝒆𝜶𝒙 [𝒄𝟏 𝐜𝐨𝐬 𝜷𝒙 + 𝒄𝟐 𝐬𝐢𝐧 𝜷𝒙]

Example 11. Solve the equation


(𝐷2 + 4)𝑦 = 0
Solution to Example 11.
First, determine the values of 𝛼 and 𝛽. The auxiliary equation is
𝑓(𝑚) = 𝑚2 + 4 = 0
𝑚 = ±2𝑖
From which 𝛼 = 0 and 𝛽 = 2. Substituting these values in general solution for case 3, we get
𝑦 = 𝑒 0𝑥 [𝑐1 cos 2𝑥 + 𝑐2 sin 2𝑥] or 𝒚 = 𝒄𝟏 𝐜𝐨𝐬 𝟐𝒙 + 𝒄𝟐 𝐬𝐢𝐧 𝟐𝒙

This module is a property of Technological University of the Philippines Visayas and intended
for EDUCATIONAL PURPOSES ONLY and is NOT FOR SALE NOR FOR REPRODUCTION.
83

Example 12. Solve the equation


(𝐷2 − 𝐷 + 2)𝑦 = 0
Solution to Example 12.
The auxiliary equation is
𝑓(𝑚) = 𝑚2 − 𝑚 + 2 = 0
Since we cannot factor the equation, to find the roots of auxiliary equation we use the
quadratic formula
−𝑏 ± √𝑏 2 − 4𝑎𝑐
𝑚=
2𝑎
−(−1) ± √(−1)2 − 4(1)(2)
𝑚=
2(1)
1 ± 𝑖√7
𝑚=
2
from which
1 √7
𝛼 ± 𝛽𝑖 ; 𝛼 = , 𝛽=
2 2
Thus, the solution of the differential equation is
𝒙 √𝟕 √𝟕
𝒚 = 𝒆𝟐 [𝒄𝟏 𝐜𝐨𝐬 𝒙 + 𝒄𝟐 𝐬𝐢𝐧 𝒙]
𝟐 𝟐

Example 13. Solve the equation


(𝐷4 − 1)𝑦 = 0
Solution to Example 13.
The auxiliary equation is
𝑓(𝑚) = 𝑚4 − 1 = 0
(𝑚2 + 1)(𝑚2 − 1) = 0

𝑚2 + 1 = 0 𝑚2 = 1
𝑚2 = −1 𝑚 = ±1
𝑚 = ±𝑖 𝑚 = 1, 𝑚 = −1

From the roots of auxiliary equation, the general solution is a combination of case 1 for 𝑚 =
±1 and case 3 with 𝛼 = 0 and 𝛽 = 1, then the general solution is

𝑦 = 𝑐1 𝑒 𝑥 + 𝑐2 𝑒 −𝑥 + 𝑒 (0)𝑥 [𝑐3 cos 𝑥 + 𝑐4 sin 𝑥] or


𝒚 = 𝒄𝟏 𝒆𝒙 + 𝒄𝟐 𝒆−𝒙 + 𝒄𝟑 𝐜𝐨𝐬 𝒙 + 𝒄𝟒 𝐬𝐢𝐧 𝒙

This module is a property of Technological University of the Philippines Visayas and intended
for EDUCATIONAL PURPOSES ONLY and is NOT FOR SALE NOR FOR REPRODUCTION.
84

Case 𝟒: Repeated Imaginary Auxiliary Equation


Repeated imaginary roots leads to solutions analogous to those of repeated real roots.
For example, if the roots 𝑚 = 1 ± 2𝑖 are repeated twice then with 𝛼 = 1 and 𝛽 = 2, the
solution is given by
𝑦 = 𝑒 𝑥 [(𝑐1 + 𝑐2 𝑥) cos 2𝑥 + (𝑐3 + 𝑐4 𝑥) sin 2𝑥 ]

Example 14. Solve the equation


(𝐷3 + 𝐷)2 𝑦 = 0
Solution to Example 14.
The auxiliary equation is
𝑓(𝑚) = (𝑚3 + 𝑚)2 = 0
𝑚2 (𝑚2 + 1)2 = 0

The roots of the auxiliary equation are


𝑚 = 0, 0, ±𝑖, ±𝑖
The solution is then a combination of case 2 with 𝑚 = 0 repeated twice and case 4 with
𝑚 = ±𝑖 where 𝛼 = 0 and 𝛽 = 1 repeated twice also.
𝑦 = 𝑒 (0)𝑥 (𝑐1 + 𝑐2 𝑥) + 𝑒 (0)𝑥 [(𝑐3 + 𝑐4 𝑥) cos 𝑥 + (𝑐5 + 𝑐6 𝑥) sin 𝑥] or
𝒚 = 𝒄𝟏 + 𝒄𝟐 𝒙 + (𝒄𝟑 + 𝒄𝟒 𝒙) 𝐜𝐨𝐬 𝒙 + (𝒄𝟓 + 𝒄𝟔 𝒙) 𝐬𝐢𝐧 𝒙

Example 15. Solve the equation


(𝐷4 + 6𝐷2 + 9)𝑦 = 0
Solution to Example 15.
The auxiliary equation is
𝑓(𝑚) = 𝑚4 + 6𝑚2 + 9 = 0
(𝑚2 + 3)2 = 0
The roots of the auxiliary equation are
𝑚 = ±3𝑖, ±3𝑖
With 𝛼 = 0, 𝛽 = 3, then the solution is
𝒚 = (𝒄𝟏 + 𝒄𝟐 𝒙) 𝐜𝐨𝐬 𝟑𝒙 + (𝒄𝟑 + 𝒄𝟒 𝒙) 𝐬𝐢𝐧 𝟑𝒙

This module is a property of Technological University of the Philippines Visayas and intended
for EDUCATIONAL PURPOSES ONLY and is NOT FOR SALE NOR FOR REPRODUCTION.
85

PROGRESS CHECK
Activity Sheets
EXERCISE 6
Review on Differential Operator, Solutions of General Linear Homogeneous Equations

Name: _____________________________________________Year & Section: __________


Subject: ____________________________________________Date Submitted: __________
Professor: __________________________________________Score: __________________

For number 1 to 10, use the exponential shifts to find the general solution.

1. (𝐷 + 9)4 𝑦 = 0 2. (𝐷 − 2)8 𝑦 = 0
3. (𝐷 + 7)5 𝑦 = 0 4. (𝐷 − 4)6 𝑦 = 0
5. (𝐷 + 1)10 𝑦 = 0 6. (𝐷 + 8)𝑦 = 0
7. (𝐷 − 3)5 𝑦 = 0 8. (𝐷 − 1)9 𝑦 = 0
9. (𝐷 + 6)2 𝑦 = 0 10. (𝐷 − 5)7 𝑦 = 0

Distinct Real Roots of Auxiliary Equation


For numbers 11 to 20, find the general solutions.
11. (𝐷2 − 5𝐷 + 4)𝑦 = 0 12. (10𝐷3 + 𝐷2 − 7𝐷 + 2)𝑦 = 0
13. (𝐷2 − 9)𝑦 = 0 14. (𝐷3 − 2𝐷2 − 𝐷)𝑦 = 0
15. (6𝐷2 − 11𝐷 − 10)𝑦 = 0 16. (𝐷3 + 2𝐷2 − 15𝐷)𝑦 = 0
17. (6𝐷2 − 5𝐷 − 6)𝑦 = 0 18. (𝐷3 − 4𝐷2 + 𝐷 + 6)𝑦 = 0
19. (𝐷3 − 5𝐷 − 2)𝑦 = 0 20. (4𝐷3 − 13𝐷 + 6)𝑦 = 0

Repeated Real Roots of Auxiliary Equation


For numbers 21 to 30, find the general solutions.
21. 4𝑦 ′′ − 6𝑦′ + 𝑦 = 0 22. (4𝐷 4 + 4𝐷3 − 3𝐷 2 − 2𝐷 + 1)𝑦 = 0
23. (𝐷3 + 3𝐷2 − 4)𝑦 = 0 24. (4𝐷4 − 4𝐷3 − 23𝐷2 + 12𝐷 + 36)𝑦 = 0
25. (4𝐷2 − 12𝐷 + 9)𝑦 = 0 26. (𝐷4 + 2𝐷3 + 𝐷2 )𝑦 = 0
27. 𝐷3 𝑦 + 3𝐷2 𝑦 + 3𝐷𝑦 + 𝑦 28. 4𝑦 ′′′ − 27𝑦 ′ + 27𝑦 = 0
29. (𝐷4 − 𝐷3 )𝑦 = 0 30. (36𝐷2 − 12𝐷 + 1)𝑦 = 0

This module is a property of Technological University of the Philippines Visayas and intended
for EDUCATIONAL PURPOSES ONLY and is NOT FOR SALE NOR FOR REPRODUCTION.
86

Distinct Imaginary Roots of Auxiliary Equation


For numbers 31 to 40, find the general solutions.
31. (𝐷2 − 2𝐷 + 2)𝑦 = 0 32. (𝐷4 + 𝐷2 )𝑦 = 0
33. (𝐷3 + 𝐷)𝑦 = 0 34. (𝐷3 − 𝐷2 + 𝐷)𝑦 = 0
35. (𝐷2 + 16)𝑦 = 0 36. (𝐷3 + 7𝐷2 + 19𝐷 + 13)𝑦 = 0
37. (3𝐷2 − 𝐷 − 1)𝑦 = 0 38. (𝐷3 + 2𝐷2 + 𝐷 + 2)𝑦 = 0
39. (𝐷3 + 25𝐷)𝑦 = 0 40. (𝐷3 − 3𝐷2 + 9𝐷 + 13)𝑦 = 0

Repeated Imaginary Roots of Auxiliary Equation


For numbers 41 to 50, find the general solutions.
41. (𝐷2 + 36)2 𝑦 = 0 42. (𝐷3 + 𝐷2 + 9𝐷 + 9)𝑦 = 0
43. (𝐷2 + 𝐷 + 2)2 𝑦 = 0 44. (𝐷2 + 4)(𝐷 + 3)𝑦 = 0
45. (8𝐷3 − 4𝐷2 − 2𝐷 + 1)𝑦 = 0 46. (𝐷4 − 2𝐷3 + 5𝐷2 − 8𝐷 + 4)𝑦 = 0
47. (4𝐷3 − 3𝐷 + 1)𝑦 = 0 48. (𝐷6 + 9𝐷4 + 24𝐷2 + 16)𝑦 = 0
49. (𝐷5 + 8𝐷3 + 16𝐷)𝑦 = 0 50. (𝐷2 − 𝐷 + 4)2 𝑦 = 0

For numbers 51 to 54, find the particular solution of the following differential equation.
51. (𝐷3 − 3𝐷 − 2)𝑦 = 0 when 𝑥 = 0, 𝑦 = 0, 𝑦 ′ = 9, 𝑦 ′′ = 0
2
52. (𝐷 − 𝐷 − 6)𝑦 = 0 when 𝑥 = 0, 𝑦 = 0 and when 𝑥 = 1, 𝑦 = 𝑒 3
53. (𝐷3 + 6𝐷2 + 12𝐷 + 8)𝑦 = 0 when 𝑥 = 0, 𝑦 = 1, 𝑦 ′ = −2, 𝑦 ′′ = 2
54. (𝐷3 + 7𝐷2 + 19𝐷 + 13)𝑦 = 0 when 𝑥 = 0, 𝑦 = 0, 𝑦 ′ = 2, 𝑦 ′′ = −12

This module is a property of Technological University of the Philippines Visayas and intended
for EDUCATIONAL PURPOSES ONLY and is NOT FOR SALE NOR FOR REPRODUCTION.
87

REFERENCES
Sergio E. Ymas Jr. Differential Equations (Simplified Approach). Sampaloc, Manila. Pablo.
L. Bustamante III Press, 1997. Chapter 5, 6.

Ricardo C. Asin. Elementary Differential Equations Reviewer. Manila, Philippines, Merriam


& Webster Bookstore, Inc., 1991.

This module is a property of Technological University of the Philippines Visayas and intended
for EDUCATIONAL PURPOSES ONLY and is NOT FOR SALE NOR FOR REPRODUCTION.
88

LEARNING GUIDE

Week No.: __10__

TOPIC/S

I. Non-Homogeneous Linear Equations


a. Construction of Homogeneous Linear Equations
b. Non-homogeneous Linear Equations
c. Solutions of Equation Having a Constant Right Member

EXPECTED COMPETENCIES
At the end of the week after studying the module, the student should be able to:
1. solve non-homogeneous linear equations;
2. know how to construct homogeneous linear equation satisfied by the given functions;
and
3. solve solutions of given differential equation having a constant right member.

CONTENT/TECHNICAL INFORMATION

NON-HOMOGENEOUS LINEAR EQUATIONS

Construction of Homogeneous Linear Equations


We recall the general homogeneous linear equation of the form

(𝑏0 𝐷𝑛 + 𝑏1 𝐷(𝑛−1) + ⋯ + 𝑏𝑛−1 𝐷 + 𝑏𝑛 )𝑦 = 0

or simply

𝑓(𝐷)𝑦 = 0

In the previous lesson, we were concerned with the solutions of the homogeneous
equations by first finding the roots of the auxiliary equations and, finally, the solution. Now,
intend to reverse the process, i.e. we are given the solution in terms of a function and we need
to find the corresponding homogeneous equation.
For example, if 𝑦 = 4𝑒 𝑥 + 5 − 2𝑥𝑒 −𝑥 , then for the term 4𝑒 𝑥 , we associate 𝑚 = 1, or
𝑚 − 1; for the constant term, we associate 𝑚 = 0, or simply 𝑚; for the term −2𝑥𝑒 −𝑥 , we
associate 𝑚 = −1, −1, or (𝑚 + 1)2. Thus, the auxiliary equation is

This module is a property of Technological University of the Philippines Visayas and intended
for EDUCATIONAL PURPOSES ONLY and is NOT FOR SALE NOR FOR REPRODUCTION.
89

𝑓(𝑚) = 𝑚(𝑚 − 1)(𝑚 + 1)2 = 0


Hence the desired homogeneous equation is
𝐷(𝐷 − 1)(𝐷 + 1)2 𝑦 = 0

Example 1. Obtain the homogeneous linear equation satisfied by the following given
functions:
a. 𝑦 = 2𝑥 2 − 4𝑒 −3𝑥
b. 𝑦 = 33𝑥 cos 2𝑥
c. 𝑦 = 𝑥 2 (𝑒 5𝑥 + 4)
d. 𝑦 = 𝑥(cos 2𝑥 + 𝑒 𝑥 )

Solution to Example 1.
a. For the term 2𝑥 2 , associate 𝑚 = 0, 0, and for −4𝑒 −3𝑥 , we have 𝑚 = −3. Then the
auxiliary equation is
𝑓(𝑚) = 𝑚2 (𝑚 + 3) = 0

The desired homogeneous equation is


𝐷2 (𝐷 + 3)𝑦 = 0.

b. For the term 𝑒 3𝑥 cos 2𝑥, associate 𝑚 = 3 ± 2𝑖. The corresponding auxiliary equation
is
𝑓(𝑚) = [𝑚 − (3 + 2𝑖)] [𝑚 − (3 − 2𝑖)] = 0
[(𝑚 − 3) − 2𝑖][(𝑚 − 3) + 2𝑖] = 0
(𝑚 − 3)2 − (2𝑖)2 = 0
𝑚2 − 6𝑚 + 9 + 4 = 0
𝑚2 − 6𝑚 + 13 = 0
Thus, the desired equation is
(𝐷2 − 6𝐷 + 13)𝑦 = 0

c. For the term 𝑥 2 𝑒 5𝑥 , associate 𝑚 = 5, 5, 5 and for that of 4𝑥 2 , 𝑚 = 0, 0, 0.


The auxiliary equation is

𝐷3 (𝐷 − 5)3 𝑦 = 0, or
(𝐷2 − 5𝐷)3 𝑦 = 0.

d. The roots of the auxiliary equation are imaginary because of the presence of cos 2𝑥.
Also, since there is 𝑥 the coefficient of cos 2𝑥, then the imaginary roots are repeated.
In the imaginary roots, 𝑎 = 0 and 𝑏 = 2. Thus, we have the auxiliary equation

This module is a property of Technological University of the Philippines Visayas and intended
for EDUCATIONAL PURPOSES ONLY and is NOT FOR SALE NOR FOR REPRODUCTION.
90

𝑓(𝑚) = [(𝑚 + 2𝑖)(𝑚 − 2𝑖)] = 0


(𝑚2 + 4)2 = 0

Thus, the differential equation is

(𝐷2 + 4)2 𝑦 = 0.

Nonhomogeneous Linear Equations


The non-homogeneous linear equation is of the form

(𝑏𝑜 𝐷𝑛 + 𝑏1 𝐷𝑛−1 + ⋯ + 𝑏𝑛−1 𝐷 + 𝑏𝑛 )𝑦 = 𝑅(𝑥)

The presence of 𝑅(𝑥) is what makes it distinct from the homogeneous linear equations. The
more compact form of the homogeneous equation is
𝑓(𝐷)𝑦 = 𝑅(𝑥)
If 𝑦𝑐 is the solution of the homogeneous equation 𝑓(𝐷)𝑦 = 0 and let 𝑦𝑝 be any particular
solution, then the general solution of
𝑓(𝐷)𝑦 = 𝑅(𝑥) (1)
is given by

𝑦 = 𝑦𝑐 + 𝑦𝑝

Various methods may apply for obtaining the solution of the equation (1). However, the most
useful and actual working technique is by the method called equating coefficients.

Example 2. Solve the equations


(𝐷2 − 3𝐷 + 2)𝑦 = 2𝑒 −𝑥 + 8
Solution to Example 2.
Solving the left side of the equation first, let
𝑓(𝑚) = 𝑚2 − 3𝑚 + 2 = 0
(𝑚 − 2)(𝑚 − 1) = 0
then,
𝑚 = 2; 𝑚=1
Thus,
𝒚𝒄 = 𝒄𝟏 𝒆𝟐𝒙 + 𝒄𝟐 𝒆𝒙

This module is a property of Technological University of the Philippines Visayas and intended
for EDUCATIONAL PURPOSES ONLY and is NOT FOR SALE NOR FOR REPRODUCTION.
91

Now, solving for the right side of the equation


If 𝑚 is a root of 𝑅(𝑥) = 0, then 𝑚 = 0 and −1. Let
𝑦𝑝 = 𝐴 + 𝐵𝑒 −𝑥
Then by differentiation:
𝐷𝑦𝑝 = −𝐵𝑒 −𝑥
𝐷2 𝑦𝑝 = 𝐵𝑒 −𝑥
By direct substitution of the above derivatives, we have
𝐷2 𝑦 − 3𝐷𝑦 + 2𝑦 = (𝐵𝑒 −𝑥 ) − 3(−𝐵𝑒 −𝑥 ) + 2(𝐴 + 𝐵𝑒 −𝑥 )

Equating the right side of the computation to the given 𝑅(𝑥) gives
4𝐵𝑒 −𝑥 + 2𝐴 + 2𝐵𝑒 −𝑥 = 2𝑒 −𝑥 + 8

By equating coefficients, we have:


1
𝑒 −𝑥 : 6𝐵 = 2, or 𝐵 =
3
Constant: 2𝐴 = 8, or 𝐴 = 4
Thus,
𝟏
𝒚𝒑 = 𝟒 + 𝒆−𝒙 .
𝟑
Therefore, the general solution is
𝑦 = 𝑦𝑐 + 𝑦𝑝
𝟏
𝒚 = 𝒄𝟏 𝒆𝟐𝒙 + 𝒄𝟐 𝒆𝒙 + 𝟒 + 𝒆−𝒙
𝟑

Example 3. Solve the equations


(𝐷2 − 3𝐷 + 2)𝑦 = 2𝑥 2 + 1

Solution to Example 3.
The left side of the equation is the same as with Example 2, the roots are 𝑚 = 2, and 1. Thus,
𝒚𝒄 = 𝒄𝟏 𝒆𝟐𝒙 + 𝒄𝟐 𝒆𝒙
Now, solving for the right side of equation

This module is a property of Technological University of the Philippines Visayas and intended
for EDUCATIONAL PURPOSES ONLY and is NOT FOR SALE NOR FOR REPRODUCTION.
92

If 𝑚 is the root of 𝑅(𝑥) = 0, then 𝑚 = 0, 0, 0, i.e. the root being repeated three times because
of the presence of 𝑥 2 . Let
𝑦𝑝 = 𝐴𝑒 0𝑥 + 𝐵𝑥𝑒 0𝑥 + 𝐶𝑥 2 𝑒 0𝑥 , or

𝑦𝑝 = 𝐴 + 𝐵𝑥 + 𝐶𝑥 2

Then by differentiation:
𝐷𝑦𝑝 = 𝐵 + 2𝐶𝑥

𝐷2 𝑦𝑝 = 2𝐶

By substitution in the given equation gives


𝐷2 𝑦 − 3𝐷𝑦 + 2𝑦 = (2𝐶) − 3(𝐵 + 2𝐶𝑥) + 2(𝐴 + 𝐵𝑥 + 𝐶𝑥 2 )
Equating the computation to the given 𝑅(𝑥), we have
2𝐶 − 3𝐵 − 6𝐶𝑥 + 2𝐴 + 2𝐵𝑥 + 2𝐶𝑥 2 = 2𝑥 2 + 1
By equating coefficients:
𝑥2: 2𝐶 = 2, or 𝐶 = 1
𝑥: −6𝐶 + 2𝐵 = 0, or 𝐵 = 3
Constants: 2𝐶 − 3𝐵 + 2𝐴 = 1, or 𝐴 = 4
Thus,
𝒚𝒑 = 𝟒 + 𝟑𝒙 + 𝒙𝟐

Therefore, the general solution is


𝑦 = 𝑦𝑐 + 𝑦𝑝

𝒚 = 𝒄𝟏 𝒆𝟐𝒙 + 𝒄𝟐 𝒆𝒙 + 𝟒 + 𝟑𝒙 + 𝒙𝟐

Solutions of Equations Having Constant Right Member


Consider a nonhomogeneous equation
(𝑏0 𝐷𝑛 + 𝑏1 𝐷𝑛−1 + ⋯ + 𝑏𝑛−1 𝐷 + 𝑏𝑛 )𝑦 = 𝑅0
where 𝑅0 is constant. Then the solution is given by
𝑅0
𝑦𝑝 = ,𝑏 ≠ 0
𝑏𝑛 𝑛

This module is a property of Technological University of the Philippines Visayas and intended
for EDUCATIONAL PURPOSES ONLY and is NOT FOR SALE NOR FOR REPRODUCTION.
93

For example, in
(𝐷2 + 4)𝑦 = 12
we have 𝑏𝑛 = 4 and 𝑅0 = 12. Thus, the particular solution is
𝑅0 12
𝑦𝑝 = = =3
𝑏𝑛 4

Example 4. Solve the equation


(𝐷2 − 𝐷 − 2)𝑦 = 30

Solution to Example 4.
The auxiliary equation is
𝑓(𝑚) = 𝑚2 − 𝑚 − 2 = (𝑚 − 2)(𝑚 + 1) = 0
whose roots are 𝑚 = −2, −1. Thus
𝑦𝑐 = 𝑐1 𝑒 −𝑥 + 𝑐2 𝑒 2𝑥
For the particular solution 𝑏𝑛 = −2 and 𝑅0 = 30. Then, we have
30
𝑦𝑝 = = −15
−2
Thus, the general solution is
𝑦 = 𝑦𝑐 + 𝑦𝑝

𝒚 = 𝒄𝟏 𝒆−𝒙 + 𝒄𝟐 𝒆𝟐𝒙 − 𝟏𝟓

Derivation of the Formula for 𝒚𝒑


𝑅0
𝑦𝑝 = , 𝑤ℎ𝑒𝑟𝑒 𝑏𝑛 ≠ 0
𝑏𝑛
By differentiation, all derivatives are zero, i.e.
𝐷𝑦 = 𝐷2 𝑦 = ⋯ = 𝐷𝑘 𝑦 = 0
By substituting all derivatives as zero in
(𝑏0 𝐷𝑛 + 𝑏1 𝐷𝑛−1 + ⋯ + 𝑏𝑛−1 𝐷 + 𝑏𝑛 )𝑦 = 𝑅0
Simplifying the left side, we have 𝑏𝑛 𝑦 = 𝑅0 . Thus
𝑅0
𝑦𝑝 =
𝑏𝑛

This module is a property of Technological University of the Philippines Visayas and intended
for EDUCATIONAL PURPOSES ONLY and is NOT FOR SALE NOR FOR REPRODUCTION.
94

Suppose 𝑏𝑛 = 0. This means that there is the lowest-ordered derivative present in the
differential equation written as
(𝑏0 𝐷𝑛 + ⋯ + 𝑏𝑘 𝐷𝑘 )𝑦 = 𝑅0
Then the solution is given by
𝑅0 𝑥 𝑘
𝑦𝑝 =
𝑘! 𝑏𝑘

For example, the equation (𝐷4 + 2𝐷3 )𝑦 = 8 has the solution


8𝑥 3 8𝑥 3 2𝑥 3
𝑦𝑝 = = =
3! 2 12 3

Example 5. Solve the equation


𝑑 4 𝑦 6𝑑 3 𝑦
− = 12
𝑑𝑥 4 𝑑𝑥 3
Solution to Example 5.
Solving for 𝑦𝑐 , we have
(𝐷4 − 6𝐷3 )𝑦 = 0
𝑚4 − 6𝑚3 = 0
𝑚3 (𝑚 − 6) = 0
Therefore, roots are 𝑚 = 0, 0, 0 and 6, thus
𝑦𝑐 = 𝑐1 𝑒 6𝑥 + 𝑐2 + 𝑐3 𝑥 + 𝑐4 𝑥 2
Solving for 𝑦𝑝 :

Since there is constant term in 𝑓(𝐷)𝑦 = 0, then the solution with 𝑘 = 3 (lowest-ordered
derivative) is
𝑅0 𝑥 𝑘 12 𝑥 3 𝑥3
𝑦𝑝 = = =−
𝑘! 𝑏𝑘 3! (−6) 3
Therefore, general solution is
𝑦 = 𝑦𝑐 + 𝑦𝑝

𝒙𝟑
𝒚 = 𝒄𝟏 𝒆𝟔𝒙 + 𝒄𝟐 + 𝒄𝟑 𝒙 + 𝒄𝟒 𝒙𝟐 −
𝟑

This module is a property of Technological University of the Philippines Visayas and intended
for EDUCATIONAL PURPOSES ONLY and is NOT FOR SALE NOR FOR REPRODUCTION.
95

PROGRESS CHECK
Activity Sheets
EXERCISE 7
Review on Nonhomogeneous Equations

Name: _____________________________________________Year & Section: __________


Subject: ____________________________________________Date Submitted: __________
Professor: __________________________________________Score: __________________

Construction of Homogeneous Linear Equations


Obtain the differential equations satisfied by the given particular functions.
1. 𝑦 = 𝑒 5𝑥 − 3𝑥 − 𝑥 2 6. 𝑦 = 3𝑥 sin 2𝑥

2. 𝑦 = 5 − 2𝑥 + 𝑒 4𝑥 7. 𝑦 = 𝑒 2𝑥 cos 5𝑥

3. 𝑦 = 𝑥 2 − 2𝑒 3𝑥 + 5𝑒 −3𝑥 8. 𝑦 = 𝑥 cos 2𝑥 − sin 𝑥

4. 𝑦 = 𝑒 𝑥 cos 3𝑥 9. 𝑦 = 𝑒 4𝑥 (2 sin 3𝑥 + 4 cos 3𝑥)

5. 𝑦 = 3 sin 2𝑥 10. 𝑦 = 5𝑥 2 − 2𝑥 + 3𝑒 −𝑥 (𝑥 + sin 𝑥)

Nonhomogeneous Linear Equations


Solve the following equations:
11. 𝑦 ′′ − 2𝑦 ′ + 𝑦 = 𝑥 2 − 1 16. 𝑦 ′′′ − 3𝑦 ′′ − 3𝑦 ′ − 𝑦 = 𝑒 𝑥 + 1

12. (𝐷2 − 5𝐷 + 4)𝑦 = 2𝑥 2 − 6𝑥 − 5 17. 𝑦 ′′ + 6𝑦 ′ + 13𝑦 = 60 cos 𝑥 + 26

13. (4𝐷2 + 4𝐷 + 1)𝑦 = 18𝑒 𝑥 − 5 18. 𝑦 ′′ + 4𝑦 ′ + 3𝑦 = 15𝑒 2𝑥 + 𝑒 −𝑥

14. 𝑦 ′ − 5𝑦 = 𝑥 2 𝑒 𝑥 − 𝑥𝑒 5𝑥 19. (𝐷2 − 1)𝑦 = 8𝑥𝑒 𝑥

15. (𝐷2 − 3𝐷 − 2)𝑦 = 100 sin 2𝑥 20. (𝐷3 − 𝐷2 + 𝐷 − 1)𝑦 = 4 sin 𝑥

This module is a property of Technological University of the Philippines Visayas and intended
for EDUCATIONAL PURPOSES ONLY and is NOT FOR SALE NOR FOR REPRODUCTION.
96

21. (𝐷2 + 1)𝑦 = 12 26. (𝐷2 − 𝐷)𝑦 = 6

22. (𝐷2 + 16)𝑦 = 36 27. (𝐷6 + 9𝐷4 )𝑦 = 15

23. (𝐷2 − 2𝐷 − 3)𝑦 = 6 28. (𝐷4 − 16𝐷2 )𝑦 = 6

24. (𝐷5 − 4𝐷3 )𝑦 = 20 29. (𝐷5 + 7𝐷4 )𝑦 = 26

25. (𝐷2 − 5𝐷 + 4)𝑦 = 28 30. (𝐷2 − 16)𝑦 = 12

This module is a property of Technological University of the Philippines Visayas and intended
for EDUCATIONAL PURPOSES ONLY and is NOT FOR SALE NOR FOR REPRODUCTION.
97

REFERENCES
Sergio E. Ymas Jr. Differential Equations (Simplified Approach). Sampaloc, Manila. Pablo.
L. Bustamante III Press, 1997. Chapter 7.

Ricardo C. Asin. Elementary Differential Equations Reviewer. Manila, Philippines, Merriam


& Webster Bookstore, Inc., 1991.

This module is a property of Technological University of the Philippines Visayas and intended
for EDUCATIONAL PURPOSES ONLY and is NOT FOR SALE NOR FOR REPRODUCTION.
98

LEARNING GUIDE

Week No.: __11__

TOPIC/S

I. Order Reduction and Variation of Parameters


II. Inverse Differential Operators
a. Exponential Shift

EXPECTED COMPETENCIES
At the end of the week after studying the module, the student should be able to:
1. solve differential equations using different methods:
a. the reduction of order;
b. the variation of parameters; and
c. exponential shift.

CONTENT/TECHNICAL INFORMATION

ORDER REDUCTION AND VARIATION OF PARAMETERS

This lesson presents two methods for solving differential equations – the reducing of
order and the variation of parameters.

Reduction of Order

The second order of linear equation may be represented by


𝑦 ′′ + 𝑃𝑦 ′ + 𝑄𝑦 = 𝑅 (1)
Let 𝑦 = 𝑦1 be a solution of the homogeneous equation, i.e.

𝑦1′′ + 𝑃𝑦1′ + 𝑄𝑦1 = 0

Let 𝑦 = 𝑣𝑦1, then by differentiation gives


𝑦 ′ = 𝑣𝑦1′ + 𝑦1 𝑣 ′

𝑦 ′′ = 𝑣𝑦1′′ + 𝑦1′ 𝑣 ′ + 𝑦1 𝑣 ′′ + 𝑣′𝑦1 ′


= 𝑣𝑦1′′ + 2𝑣 ′ 𝑦1 ′ + 𝑦1 𝑣 ′′

Substituting the above equation in the given equation (1) gives


𝑦 ′′ + 𝑃𝑦 ′ + 𝑄𝑦 = 𝑅
(𝑣𝑦1′′ + 2𝑣 ′ 𝑦1′ + 𝑦1 𝑣 ′′ ) + 𝑃(𝑣𝑦1′ + 𝑦1 𝑣 ′ ) + 𝑄(𝑣𝑦1 ) = 𝑅
𝑦1 𝑣 ′′ + 2𝑦1′ 𝑣 ′ + 𝑃𝑦1 𝑣 ′ + 𝑣(𝑦1′′ + 𝑃𝑦1′ + 𝑄𝑦1 ) = 𝑅

This module is a property of Technological University of the Philippines Visayas and intended
for EDUCATIONAL PURPOSES ONLY and is NOT FOR SALE NOR FOR REPRODUCTION.
99

𝑦1 𝑣 ′′ + 2𝑦1′ 𝑣 ′ + 𝑃𝑦1 𝑣 ′ + 𝑣(0) = 𝑅


𝑦1 𝑣 ′′ + 2𝑦1′ 𝑣 ′ + 𝑃𝑦1 𝑣 ′ = 𝑅
Let 𝑧 = 𝑣 ′ , then
𝑦1 𝑧 + (2𝑦1′ + 𝑃𝑦1 )𝑧 = 𝑅
which is linear in 𝑧.

Example 1. Solve the equation 𝑦 ′′ − 𝑦 = 𝑒 2𝑥

Solution to Example 1.
The roots of auxiliary equation 𝑓(𝑚) = 𝑚2 − 1 = 0 are 𝑚 = 1, −1. Thus, we have the
complementary solution as
𝑦𝑐 = 𝑐1 𝑒 𝑥 + 𝑐2 𝑒 −𝑥
For the transformation, let
𝑦 = 𝑣𝑒 𝑥
𝑦 ′ = 𝑣𝑒 𝑥 + 𝑣 ′ 𝑒 𝑥
𝑦 ′′ = 𝑣𝑒 𝑥 + 𝑣 ′ 𝑒 𝑥 + 𝑣 ′ 𝑒 𝑥 + 𝑣 ′′ 𝑒 𝑥
= 𝑣 ′′ 𝑒 𝑥 + 2𝑣 ′ 𝑒 𝑥 + 𝑣𝑒 𝑥
Substituting in the given equation gives
(𝑣 ′′ 𝑒 𝑥 + 2𝑣 ′ 𝑒 𝑥 + 𝑣𝑒 𝑥 ) − 𝑣𝑒 𝑥 = 𝑒 2𝑥
𝑣 ′′ 𝑒 𝑥 + 2𝑣 ′ 𝑒 𝑥 = 𝑒 2𝑥
𝑣 ′′ + 2𝑣 ′ = 𝑒 𝑥
Let 𝑧 = 𝑣 ′ , then
𝑧 ′ + 2𝑧 = 𝑒 𝑥
Which is now linear in 𝑧. Then the integrating factor is
𝐼 = 𝑒 ∫ 2𝑑𝑥 = 𝑒 2𝑥
Multiplying both sides of the linear equation in 𝑧 by 𝑒 2𝑥 , we have:
𝑒 2𝑥 𝑧 ′ + 2𝑧 𝑒 2𝑥 = 𝑒 3𝑥
𝑒 2𝑥 𝑑𝑧 + 2𝑧𝑒 2𝑥 𝑑𝑥 = 𝑒 3𝑥 𝑑𝑥
𝑑(𝑧𝑒 2𝑥 ) = 𝑒 3𝑥 𝑑𝑥
Integrating:
1 3𝑥
𝑧𝑒 2𝑥 = 𝑒 +𝑐
3
Substitute 𝑧 = 𝑣 ′ :
1 3𝑥
𝑣 ′ 𝑒 2𝑥 = 𝑒 +𝑐
3

This module is a property of Technological University of the Philippines Visayas and intended
for EDUCATIONAL PURPOSES ONLY and is NOT FOR SALE NOR FOR REPRODUCTION.
100

Multiply by 𝑒 −2𝑥 :
1 𝑥
𝑣′ = 𝑒 + 𝑒 −2𝑥 𝑐
3
Integrating:
1
𝑣 = 𝑒 𝑥 + 𝑐𝑒 −2𝑥 + 𝑐1
3
𝑦
since 𝑣 = 𝑒 𝑥, so
𝑦 1 𝑥
𝑥
= 𝑒 + 𝑐𝑒 −2𝑥 + 𝑐1
𝑒 3
𝟏
𝒚 = 𝒆𝟐𝒙 + 𝒄𝒆−𝒙 + 𝒄𝟏 𝒆𝒙
𝟑

Alternative Solution
𝐿𝑒𝑡 𝑦 = 𝑣𝑒 −𝑥
𝑦 ′ = 𝑣 ′ 𝑒 −𝑥 − 𝑣𝑒 −𝑥
𝑦 ′′ = 𝑣 ′′ 𝑒 −𝑥 − 𝑣 ′ 𝑒 −𝑥 + 𝑣𝑒 −𝑥 − 𝑣 ′ 𝑒 −𝑥
= 𝑣 ′′ 𝑒 −𝑥 − 2𝑣 ′ 𝑒 −𝑥 + 𝑣𝑒 −𝑥
Substituting in the given equation gives the following computations:
𝑦 ′′ − 𝑦 = 𝑒 2𝑥
(𝑣 ′′ 𝑒 −𝑥 − 2𝑣 ′ 𝑒 −𝑥 + 𝑣𝑒 −𝑥 ) − 𝑣𝑒 −𝑥 = 𝑒 2𝑥
𝑣 ′′ 𝑒 −𝑥 − 2𝑣 ′ 𝑒 −𝑥 = 𝑒 2𝑥
𝑣 ′′ − 2𝑣 ′ = 𝑒 3𝑥

Let 𝑧 = 𝑣 , then
𝑧 ′ − 2𝑧 = 𝑒 3𝑥
which is now linear in 𝑧. Then the integrating factor is
𝐼 = 𝑒 ∫ −2𝑑𝑥 = 𝑒 −2𝑥
Multiplying the linear equation by 𝑒 −2𝑥 , we have
𝑒 −2𝑥 𝑧 ′ − 2𝑧𝑒 −2𝑥 = 𝑒 𝑥
𝑒 −2𝑥 𝑑𝑧 − 2𝑧𝑒 −2𝑥 𝑑𝑥 = 𝑒 𝑥 𝑑𝑥

∫ 𝑑(𝑧𝑒 −2𝑥 ) = ∫ 𝑒 𝑥 𝑑𝑥

𝑧𝑒 −2𝑥 = 𝑒 𝑥 + 𝑐
𝑣 ′ = 𝑒 3𝑥 + 𝑐𝑒 2𝑥
1 3𝑥
𝑣= 𝑒 + 𝑐𝑒 2𝑥 + 𝑐1
3
𝟏
𝒚 = 𝒆𝟐𝒙 + 𝒄𝒆𝒙 + 𝒄𝟏 𝒆−𝒙
𝟑

This module is a property of Technological University of the Philippines Visayas and intended
for EDUCATIONAL PURPOSES ONLY and is NOT FOR SALE NOR FOR REPRODUCTION.
101

Example 2. Solve the equation


(𝐷2 − 5𝐷 + 6)𝑦 = 2𝑒 𝑥
Solution to Example 2.
The roots of the auxiliary equation 𝑓(𝑚) = 𝑚2 − 5𝑚 + 6 = 0 are 𝑚 = 2, 3. Thus,
𝑦𝑐 = 𝑐1 𝑒 2𝑥 + 𝑐2 𝑒 3𝑥
Let
𝑦 = 𝑣𝑒 2𝑥
𝑦 ′ = 𝑣 ′ 𝑒 2𝑥 + 2𝑣𝑒 2𝑥
𝑦 ′′ = 𝑣 ′′ 𝑒 2𝑥 + 2𝑣 ′ 𝑒 2𝑥 + 4𝑣𝑒 2𝑥 + 2𝑣 ′ 𝑒 2𝑥
= 𝑣 ′′ 𝑒 2𝑥 + 4𝑣 ′ 𝑒 2𝑥 + 4𝑣𝑒 2𝑥
Substituting the above derivatives, we have:
𝐷2 𝑦 − 5𝐷𝑦 + 6𝑦 = 2𝑒 𝑥
(𝑣 ′′ 𝑒 2𝑥 + 4𝑣 ′ 𝑒 2𝑥 + 4𝑣𝑒 2𝑥 ) − 5(𝑣 ′ 𝑒 2𝑥 + 2𝑣𝑒 2𝑥 ) + 6(𝑣𝑒 2𝑥 ) = 2𝑒 𝑥
𝑣 ′′ 𝑒 2𝑥 − 𝑣 ′ 𝑒 2𝑥 = 2𝑒 𝑥
𝑣 ′′ − 𝑣 ′ = 2𝑒 −𝑥
Let 𝑧 = 𝑣 ′ , then
𝑧 ′ − 𝑧 = 2𝑒 −𝑥
which is linear in 𝑧. Then the integrating factor is
𝐼 = 𝑒 ∫ −𝑑𝑥 = 𝑒 −𝑥
Multiplying the linear equation in 𝑧 by 𝑒 −𝑥 , we get
𝑒 −𝑥 𝑧 ′ − 𝑧𝑒 −𝑥 = 2𝑒 −𝑥
𝑒 −𝑥 𝑑𝑧 − 𝑧𝑒 −𝑥 𝑑𝑥 = 2𝑒 −2𝑥 𝑑𝑥

∫ 𝑑(𝑒 −𝑥 𝑧) = ∫ 2𝑒 −2𝑥 𝑑𝑥

𝑧𝑒 −𝑥 = −𝑒 −2𝑥 + 𝑐
𝑧 = −𝑒 −𝑥 + 𝑐𝑒 𝑥
𝑣 ′ = −𝑒 −𝑥 + 𝑐𝑒 𝑥
𝑣 = 𝑒 −𝑥 + 𝑐𝑒 𝑥 + 𝑐1
𝑦
2𝑥
= 𝑒 −𝑥 + 𝑐𝑒 𝑥 + 𝑐1
𝑒
𝒚 = 𝒆𝒙 + 𝒄𝒆𝟑𝒙 + 𝒄𝟏 𝒆𝟐𝒙

This module is a property of Technological University of the Philippines Visayas and intended
for EDUCATIONAL PURPOSES ONLY and is NOT FOR SALE NOR FOR REPRODUCTION.
102

Variation of Parameters
Consider the general second-order linear equation
𝑦 ′′ + 𝑃𝑦 ′ + 𝑄𝑦 = 𝑅
Previously, we developed a method for obtaining the solution of the above equation using
one part of the solution 𝑦𝑐 pertaining to one root of the auxiliary equation. Now in this
section, we shall employ two roots of the auxiliary equation.
Suppose the general solution of the homogeneous equation is
𝑦𝑐 = 𝑐1 𝑦1 + 𝑐2 𝑦2
Based on the form of 𝑦1 and 𝑦2 , let
𝑦 = 𝐴𝑦1 + 𝐵𝑦2
Then by differentiation,
𝑦 ′ = 𝐴𝑦1 ′ + 𝐵𝑦2′ + 𝐴′ 𝑦1 + 𝐵 ′ 𝑦2 = 0
Let
𝐴′ 𝑦1 + 𝐵 ′ 𝑦2 = 0
It follows that 𝑦 ′ in terms of 𝑦1 ′ and 𝑦2 ′ is
𝑦 ′ = 𝐴𝑦1′ + 𝐵𝑦2 ′
By differentiation
𝑦 ′′ = 𝐴𝑦1 ′′ + 𝐵𝑦2′′ + 𝐴′ 𝑦1 ′ + 𝐵 ′ 𝑦2 ′

By substitution, we have the following computations:


𝑦 ′′ + 𝑃𝑦 ′ + 𝑄𝑦 = 𝑅
(𝐴𝑦1 ′′ + 𝐵𝑦2′′ + 𝐴′ 𝑦1′ + 𝐵 ′ 𝑦2′ ) + 𝑃(𝐴𝑦1′ + 𝐵𝑦2′ ) + 𝑄(𝐴𝑦1 + 𝐵𝑦2 ) = 0

By arranging the terms, we get


𝐴(𝑦1′′ + 𝑃𝑦1′ + 𝑄𝑦1 ) + 𝐵(𝑦2′′ + 𝑃𝑦2′ + 𝑄𝑦2 ) + 𝐴′𝑦1′ + 𝐵 ′𝑦2′ = 𝑅(𝑥) (1)

But 𝑦1 and 𝑦2 are solutions of the homogeneous equation, i.e.


𝑦1′′ + 𝑃𝑦1′ + 𝑄𝑦1 = 0, and 𝑦2′′ + 𝑃𝑦2′ + 𝑄𝑦2 = 0

Thus, we find the equation in (1) only in terms of 𝐴′ and 𝐵 ′ as follows:


𝐴′ 𝑦1′ + 𝐵 ′ 𝑦2′ = 0
𝐴′ 𝑦1′ + 𝐵 ′ 𝑦2′ = 𝑅(𝑥)

This module is a property of Technological University of the Philippines Visayas and intended
for EDUCATIONAL PURPOSES ONLY and is NOT FOR SALE NOR FOR REPRODUCTION.
103

Example 3. Solve the equation


2
(𝐷2 − 1)𝑦 =
1 + 𝑒𝑥
Solution to Example 3.
The auxiliary equation of 𝑓(𝑚) = 𝑚2 − 1 = 0 has the roots −1 and 1 have the solution
𝑦𝑐 = 𝑐1 𝑒 −𝑥 + 𝑐2 𝑒 𝑥
Based on the roots of the auxiliary equation, take 𝑦1 and 𝑦2 as 𝑒 −𝑥 and 𝑒 𝑥 such that
𝑦 = 𝐴𝑒 −𝑥 + 𝐵𝑒 𝑥
𝑦 ′ = 𝐴′ 𝑒 −𝑥 + 𝐵 ′ 𝑒 𝑥 − 𝐴𝑒 −𝑥 + 𝐵𝑒 𝑥
Let 𝐴′ 𝑒 −𝑥 + 𝐵 ′ 𝑒 𝑥 = 0, and so we have
𝑦 ′ = −𝐴𝑒 −𝑥 + 𝐵𝑒 𝑥
𝑦 ′′ = −𝐴′ 𝑒 −𝑥 + 𝐵 ′ 𝑒 𝑥 + 𝐴𝑒 −𝑥 + 𝐵𝑒 𝑥
By substitution, we have:
2
𝐷2 𝑦 − 𝑦 =
1 + 𝑒𝑥
2
(−𝐴′ 𝑒 −𝑥 + 𝐵 ′ 𝑒 𝑥 + 𝐴𝑒 −𝑥 + 𝐵𝑒 𝑥 ) − (𝐴𝑒 −𝑥 + 𝐵𝑒 𝑥 ) =
1 + 𝑒𝑥
Simplifying the above equation, what is left will be in terms of 𝐴′ and 𝐵 ′ only, i.e.
2
−𝐴′ 𝑒 −𝑥 + 𝐵 ′ 𝑒 𝑥 =
1 + 𝑒𝑥
Solve the equations in terms of 𝐴′ and 𝐵 ′ simultaneously.
2
−𝐴′ 𝑒 −𝑥 + 𝐵 ′ 𝑒 𝑥 =
1 + 𝑒𝑥
𝐴′ 𝑒 −𝑥 + 𝐵 ′ 𝑒 𝑥 = 0
By addition, we have

′ 𝑥
2 𝑒 −𝑥
2𝐵 𝑒 = , 𝑜𝑟 𝐵′ =
1 + 𝑒𝑥 1 + 𝑒𝑥

𝑒 −2𝑥
𝐵 = −𝑥
𝑒 +1
By division we can rewrite it in the form

′ −𝑥
𝑒 −𝑥
𝐵 =𝑒 − −𝑥
𝑒 +1
By integration, we have
𝐵 = −𝑒 −𝑥 + ln(𝑒 −𝑥 + 1)

This module is a property of Technological University of the Philippines Visayas and intended
for EDUCATIONAL PURPOSES ONLY and is NOT FOR SALE NOR FOR REPRODUCTION.
104

Substituting 𝐵 ′ in 𝐴′ 𝑒 −𝑥 + 𝐵 ′ 𝑒 𝑥 = 0, we find

′ −𝑥
𝑒 −𝑥 𝑒𝑥
𝐴𝑒 +( ) 𝑒 𝑥 = 0, 𝑜𝑟 𝐴= −
1 + 𝑒𝑥 1 + 𝑒𝑥
By integration, we have
𝐴 = − ln(1 + 𝑒 𝑥 )
The particular solution 𝑦𝑝 is
𝑦 = 𝐴𝑒 −𝑥 + 𝐵𝑒 𝑥
𝑦𝑝 = −𝑒 −𝑥 ln(1 + 𝑒 𝑥 ) + 𝑒 𝑥 [−𝑒 −𝑥 + ln(𝑒 −𝑥 + 1)]
𝑦𝑝 = 1 + (𝑒 𝑥 − 𝑒 −𝑥 ) ln(1 + 𝑒 𝑥 )
Thus, the general solution is
𝑦 = 𝑦𝑐 + 𝑦𝑝
𝒚 = 𝒄𝟏 𝒆−𝒙 + 𝒄𝟐 𝒆𝒙 + 𝟏 + (𝒆𝒙 − 𝒆−𝒙 ) 𝐥𝐧(𝟏 + 𝒆𝒙 )

INVERSE DIFFERENTIAL OPERATORS


In module lesson Week 7, we discussed the differential operator 𝐷 ass well as its properties.
A theorem on exponential shift which was so basic in solving differential equations will be
taken up again in this lesson.

Exponential Shift
In this lesson we utilize the exponential shift as an alternative method in solving
nonhomogeneous equations. Thus, it is important to recall the theorem on exponential shift
stated as
𝑒 𝑎𝑥 𝑓(𝐷)𝑦 = 𝑓(𝐷 − 𝑎)(𝑒 𝑎𝑥 𝑦)
where 𝑓(𝐷) is a linear differential operator.

Example 4. Solve the operators


(𝐷2 − 4𝐷 + 4)𝑦 = 12𝑥𝑒 2𝑥
Solution to Example 4.
(𝐷2 − 4𝐷 + 4)𝑦 = 12𝑥𝑒 2𝑥
(𝐷 − 2)2 𝑦 = 12𝑥𝑒 2𝑥
𝑒 −2𝑥 (𝐷 − 2)2 𝑦 = 12𝑥
By exponential shift,
𝐷2 (𝑒 −2𝑥 𝑦) = 12𝑥

This module is a property of Technological University of the Philippines Visayas and intended
for EDUCATIONAL PURPOSES ONLY and is NOT FOR SALE NOR FOR REPRODUCTION.
105

𝐷(𝑒 −2𝑥 𝑦) = 6𝑥 2
𝑒 −2𝑥 𝑦 = 2𝑥 3
Thus, the particular solution 𝑦𝑝 is
𝑦𝑝 = 2𝑥 3 𝑒 2𝑥

Now, since the roots of auxiliary equation 𝑓(𝑚) = 𝑚2 − 4𝑚 + 4 = 0 is 𝑚 = 2, and 2, then


𝑦𝑐 = 𝑐1 𝑒 2𝑥 + 𝑐2 𝑥𝑒 2𝑥
Thus, the solution is
𝑦 = 𝑦𝑐 + 𝑦𝑝
𝒚 = 𝒄𝟏 𝒆𝟐𝒙 + 𝒄𝟐 𝒙𝒆𝟐𝒙 + 𝟐𝒙𝟑 𝒆𝟐𝒙

Example 5. Solve the equation


(𝐷2 − 4𝐷 + 4)𝑦 = 12𝑥𝑒 2𝑥 + 24𝑥 + 6
Solution to Example 5.
Because of 𝑒 2𝑥 and 24𝑥 + 6, being different functions, it would be more convenient to split
the solution as follows:
(𝐷2 − 4𝐷 + 4)𝑦1 = 12𝑥𝑒 2𝑥
(𝐷2 − 4𝐷 + 4)𝑦2 = 24𝑥 + 6
But the first problem (𝐷 2 − 4𝐷 + 4)𝑦 = 12𝑥𝑒 2𝑥 was already done in the preceding
problem, i.e. 𝑦1 = 2𝑥 3 𝑒 2𝑥 . We need only to solve
(𝐷2 − 4𝐷 + 4)𝑦2 = 24𝑥 + 6
The form of the solution is
𝑦2 = 𝐴𝑥 + 𝐵
𝐷𝑦2 = 𝐴
𝐷2 𝑦2 = 0
By substitution:
𝐷2 𝑦 − 4𝐷𝑦 + 4𝑦 = 24𝑥 + 6
−4(𝐴) + 4(𝐴𝑥 + 𝐵) = 24𝑥 + 6
15
By equating coefficients; 4𝐴 = 24, or 𝐴 = 6 and −4𝐴 + 4𝐵 = 6, or 𝐵 = 2
15
Thus, 𝑦2 = 6𝑥 + and the general solution is
2

𝑦 = 𝑦𝑐 + 𝑦1 + 𝑦2
𝟏𝟓
𝒚 = 𝒄𝟏 𝒆𝟐𝒙 + 𝒄𝟐 𝒙𝒆𝟐𝒙 + 𝟐𝒙𝟑 𝒆𝟐𝒙 + 𝟔𝒙 +
𝟐

This module is a property of Technological University of the Philippines Visayas and intended
for EDUCATIONAL PURPOSES ONLY and is NOT FOR SALE NOR FOR REPRODUCTION.
106

Example 6. Solve the equation


(𝐷2 − 2𝐷 + 2)𝑦 = 12𝑥 3 𝑒 2𝑥
Solution to Example 6.
The roots of auxiliary equation
𝑓(𝑚) = 𝑚2 − 2𝑚 + 2 = 0 are 𝑚 = 1 ± 𝑖. Thus, 𝑦𝑐 = 𝑒 𝑥 (𝑐1 cos 𝑥 + 𝑐2 sin 𝑥)

Base on the form of 𝑅(𝑥), the right side of the differential equation,

𝑦 = 𝐴𝑒 2𝑥 + 𝐵𝑥𝑒 2𝑥 + 𝐶𝑥 2 𝑒 2𝑥 + 𝐷𝑥 3 𝑒 2𝑥

The differentiation and substituting of derivatives in the given equation is a long process.
However, the method on exponential shift will make the computation simpler, i.e.

𝑒 −2𝑥 (𝐷2 − 2𝐷 + 2)𝑦 = 12𝑥 3

with 𝑎 = −2, we then replace 𝐷 by 𝐷 + 𝑎, or 𝐷 + 2, i.e.

[(𝐷 + 2)2 − 2(𝐷 + 2) + 2] (𝑒 −2𝑥 𝑦) = 12𝑥 3

(𝐷2 + 2𝐷 + 2)(𝑒 −2𝑥 𝑦) = 12𝑥 3 (1)

By differentiating 𝑦𝑝 , we have the following computations:

𝑒 −2𝑥 𝑦 = 𝐴 + 𝐵𝑥 + 𝐶𝑥 2 + 𝐷𝑥 3 (2)

𝐷(𝑒 −2𝑥 𝑦) = 𝐵 + 2𝐶𝑥 + 3𝐷𝑥 2

𝐷2 (𝑒 −2𝑥 𝑦) = 2𝐶 + 6𝐷𝑥

Substituting those derivatives in (1), we have

(2𝐶 + 6𝐷𝑥) + 2(𝐵 + 2𝐶𝑥 + 3𝐷𝑥 2 ) + 2(𝐴 + 𝐵𝑥 + 𝐶𝑥 2 + 𝐷𝑥 3 ) = 12𝑥 3

By equating coefficients:

𝑥 3 : 2𝐷 = 12, 𝑜𝑟 𝐷 = 6

𝑥2: 6𝐷 + 2𝐶 = 0, 𝑜𝑟 𝐶 = −18

𝑥: 6𝐷 + 4𝐶 + 2𝐵 = 0, 𝑜𝑟 𝐵 = 18

𝑥 0 : 2𝐶 + 2𝐵 + 2𝐴 = 0, 𝑜𝑟 𝐴 = 0

Substituting the values in (2), we have

𝑒 −2𝑥 𝑦𝑝 = 18𝑥 − 18𝑥 2 + 6𝑥 3

Thus, the general solution is

𝑦 = 𝑦𝑐 + 𝑦𝑝

𝒚 = 𝒆𝒙 (𝒄𝟏 𝐜𝐨𝐬 𝒙 + 𝒄𝟐 𝐬𝐢𝐧 𝒙) + 𝒆𝟐𝒙 (𝟏𝟖𝒙 − 𝟏𝟖𝒙𝟐 + 𝟔𝒙𝟑 )

This module is a property of Technological University of the Philippines Visayas and intended
for EDUCATIONAL PURPOSES ONLY and is NOT FOR SALE NOR FOR REPRODUCTION.
107

PROGRESS CHECK
Activity Sheets
EXERCISE 8
Review on Order Reduction, Variation Parameters, and Exponential Shift

Name: _____________________________________________Year & Section: __________


Subject: ____________________________________________Date Submitted: __________
Professor: __________________________________________Score: __________________

Solve the following equations.


Order Reduction
1. (𝐷2 − 2𝐷 + 1)𝑦 = 𝑒 𝑥

2. (𝐷2 − 5𝐷 + 6)𝑦 = 2𝑒 𝑥

3. (𝐷2 − 1)𝑦 = 𝑥 − 2

4. (𝐷2 − 4)𝑦 = 𝑥 − 3

5. (𝐷2 + 2𝐷 + 1)𝑦 = (𝑒 𝑥 + 1)−2

Variation of Parameters
1
6. (𝐷2 − 3𝐷 + 2) =
1+𝑒 −𝑥

7. (𝐷2 − 1)𝑦 = 𝑒 𝑥 + 1

8. 𝑦 ′′ − 𝑦 ′ − 2𝑦 = 𝑒 3𝑥

9. (𝐷2 − 2𝐷 + 1)𝑦 = 𝑒 2𝑥 (𝑒 𝑥 + 1)−2

2
10. (𝐷2 − 1)𝑦 =
𝑒 𝑥 −𝑒 −𝑥

Exponential Shift
11. (𝐷 − 3)2 𝑦 = 2𝑒 3𝑥

12. (𝐷 + 1)2 𝑦 = 6𝑥𝑒 −𝑥

This module is a property of Technological University of the Philippines Visayas and intended
for EDUCATIONAL PURPOSES ONLY and is NOT FOR SALE NOR FOR REPRODUCTION.
108

13. (𝐷 − 2)3 𝑦 = 18𝑥𝑒 2𝑥

14. 𝐷2 (𝐷 − 2)2 𝑦 = 24𝑒 2𝑥

15. (𝐷2 − 𝐷 − 2)𝑦 = 72𝑥𝑒 2𝑥

16. 𝐷2 (𝐷 − 3)2 𝑦 = −12𝑒 −3𝑥

This module is a property of Technological University of the Philippines Visayas and intended
for EDUCATIONAL PURPOSES ONLY and is NOT FOR SALE NOR FOR REPRODUCTION.
109

REFERENCES
Sergio E. Ymas Jr. Differential Equations (Simplified Approach). Sampaloc, Manila. Pablo.
L. Bustamante III Press, 1997. Chapter 8 & 9.

Ricardo C. Asin. Elementary Differential Equations Reviewer. Manila, Philippines, Merriam


& Webster Bookstore, Inc., 1991.

This module is a property of Technological University of the Philippines Visayas and intended
for EDUCATIONAL PURPOSES ONLY and is NOT FOR SALE NOR FOR REPRODUCTION.
110

LEARNING GUIDE

Week No.: __12__

TOPIC/S

I. Applications of 2nd Order Differential Equations

EXPECTED COMPETENCIES
At the end of the week after studying the module, the student should be able to:
1. solve a second-order differential equation representing simple harmonic motion;
2. solve a second-order differential equation representing damped simple harmonic
motion;
3. solve a second-order differential equation representing forced simple harmonic
motion; and
4. solve a second-order differential equation representing charge and current in an RLC
series circuit.

CONTENT/TECHNICAL INFORMATION

INTRODUCTION
The second-order linear differential equations are used to model many situations in
physics and engineering. In this section, we look at how this works for systems of an object
with mass attached to a vertical spring and an electric circuit containing a resistor, an inductor,
and a capacitor connected in series. Models such as these can be used to approximate other
more complicated situations; for example, bonds between atoms or molecules are often
modeled as springs that vibrate, as described by these same differential equations.

SIMPLE HARMONIC MOTION


Consider a mass suspended from a spring attached to a rigid support. (This is commonly
called a spring-mass system.) Gravity is pulling the mass downward and the restoring force of
the spring is pulling the mass upward. As shown in Figure 12.1, when these two forces are
equal, the mass is said to be at the equilibrium position. If the mass is displaced from
equilibrium, it oscillates up and down. This behavior can be modeled by a second-order
constant-coefficient differential equation.

This module is a property of Technological University of the Philippines Visayas and intended
for EDUCATIONAL PURPOSES ONLY and is NOT FOR SALE NOR FOR REPRODUCTION.
111

(a) (b) (c)


A spring in its natural position (a), at equilibrium with a mass 𝑚 attached (b), and in
oscillatory motion (c).
Figure 12.1

Let 𝑥(𝑡) denote the displacement of the mass from equilibrium. Note that for spring-
mass systems of this type, it is customary to adopt the convention that down is positive. Thus,
a positive displacement indicates the mass is below the equilibrium point, whereas a negative
displacement indicates the mass is above equilibrium. Displacement is usually given in feet in
the English system or meters in the metric system.
Consider the forces acting on the mass. The force of gravity is given by mg.mg. In the
English system, mass is in slugs and the acceleration resulting from gravity is in feet per second
squared. The acceleration resulting from gravity is constant, so in the English system,
𝑔 = 32 𝑓𝑡/𝑠 2 . Recall that 1 𝑠𝑙𝑢𝑔 − 𝑓𝑡/𝑠 2 is a pound, so the expression mg can be expressed
in pounds. Metric system units are kilograms (𝑘𝑔) for mass and 𝑚/𝑠 2 for gravitational
acceleration. In the metric system, we have 𝑔 = 9.8 𝑚/𝑠 2 .
According to Hooke’s law, the restoring force of the spring is proportional to the
displacement and acts in the opposite direction from the displacement, so the restoring force is
given by −𝑘(𝑠 + 𝑥). The spring constant is given in pounds per foot in the English system and
in newtons per meter in the metric system.
Now, by Newton’s second law, the sum of the forces on the system (gravity plus the
restoring force) is equal to mass times acceleration, so we have
𝑚𝑥 ′′ = −𝑘(𝑠 + 𝑥) + 𝑚𝑔
= −𝑘𝑠 − 𝑘𝑥 + 𝑚𝑔

However, by the way we have defined our equilibrium position, 𝑚𝑔 = 𝑘𝑠, the
differential equation becomes

𝑚𝑥 ′′ + 𝑘𝑥 = 0

This module is a property of Technological University of the Philippines Visayas and intended
for EDUCATIONAL PURPOSES ONLY and is NOT FOR SALE NOR FOR REPRODUCTION.
112

It is convenient to rearrange this equation and introduce a new variable, called the angular
frequency, 𝜔. Letting 𝜔 = √𝑘⁄𝑚, we can write the equation as

𝑥 ′′ + 𝜔2 𝑥 = 0.

This differential equation has the general solution


𝑥(𝑡) = 𝑐1 cos 𝜔𝑡 + 𝑐2 sin 𝜔𝑡, (1)
which gives the position of the mass at any point in time. The motion of the mass is called
simple harmonic motion. The period of this motion (the time it takes to complete one
2𝜋 1 𝜔
oscillation) is 𝑇 = and the frequency is 𝑓 = = (Figure 12.2).
𝜔 𝑇 2𝜋

A graph of vertical displacement versus time for simple harmonic motion


Figure 12.2

Example 1. Assume an object weighing 2 𝑙𝑏 stretches a spring 6 in. Find the equation of
motion if the spring is released from the equilibrium position with an upward velocity of
16 𝑓𝑡/𝑠𝑒𝑐. What is the period of motion?

Solution to Example 1.
We first need to find the spring constant. We have,
𝑚𝑔 = 𝑘𝑠
1
2 = 𝑘( )
2
𝑘=4
We also know that weight 𝑊 equals to the product of mass 𝑚 and the acceleration due to
gravity 𝑔. In English units, the acceleration due to gravity is 32 𝑓𝑡/𝑠 2 .
𝑊 = 𝑚𝑔
2 = 𝑚(32)
1
𝑚=
16

This module is a property of Technological University of the Philippines Visayas and intended
for EDUCATIONAL PURPOSES ONLY and is NOT FOR SALE NOR FOR REPRODUCTION.
113

Thus, the differential equation representing the system is


𝟏 ′′
𝒙 + 𝟒𝒙 = 𝟎
𝟏𝟔
Multiplying through by 16, we get 𝑥 ′′ + 64𝑥 = 0, which can also be written in the form
𝑥 ′′ + (8)2 𝑥 = 0. This equation has the general solution
𝑥(𝑡) = 𝑐1 cos(8𝑡) + 𝑐2 sin(8𝑡).
The mass was released from the equilibrium position, so 𝑥(0) = 0, and it had an initial
upward velocity of 16 𝑓𝑡/𝑠𝑒𝑐, so 𝑥 ′ (0) = −16. Applying these initial conditions to solve
for 𝑐1 and 𝑐2 , gives
𝑥(𝑡) = −2 sin(8𝑡).
𝟐𝝅 𝝅
The period of this motion is = sec.
𝟖 𝟒

Writing the general solution in the form 𝑥(𝑡) = 𝑐1 cos 𝜔𝑡 + 𝑐2 sin 𝜔𝑡, (eq.1) has some
advantages. It is easy to see the link between the differential equation and the solution, and the
period and frequency of motion are evident. This form of the function tells us very little about
the amplitude of the motion, however. In some situations, we may prefer to write the solution
in the form
𝑥(𝑡) = 𝐴 sin(𝜔𝑡 + 𝜙)
Although the link to the differential equation is not as explicit in this case, the period
and frequency of motion are still evident. Furthermore, the amplitude of the motion, A, is
obvious in this form of the function. The constant ϕ is called a phase shift and has the effect of
shifting the graph of the function to the left or right.

To convert solution to this form, we want to find the values of 𝐴 and 𝜙 such that
𝑐1 cos(𝜔𝑡) + 𝑐2 sin(𝜔𝑡) = 𝐴 sin(𝜔𝑡 +𝜙 ).
We first apply the trigonometric identity
sin(𝛼 + 𝛽) = sin 𝛼 cos 𝛽 + cos 𝛼 sin 𝛽
to get
𝑐1 cos(𝜔𝑡) + 𝑐2 sin(𝜔𝑡) = 𝐴(sin(𝜔𝑡) cos 𝜙 + cos(𝜔𝑡) sin 𝜙 )
= 𝐴 sin 𝜙 (cos(𝜔𝑡)) + 𝐴 cos 𝜙 (sin(𝜔𝑡))
Thus,
𝑐1 = 𝐴 sin 𝜙 𝑎𝑛𝑑 𝑐2 = 𝐴 cos 𝜙 .

This module is a property of Technological University of the Philippines Visayas and intended
for EDUCATIONAL PURPOSES ONLY and is NOT FOR SALE NOR FOR REPRODUCTION.
114

If we square both of these equations and add them together, we get


𝑐12 + 𝑐22 = 𝐴2 sin2 𝜙 + A2 cos2 𝜙
= 𝐴2 (sin2 𝜙 + cos2 𝜙 )
= 𝐴2 .
Thus,

𝐴 = √𝑐12 + 𝑐22 .

Now, to find ϕ, go back to the equations for 𝑐1 and 𝑐2 , but this time, divide the first equation
by the second equation to get
𝑐1 𝐴 sin 𝜙
=
𝑐2 𝐴 cos 𝜙
= tan 𝜙
Then,
𝑐1
tan 𝜙 = .
𝑐2
We summarize this finding in the following theorem.

Solution to the Equation for Simple Harmonic Motion

The function 𝑥(𝑡) = 𝑐1 cos(𝜔𝑡) + 𝑐2 sin(𝜔𝑡) can be written in the form


𝑐1
𝑥(𝑡) = 𝐴 sin(𝜔𝑡 +𝜙 ), where 𝐴 = √𝑐12 + 𝑐22 and tan 𝜙 = .
𝑐2

𝑐1
Note that when using the formula tan 𝜙 = to find 𝜙, we must take care to ensure 𝜙 is in
𝑐2
the right quadrant (Figure 12.3).

Figure 12.3

This module is a property of Technological University of the Philippines Visayas and intended
for EDUCATIONAL PURPOSES ONLY and is NOT FOR SALE NOR FOR REPRODUCTION.
115

Figure 12.3: A graph of vertical displacement versus time for simple harmonic motion with a
phase change.

Example 2. Express the following functions in the form 𝐴 sin(𝜔𝑡 +𝜙). What is the
frequency motion? The amplitude?
a. 𝑥(𝑡) = 2 cos(3𝑡) + sin(3𝑡)
b. 𝑥(𝑡) = 3 cos(2𝑡) − 2 sin(2𝑡)

Solution to Example 2.
a. We have

𝐴 = √𝑐12 + 𝑐22 = √(2)2 + (1)2 = √5

and
𝑐1 2
tan 𝜙 = = =2
𝑐2 1
Note that both 𝑐1 and 𝑐2 are positive, so 𝜙 is in the first quadrant. Thus,
𝜙 = 1.107 𝑟𝑎𝑑,
So, we have
𝑥(𝑡) = 2 cos(3𝑡) + sin(3𝑡) = √5 sin(3𝑡 + 1.107)
𝜔 3
The frequency is = ≈ 𝟎. 𝟒𝟕𝟕. The amplitude is √𝟓.
2𝜋 2𝜋

b. We have

𝐴 = √𝑐12 + 𝑐22 = √(3)2 + (2)2 = √13

and
𝑐1 3 3
tan 𝜙 = = =− .
𝑐2 −2 2
Note that both 𝑐1 is positive but 𝑐2 is negative, so 𝜙 is in the fourth quadrant. Thus,
𝜙 = −0.983 𝑟𝑎𝑑,
So, we have
𝑥(𝑡) = 3 cos(2𝑡) − 2 sin(2𝑡) = √13 sin(2𝑡 − 0.983)
𝜔 2
The frequency is = ≈ 𝟎. 𝟑𝟏𝟖. The amplitude is √𝟏𝟑.
2𝜋 2𝜋

This module is a property of Technological University of the Philippines Visayas and intended
for EDUCATIONAL PURPOSES ONLY and is NOT FOR SALE NOR FOR REPRODUCTION.
116

Damped Vibrations

With the model just described, the motion of the mass continues indefinitely. Clearly,
this doesn’t happen in the real world. In the real world, there is almost always some friction
in the system, which causes the oscillations to die off slowly—an effect called damping. So
now let’s look at how to incorporate that damping force into our differential equation.

Physical spring-mass systems almost always have some damping as a result of


friction, air resistance, or a physical damper, called a dashpot (a pneumatic cylinder;
Figure 12.4).

A dashpot is a pneumatic cylinder that dampens the motion of an oscillating system


Figure 12.4

Because damping is primarily a friction force, we assume it is proportional to the


velocity of the mass and acts in the opposite direction. So, the damping force is given
by −𝑏𝑥′ for some constant 𝑏 > 0. Again, applying Newton’s second law, the differential
equation becomes
𝑚𝑥 ′′ + 𝑏𝑥 ′ + 𝑘𝑥 = 0
Then, the associated characteristic equation is
𝑚𝜆2 + 𝑏𝜆 + 𝑘 = 0.
Applying the quadratic formula, we have
−𝑏 ± √𝑏 2 − 4𝑚𝑘
𝜆=
2𝑚
Just as in Second-Order Linear Equations we consider three cases, based on whether
the characteristic equation has distinct real roots, a repeated real root, or complex conjugate
roots.

Case 1: Overdamped Vibrations


When 𝒃𝟐 > 𝟒𝒎𝒌, we Say the system is overdamped. The general solution has the form

𝑥(𝑡) = 𝑐1 𝑒 𝜆1𝑡 + 𝑐2 𝑒 𝜆2𝑡

This module is a property of Technological University of the Philippines Visayas and intended
for EDUCATIONAL PURPOSES ONLY and is NOT FOR SALE NOR FOR REPRODUCTION.
117

Where both 𝜆1 and 𝜆2 are less than zero. Because the exponents are negative, the
displacement decays to zero over time, usually quite quickly. Overdamped systems do not
oscillate (no more than one change of direction), but simply move back toward the
equilibrium position. Figure 12.5 shows what typical critically damped behavior looks like.

(a) (b)
Behavior of an overdamped spring-mass Behavior of an overdamped spring-mass
system, with no change in direction system, only one change in direction
Figure 12.5

Example 3. A 16-lb mass is attached to a 10-ft spring. When the mass comes to rest in the
equilibrium position, the spring measures 15 ft 4 in. The system is immersed in a medium
that imparts a damping force equal to 5252 times the instantaneous velocity of the mass. Find
the equation of motion if the mass is pushed upward from the equilibrium position with an
initial upward velocity of 5 ft/sec. What is the position of the mass after 10 sec? Its velocity?

Solution to Example 3.
16 16
The mass stretches the spring 5 ft 4 in., or ft. Thus, 16 = 𝑘, so 𝑘 = 3. We also have
3 3
16 1
𝑚= = , so, the differential equation is
32 2
1 ′′ 5 ′
𝑥 + 𝑥 + 3𝑥 = 0.
2 2
Multiplying through by 2 gives 𝑥 ′′ + 5𝑥 ′ + 6𝑥 = 0, which has the general solution
𝑥(𝑡) = 𝑐1 𝑒 −2𝑡 + 𝑐2 𝑒 −3𝑡 .
Applying the initial conditions, 𝑥(0) = 0 and 𝑥 ′ (0) = −5, we get
𝑥(𝑡) = −5𝑒 −2𝑡 + 5𝑒 −3𝑡 .
After 10 sec. the mass is at position
𝑥(10) = −5𝑒 −20 + 5𝑒 −30 ≈ −1.0305 × 10−8 ≈ 0,

This module is a property of Technological University of the Philippines Visayas and intended
for EDUCATIONAL PURPOSES ONLY and is NOT FOR SALE NOR FOR REPRODUCTION.
118

so it is, effectively, at the equilibrium position. We have 𝑥 ′ (𝑡) = 10𝑒 −2𝑡 − 15𝑒 −3𝑡 , so after
10 sec. the mass is moving at a velocity of
𝑥(10) = 10𝑒 −20 − 15𝑒 −30 ≈ 2.061 × 10−8 ≈ 0.
After only 10 sec, the mass is barely moving.

Case 2: Critically Damped Vibrations


When 𝒃𝟐 = 𝟒𝒎𝒌, we say the system is critically damped. The general solution has the form

𝑥(𝑡) = 𝑐1 𝑒 𝜆1𝑡 + 𝑐2 𝑡𝑒 𝜆1𝑡


where 𝜆1 is less than zero. The motion of a critically damped system is very similar to that of
an overdamped system. It does not oscillate. However, with a critically damped system, if the
damping is reduced even a little, oscillatory behavior results. From a practical perspective,
physical systems are almost always either overdamped or underdamped (case 3, which we
consider next). It is impossible to fine-tune the characteristics of a physical system so
that 𝑏 2 and 4𝑚𝑘 are exactly equal. Figure 12.6 shows what typical critically damped
behavior looks like.

(a) (b)
Behavior of a critically damped spring-mass system. The system graphed in part (𝑎) has
more damping than the system graphed in part (𝑏)
Figure 12.6

Example 4. A 1-kg mass stretches a spring 20 cm. The system is attached to a dashpot that
imparts a damping force equal to 14 times the instantaneous velocity of the mass. Find the
equation of motion if the mass is released from equilibrium with an upward velocity of 3
m/sec.

Solution to Example 4.
We have 𝑚𝑔 = 1(9.8) = 0.2𝑘, so 𝑘 = 49. Then, the differential equation is
𝑥 ′′ + 14𝑥 ′ + 49𝑥 = 0,

This module is a property of Technological University of the Philippines Visayas and intended
for EDUCATIONAL PURPOSES ONLY and is NOT FOR SALE NOR FOR REPRODUCTION.
119

which has general solution


𝑥(𝑡) = 𝑐1 𝑒 −7𝑡 + 𝑐2 𝑡𝑒 −7𝑡 .
Applying the initial conditions 𝑥(0) = 0 and 𝑥 ′ (0) = −3 gives
𝒙(𝒕) = −𝟑𝒕𝒆−𝟕𝒕 .

Case 3: Undamped Vibrations


When 𝒃𝟐 < 𝟒𝒎𝒌, we say the system is underdamped. The general solution has the form\

𝑥(𝑡) = 𝑒 𝛼𝑡 (𝑐1 cos(𝛽𝑡) + 𝑐2 sin(𝛽𝑡)),

where 𝛼 is less than zero. Underdamped systems do oscillate because of the sine and cosine
terms in the solution. However, the exponential term dominates eventually, so the amplitude
of the oscillations decreases over time. Figure 12.7 shows what typical underdamped
behavior looks like.

Behavior of an underdamped spring-mass system


Figure 12.7
Note that for all damped systems, lim 𝑥(𝑡) = 0. The system always approaches the
𝑡→∞
equilibrium position over time.

Example 5. A 16-lb weight stretches a spring 3.2 ft. Assume the damping force on the
system is equal to the instantaneous velocity of the mass. Find the equation of motion if the
mass is released from rest at a point 9 in. below equilibrium.

Solution to Example 5.
16 16 1
We have 𝑘 = = 5 and 𝑚 = = , so the differential equation is
3.2 32 2

This module is a property of Technological University of the Philippines Visayas and intended
for EDUCATIONAL PURPOSES ONLY and is NOT FOR SALE NOR FOR REPRODUCTION.
120

1 ′′
𝑥 + 𝑥 ′ + 5𝑥 = 0, 𝑜𝑟 𝑥 ′′ + 2𝑥 ′ + 10𝑥 = 0.
2
This equation has the general solution
𝑥(𝑡) = 𝑒 −𝑡 (𝑐1 cos(3𝑡) + 𝑐2 sin(3𝑡)).
3
Applying the initial conditions, 𝑥(0) = 4 and 𝑥 ′ (0) = 0, we get
𝟑 𝟏
𝒙(𝒕) = 𝒆−𝒕 ( 𝐜𝐨𝐬(𝟑𝒕) + 𝐬𝐢𝐧(𝟑𝒕)).
𝟒 𝟒

Example 6. Chapter Opener: Modeling a Motorcycle Suspension System


For motocross riders, the suspension systems on their
motorcycles are very important. The off-road courses on
which they ride often include jumps and losing control of the
motorcycle when they land could cost them the race.

This suspension system can be modeled as a damped spring-


mass system. We define our frame of reference with respect
to the frame of the motorcycle. Assume the end of the shock
absorber attached to the motorcycle frame is fixed. Then, the
“mass” in our spring-mass system is the motorcycle wheel.
We measure the position of the wheel with respect to the
motorcycle frame. This may seem counterintuitive, since, in
many cases, it is actually the motorcycle frame that moves,
but this frame of reference preserves the development of the
differential equation that was done earlier. As with earlier
development, we define the downward direction to be
positive. Credit: modification of work by Seika, Flickr
Figure 12.8
When the motorcycle is lifted by its frame, the wheel hangs freely and the spring is
uncompressed. This is the spring’s natural position. When the motorcycle is placed on the
ground and the rider mounts the motorcycle, the spring compresses and the system is in the
equilibrium position (Figure 12.9).

Figure 12.9

This module is a property of Technological University of the Philippines Visayas and intended
for EDUCATIONAL PURPOSES ONLY and is NOT FOR SALE NOR FOR REPRODUCTION.
121

This system can be modeled using the same differential equation we used before:
𝑚𝑥 ′′ + 𝑏𝑥 ′ + 𝑘𝑥 = 0

A motocross motorcycle weighs 204 𝑙𝑏, and we assume a rider weight of 180 𝑙𝑏. When the
rider mounts the motorcycle, the suspension compresses 4 in., then comes to rest at
equilibrium. The suspension system provides damping equal to 240 times the instantaneous
vertical velocity of the motorcycle (and rider).
1. Set up the differential equation that models the behavior of the motorcycle suspension
system.
2. We are interested in what happens when the motorcycle lands after taking a jump. Let
time, 𝑡 = 0 denote the time when the motorcycle first contacts the ground. If the
motorcycle hits the ground with a velocity of 10 ft/sec downward, find the equation
of motion of the motorcycle after the jump.
3. Graph the equation of motion over the first second after the motorcycle hits the
ground.

Solution to Example 6.
1. We have defined equilibrium to be the point where 𝑚𝑔 = 𝑘𝑠, so we have
𝑚𝑔 = 𝑘𝑠
1
384 = 𝑘 ( )
3
𝑘 = 1152.
We also have,
𝑊 = 𝑚𝑔
384 = 𝑚(32)
𝑚 = 12.
Therefore, the differential equation that models the behavior of the motorcycle
suspension is
12𝑥 ′′ + 240𝑥 ′ + 1152𝑥 = 0
Dividing through by 12, we get
𝑥 ′′ + 20𝑥 ′ + 96𝑥 = 0.

2. The differential equation found in part a has the general solution


𝑥(𝑡) = 𝑐1 𝑒 −8𝑡 + 𝑐2 𝑒 −12𝑡

This module is a property of Technological University of the Philippines Visayas and intended
for EDUCATIONAL PURPOSES ONLY and is NOT FOR SALE NOR FOR REPRODUCTION.
122

Now, to determine our initial conditions, we consider the position and velocity of the
motorcycle wheel when the wheel first contacts the ground. Since the motorcycle was in the
air prior to contacting the ground, the wheel was hanging freely, and the spring was
1
uncompressed. Therefore, the wheel is 4 in. (3 𝑓𝑡) below the equilibrium position (with
1
respect to the motorcycle frame), and we have 𝑥(0) = . According to the problem
3
statement, the motorcycle has a velocity of 10 ft/sec downward when the motorcycle contacts
7 19
the ground, so 𝑥 ′ (0) = 10. Applying these initial conditions, we get 𝑐1 = and 𝑐2 = − 6
2
so the equation of motion is

7 19 −12𝑡
𝑥(𝑡) = 𝑒 −8𝑡 − 𝑒 .
2 6
3. Figure 12.10: Graph of the equation of motion over a time of one second

Figure 12.10

Forced Vibrations
The last case we consider is when an external force acts on the system. In the case of the
motorcycle suspension system, for example, the bumps in the road act as an external force
acting on the system. Another example is a spring hanging from a support; if the support is
set in motion, that motion would be considered an external force on the system. We model
these forced systems with the nonhomogeneous differential equation

𝑚𝑥 ′′ + 𝑏𝑥 ′ + 𝑘𝑥 = 𝑓(𝑡),

This module is a property of Technological University of the Philippines Visayas and intended
for EDUCATIONAL PURPOSES ONLY and is NOT FOR SALE NOR FOR REPRODUCTION.
123

where the external force is represented by the f(t)f(t) term. As we saw in Non-homogenous
Linear Equations, differential equations such as this have solutions of the form
𝑥(𝑡) = 𝑐1 𝑥1 (𝑡) + 𝑐2 𝑥2 (𝑡) + 𝑥𝑝 (𝑡),
where 𝑐1 𝑥1 (𝑡) + 𝑐2 𝑥2 (𝑡) is the general solution to the complementary equation and 𝑥𝑝 (𝑡) is a
particular solution to the nonhomogeneous equation. If the system is damped, lim 𝑐1 𝑥1 (𝑡) +
𝑡→∞
𝑐2 𝑥2 (𝑡) = 0. Since these terms do not affect the long-term behavior of the system, we call
this part of the solution the transient solution. The long-term behavior of the system is
determined by 𝑥𝑝 (𝑡) so we call this part of the solution the steady-state solution.

Example 7. A mass of 1 slug stretches a spring 2 ft and comes to rest at equilibrium. The
system is attached to a dashpot that imparts a damping force equal to eight times the
instantaneous velocity of the mass. Find the equation of motion if an external force equal
to 𝑓(𝑡) = 8 sin(4𝑡) is applied to the system beginning at time 𝑡 = 0. What is the transient
solution? What is the steady-state solution?

Solution to Example 7.
We have 𝑚𝑔 = 1(32) = 2𝑘, so 𝑘 = 16 and the differential equation is
𝑥 ′′ + 8𝑥 ′ + 16𝑥 = 8 sin(4𝑡).
The general solution to the complementary equation is
𝑐1 𝑒 −4𝑡 + 𝑐2 𝑡𝑒 −4𝑡 .
Assuming a particular solution of the form 𝑥𝑝 (𝑡) = 𝐴 cos(4𝑡) + 𝐵 sin(4𝑡) and using the
1
method of undetermined coefficients, we find 𝑥𝑝 (𝑡) = − 4 cos(4𝑡), so

1
𝑥(𝑡) = 𝑐1 𝑒 −4𝑡 + 𝑐2 𝑡𝑒 −4𝑡 − cos(4𝑡).
4
At 𝑡 = 0, the mass is at rest in the equilibrium position, so 𝑥(0) = 𝑥 ′ (0) = 0. Applying these
initial conditions to solve the 𝑐1 and 𝑐2 , we get
1 −4𝑡 1
𝑥(𝑡) = 𝑒 + 𝑡𝑒 −4𝑡 − cos(4𝑡).
4 4
𝟏 𝟏
The transient solution is 𝒆−𝟒𝒕 + 𝒕𝒆−𝟒𝒕 . The steady-state solution is − 𝐜𝐨𝐬(𝟒𝒕).
𝟒 𝟒

This module is a property of Technological University of the Philippines Visayas and intended
for EDUCATIONAL PURPOSES ONLY and is NOT FOR SALE NOR FOR REPRODUCTION.
124

The RLC Series Circuit


Consider an electrical circuit containing a resistor, an inductor, and a capacitor, as shown in
Figure 12.12. Such a circuit is called an RLC series circuit. RLC circuits are used in many
electronic systems, most notably as tuners in AM/FM radios. The tuning knob varies the
capacitance of the capacitor, which in turn tunes the radio. Such circuits can be modeled by
second-order, constant-coefficient differential equations.

An 𝑅𝐿𝐶 series circuit can be modeled by the same differential equation as a mass-spring
system
Figure 12.12

Let 𝐼(𝑡) denote the current in the RLC circuit and 𝑞(𝑡) denote the charge on the capacitor.
Furthermore, let 𝐿 denote inductance in henrys (𝐻), 𝑅 denote resistance in ohms (𝛺),
and 𝐶 denote capacitance in farads (𝐹). Last, let 𝐸(𝑡) denote electric potential in volts (𝑉).

Kirchhoff’s voltage rule states that the sum of the voltage drops around any closed loop must
be zero. So, we need to consider the voltage drops across the inductor (denoted 𝐸𝐿 ), the resistor
(denoted 𝐸𝑅 ), and the capacitor (denoted 𝐸𝐶 ). Because the 𝑅𝐿𝐶 circuit shown in
Figure 12.12 includes a voltage source, 𝐸(𝑡), which adds voltage to the circuit, we have
𝐸𝐿 + 𝐸𝑅 + 𝐸𝐶 = 𝐸(𝑡).

We present the formulas below without further development and those of you interested in
the derivation of these formulas can review the links. Using Faraday’s law and Lenz’s law,
the voltage drop across an inductor can be shown to be proportional to the instantaneous rate
of change of current, with proportionality constant 𝐿. Thus,
𝑑𝐼
𝐸𝐿 = 𝐿 .
𝑑𝑡

Next, according to Ohm’s law, the voltage drop across a resistor is proportional to the current
passing through the resistor, with proportionality constant 𝑅. Therefore,
𝐸𝑅 = 𝑅𝐼.

This module is a property of Technological University of the Philippines Visayas and intended
for EDUCATIONAL PURPOSES ONLY and is NOT FOR SALE NOR FOR REPRODUCTION.
125

Last, the voltage drop across a capacitor is proportional to the charge, 𝑞, on the capacitor,
1
with proportionality constant . Thus,
𝐶
1
𝐸𝑐 = 𝑞.
𝐶
Adding these terms together, we get
𝑑𝐼 1
𝐿 + 𝑅𝐼 + 𝑞 = 𝐸(𝑡)
𝑑𝑡 𝐶
𝑑𝑞
Noting that 𝐼 = , this becomes
𝑑𝑡
𝑑2𝑞 𝑑𝑞 1
𝐿 + 𝑅 + 𝑞 = 𝐸(𝑡)
𝑑𝑡 2 𝑑𝑡 𝐶
Mathematically, this system is analogous to the spring-mass systems we have been
examining in this section.

Example 8. Find the charge on the capacitor in an RLC series circuit where
5 1
𝐿 = 3 𝐻, 𝑅 = 10𝛺, 𝐶 = 30 𝐹, and 𝐸(𝑡) = 300𝑉. Assume the initial charge on the capacitor is
0 𝐶 and the initial current is 9 𝐴. What happens to the charge on the capacitor over time?

Solution to Example 8.
We have
𝑑2𝑞 𝑑𝑞 1
𝐿 2
+𝑅 + 𝑞 = 𝐸(𝑡)
𝑑𝑡 𝑑𝑡 𝐶
5 𝑑2𝑞 𝑑𝑞
( ) 2 + (10) + (30)𝑞 = 300
3 𝑑𝑡 𝑑𝑡
𝑑2 𝑞 𝑑𝑞
+ 6 + 18𝑞 = 180
𝑑𝑡 2 𝑑𝑡
The general solution to the complementary equation is
𝑒 −3𝑡 (𝑐1 cos(3𝑡) + 𝑐2 sin(3𝑡)).
Assume a particular solution of the form 𝑞𝑝 = 𝐴, where 𝐴 is a constant. Using the method of
undetermined coefficients, we find 𝐴 = 10. So,

𝑞(𝑡) = 𝑒 −3𝑡 (𝑐1 cos(3𝑡) + 𝑐2 sin(3𝑡)) + 10


𝑑𝑞
Applying the initial conditions 𝑞(0) = 0 and 𝑖(0) = ( 𝑑𝑡 ) (0) = 9, we find 𝑐1 = −10 and
𝑐2 = −7. So, the charge on the capacitor is

𝑞(𝑡) = −10𝑒 −3𝑡 cos(3𝑡) − 7𝑒 −3𝑡 sin(3𝑡) + 10.

This module is a property of Technological University of the Philippines Visayas and intended
for EDUCATIONAL PURPOSES ONLY and is NOT FOR SALE NOR FOR REPRODUCTION.
126

Looking closely at this function, we see the first two terms will decay over time (as a result of
the negative exponent in the exponential function). Therefore, the capacitor eventually
approaches a steady-state charge of 10 𝐶.

Key Concepts

• Second-order constant-coefficient differential equations can be used to model spring-


mass systems.
• An examination of the forces on a spring-mass system results in a differential
equation of the form

𝑚𝑥″ + 𝑏𝑥′ + 𝑘𝑥 = 𝑓(𝑡),

where mm represents the mass, bb is the coefficient of the damping force, 𝑘is the
spring constant, and 𝑓(𝑡) represents any net external forces on the system.

• If 𝑏 = 0, there is no damping force acting on the system, and simple harmonic motion
results.
• If 𝑏 ≠ 0,the behavior of the system depends on whether
𝑏 2 − 4𝑚𝑘 > 0, 𝑏 2 − 4𝑚𝑘 = 0, 𝑜𝑟 𝑏 2 − 4𝑚𝑘 < 0.
❖ If 𝑏 2 − 4𝑚𝑘 > 0, the system is overdamped and does not exhibit oscillatory
behavior.
❖ If 𝑏 2 − 4𝑚𝑘 = 0, the system is critically damped. It does not exhibit
oscillatory behavior, but any slight reduction in the damping would result in
oscillatory behavior.
❖ If 𝑏 2 − 4𝑚𝑘 < 0, the system is underdamped. It exhibits oscillatory behavior,
but the amplitude of the oscillations decreases over time.
• If 𝑓(𝑡) ≠ 0, the solution to the differential equation is the sum of a transient solution
and a steady-state solution. The steady-state solution governs the long-term behavior
of the system.
• The charge on the capacitor in an 𝑅𝐿𝐶 series circuit can also be modeled with a
second-order constant-coefficient differential equation of the form

𝑑2𝑞 𝑑𝑞 1
𝐿 + 𝑅 + 𝑞 = 𝐸(𝑡),
𝑑𝑡 2 𝑑𝑡 𝐶
where 𝐿 is the inductance, 𝑅 is the resistance, 𝐶 is the capacitance, and 𝐸(𝑡) is the
voltage source.

This module is a property of Technological University of the Philippines Visayas and intended
for EDUCATIONAL PURPOSES ONLY and is NOT FOR SALE NOR FOR REPRODUCTION.
127

Key Equations

• Equation of a simple harmonic motion


𝑥 ′′ + 𝜔2 𝑥 = 0

• Solution for simple harmonic motion


𝑥(𝑡) = 𝑐1 cos(𝜔𝑡) + 𝑐2 sin(𝜔𝑡)

• Alternative form of solution for SHM


𝑥(𝑡) = 𝐴 sin(𝜔𝑡 +𝜙)

• Forced harmonic motion


𝑚𝑥 ′′ + 𝑏𝑥 ′ + 𝑘𝑥 = 𝑓(𝑡)

• Charge in a 𝑅𝐿𝐶 series circuit


𝑑2𝑞 𝑑𝑞 1
𝐿 2
+𝑅 + 𝑞 = 𝐸(𝑡)
𝑑𝑡 𝑑𝑡 𝐶

This module is a property of Technological University of the Philippines Visayas and intended
for EDUCATIONAL PURPOSES ONLY and is NOT FOR SALE NOR FOR REPRODUCTION.
128

PROGRESS CHECK
Activity Sheets
EXERCISE 9
Review on Applications of 2nd Order Differential Equations

Name: _____________________________________________Year & Section: __________


Subject: ____________________________________________Date Submitted: __________
Professor: __________________________________________Score: __________________

Solve the following problem:

Simple Harmonic Motion

1. A 200-g mass stretches a spring 5 cm. Find the equation of motion of the mass if it is
released from rest from a position 10 cm below the equilibrium position. What is the
frequency of this motion?
Hint: First find the spring constant
14
Ans: 𝑥(𝑡) = 0.1 cos(14𝑡)(in meters); frequency is Hz.
2𝜋

2. Express the function 𝑥(𝑡) = cos(4𝑡) + 4 sin(4𝑡) in the form 𝐴 sin(𝜔𝑡 +𝜙). What is
the frequency of motion? The amplitude?
4
Ans: 𝑥(𝑡) = √17 sin(4𝑡 + 0.245), frequency = 2𝜋 ≈ 0.637, 𝐴 = √17

Overdamped Vibrations

3. A 2-kg mass is attached to a spring with spring constant 24 N/m. The system is then
immersed in a medium imparting a damping force equal to 16 times the instantaneous
velocity of the mass. Find the equation of motion if it is released from rest at a point
40 cm below equilibrium.
Ans: 𝑥(𝑡) = 0.6𝑒 −2𝑡 − 0.2𝑒 −6𝑡

Critically Damped Vibrations

4. A 1-lb weight stretches a spring 6 in., and the system is attached to a dashpot that
imparts a damping force equal to half the instantaneous velocity of the mass. Find the
equation of motion if the mass is released from rest at a point 6 in. below equilibrium.
1
Ans: 𝑥(𝑡) = 2 𝑒 −8𝑡 + 4𝑡𝑒 −8𝑡

This module is a property of Technological University of the Philippines Visayas and intended
for EDUCATIONAL PURPOSES ONLY and is NOT FOR SALE NOR FOR REPRODUCTION.
129

Undamped Vibrations

5. A 1-kg mass stretches a spring 49 cm. The system is immersed in a medium that
imparts a damping force equal to four times the instantaneous velocity of the mass.
Find the equation of motion if the mass is released from rest at a point 24 cm above
equilibrium.
Ans: 𝑥(𝑡) = −0.24𝑒 −2𝑡 cos(4𝑡) − 0.12𝑒 −2𝑡 sin(4𝑡)

Forced Vibrations

6. A mass of 2 kg is attached to a spring with constant 32 N/m and comes to rest in the
equilibrium position. Beginning at time𝑡 = 0, an external force equal to 𝑓(𝑡) =
68𝑒 −2𝑡 cos(4𝑡) is applied to the system. Find the equation of motion if there is no
damping. What is the transient solution? What is the steady-state solution?
1 9 1
Ans: 𝑥(𝑡) = − cos(4𝑡) + sin(4𝑡) + 𝑒 −2𝑡 cos(4𝑡) − 2𝑒 −2𝑡 sin(4𝑡)
2 4 2
1 −2𝑡
Transient Solution: 𝑒 cos(4𝑡) − 2𝑒 −2𝑡 sin(4𝑡)
2
1 9
Steady-state Solution: − 2 cos(4𝑡) + 4 sin(4𝑡)

RLC Series Circuit

1 2
7. Find the charge on the capacitor in an 𝑅𝐿𝐶 series circuit where 𝐿 = 5 𝐻, 𝑅 = 5 𝛺, 𝐶 =
1
𝐹, and 𝐸(𝑡) = 50 𝑉. Assume the initial charge on the capacitor is 0 𝐶 and the initial
2
is 4 𝐴.
Ans: 𝑞(𝑡) = −25𝑒 −𝑡 cos(3𝑡) − 7𝑒 −𝑡 sin(3𝑡) + 25

This module is a property of Technological University of the Philippines Visayas and intended
for EDUCATIONAL PURPOSES ONLY and is NOT FOR SALE NOR FOR REPRODUCTION.
130

REFERENCES

Gilbert Strang & Edwin “Jed” Herman. Applications of Second-Order Differential Equations.
OpenStax CNX., August 2020.

Applications of Second-Order Differential Equations, (2020). Retrieved from


https://math.libretexts.org/Bookshelves/Calculus/Book%3A_Calculus_(OpenStax)/17%3A_S
econd-Order_Differential_Equations/17.3%3A_Applications_of_Second-
Order_Differential_Equations

Ricardo C. Asin. Elementary Differential Equations Reviewer. Manila, Philippines, Merriam


& Webster Bookstore, Inc., 1991.

This module is a property of Technological University of the Philippines Visayas and intended
for EDUCATIONAL PURPOSES ONLY and is NOT FOR SALE NOR FOR REPRODUCTION.
131

LEARNING GUIDE

Week No.: __13__

TOPIC/S

I. The Laplace Transforms of Functions


a. Definition
b. Transform of Elementary Functions
c. Inverse Transforms
d. Transforms of Derivatives
EXPECTED COMPETENCIES
At the end of the week after studying the module, the student should be able to:
1. know the definition and properties of Laplace;
2. evaluate the Laplace transforms;
3. know the definition and properties of inverse Laplace; and
4. evaluate inverse Laplace transforms;

CONTENT/TECHNICAL INFORMATION

THE LAPLACE TRANSFORM

Definition and Properties of Laplace Transform


Let 𝐹(𝑡) be any function. The Laplace Transform of 𝐹(𝑡), denoted by 𝐿{𝐹(𝑡)}, is defined by

𝐿{𝐹(𝑡)} = ∫ 𝑒 −𝑠𝑡 𝐹(𝑡)𝑑𝑡


0

The result of the integral of Laplace is always a function 𝑠. If the function is denoted by call
the 𝑓(𝑠), then

𝐿{𝐹(𝑡)} = ∫ 𝑒 −𝑠𝑡 𝐹(𝑡)𝑑𝑡


0

Properties of Laplace Transform


1. 𝐿{𝑐𝐹(𝑡)} = 𝑐𝐿{𝐹(𝑡)}

2. 𝐿{𝑐1 𝐹1 (𝑡) + 𝑐2 𝐹2 (𝑡)} = 𝑐1 𝐿{𝐹1 (𝑡)} + 𝑐2 𝐿{𝐹2 (𝑡)}

1 𝑐
3. 𝐿{1} = and 𝐿{𝑐} =
𝑠 𝑠

This module is a property of Technological University of the Philippines Visayas and intended
for EDUCATIONAL PURPOSES ONLY and is NOT FOR SALE NOR FOR REPRODUCTION.
132

To show property 3, set 𝐹(𝑡) = 1 such that




1
𝐿{1} = ∫ 𝑒 −𝑠𝑡 𝐹(𝑡)𝑑𝑡 = − 𝑒 −𝑠𝑡 |
𝑠 0
0

1 ∞
= − 𝑠𝑡 |
𝑠𝑒 0
1
Note: lim 𝑒 𝑠𝑡 = 0, if 𝑠 > 1
𝑡→∞
1
= 0+
𝑠
𝟏
𝑳{𝟏} =
𝒔
𝑐 𝑐
In general, 𝐿{𝑐} = because 𝐿{𝑐} = 𝑐𝐿{1} = .
𝑠 𝑠

Example 1. Find 𝐿{𝑒 𝑘𝑡 }.

Solution to Example 1.

𝐿{𝑒 𝑘𝑡 } = ∫ 𝑒 −𝑠𝑡 𝑒 𝑘𝑡 𝑑𝑡
0

= ∫ 𝑒 −𝑡(𝑠−𝑘) 𝑑𝑡
0

1
= − 𝑒 −𝑡(𝑠−𝑘) |
𝑠−𝑘 0

1
= − |
(𝑠 − 𝑘)𝑒 𝑡(𝑠−𝑘) 0
1
= 0+
𝑠−𝑘
𝟏
𝑳{𝒆𝒌𝒕 } = ,𝑠 > 𝑘
𝒔−𝒌

It is important to consider the imposed condition such as 𝑠 > 𝑘.


For example,
1
𝐿{𝑒 5𝑡 } =
𝑠−5
1
𝐿{𝑒 𝑘𝑡 } =
𝑠−𝑘

This module is a property of Technological University of the Philippines Visayas and intended
for EDUCATIONAL PURPOSES ONLY and is NOT FOR SALE NOR FOR REPRODUCTION.
133

Example 2. Find 𝐿{cos 𝑘𝑡}.

Solution to Example 2.

𝐿{cos 𝑘𝑡} = ∫ 𝑒 −𝑠𝑡 cos 𝑘𝑡 𝑑𝑡


0

By parts, let
𝑢 = 𝑒 −𝑠𝑡 𝑑𝑣 = cos 𝑘𝑡 𝑑𝑡
1
𝑑𝑢 = −𝑠𝑒 −𝑠𝑡 𝑑𝑡 𝑣= sin 𝑘𝑡
𝑘
By parts formula,
∞ ∞

∫ 𝑒 −𝑠𝑡 cos 𝑘𝑡 𝑑𝑡 = 𝑢𝑣 − ∫ 𝑣𝑑𝑢


0 0

1 𝑠
= 𝑒 −𝑠𝑡 sin 𝑘𝑡 + ∫ 𝑒 −𝑠𝑡 sin 𝑘𝑡 𝑑𝑡
𝑘 𝑘
0
sin 𝑘𝑡
Since lim 𝑒 −𝑠𝑡 sin 𝑘𝑡 = 0, and lim = 0, where 𝑠 > 0
𝑡→∞ 𝑡→0 𝑒 𝑠𝑡
∞ ∞
𝑠
∫ 𝑒 −𝑠𝑡 cos 𝑘𝑡 𝑑𝑡 = ∫ 𝑒 −𝑠𝑡 sin 𝑘𝑡 𝑑𝑡
𝑘
0 0

To integrate the right member, we use by parts. Let


𝑢 = 𝑒 −𝑠𝑡 𝑑𝑣 = sin 𝑘𝑡 𝑑𝑡
1
𝑑𝑢 = −𝑠𝑒 −𝑠𝑡 𝑑𝑡 𝑣 = − cos 𝑘𝑡
𝑘

∞ ∞
−𝑠𝑡
∫𝑒 sin 𝑘𝑡 𝑑𝑡 = 𝑢𝑣 − ∫ 𝑣𝑑𝑢
0 0

1 𝑠
= − 𝑒 −𝑠𝑡 cos 𝑘𝑡 − ∫ 𝑒 −𝑠𝑡 cos 𝑘𝑡 𝑑𝑡
𝑘 𝑘
0
∞ ∞ ∞
𝑒 −𝑠𝑡 𝑠
∫ 𝑒 −𝑠𝑡 sin 𝑘𝑡 𝑑𝑡 = − cos 𝑘𝑡| − ∫ 𝑒 −𝑠𝑡 cos 𝑘𝑡 𝑑𝑡
𝑘 0
𝑘
0 0

cos 𝑘𝑡 𝑒 −𝑠𝑡 cos 𝑘𝑡


Since lim 𝑒 𝑠𝑡
= = 0, and lim = 1, where 𝑠 > 0
𝑡→∞ 𝑘 𝑡→0 𝑒 𝑠𝑡

This module is a property of Technological University of the Philippines Visayas and intended
for EDUCATIONAL PURPOSES ONLY and is NOT FOR SALE NOR FOR REPRODUCTION.
134

Thus,

𝑠
𝐿{cos 𝑘𝑡} = ∫ 𝑒 −𝑠𝑡 sin 𝑘𝑡 𝑑𝑡
𝑘
0
∞ ∞
−𝑠𝑡
𝑠 1 𝑠
∫𝑒 cos 𝑘𝑡 𝑑𝑡 = [ − ∫ 𝑒 −𝑠𝑡 cos 𝑘𝑡 𝑑𝑡]
𝑘 𝑘 𝑘
0 0

𝑠 𝑠2
= 2 − 2 ∫ 𝑒 −𝑠𝑡 cos 𝑘𝑡 𝑑𝑡
𝑘 𝑘
0

𝑠2 𝑠
[1 + 2
] ∫ 𝑒 −𝑠𝑡 cos 𝑘𝑡 𝑑𝑡 = 2
𝑘 𝑘
0

𝑠 𝑘2
∫ 𝑒 −𝑠𝑡 cos 𝑘𝑡 𝑑𝑡 = ∙
𝑘2 𝑠2 + 𝑘2
0
𝒔
𝑳{𝐜𝐨𝐬 𝒌𝒕} = ,𝑠 > 0
𝒔𝟐 + 𝒌𝟐

Example 3. Obtain 𝐿{𝑡 𝑛 }

Solution to Example 3.

𝑛}
𝐿{𝑡 = ∫ 𝑒 −𝑠𝑡 𝑡 𝑛 𝑑𝑡
0

By parts, we have
𝑢 = 𝑡𝑛 𝑑𝑣 = 𝑒 −𝑠𝑡 𝑑𝑡
1
𝑑𝑢 = 𝑛𝑡 𝑛−1 𝑑𝑡 𝑣 = − 𝑒 −𝑠𝑡
𝑠

∞ ∞
𝑡𝑛 ∞ 𝑛
∫ 𝑒 𝑡 𝑑𝑡 = − | + ∫ 𝑒 −𝑠𝑡 𝑡 𝑛−1 𝑑𝑡
−𝑠𝑡 𝑛
𝑠 0 𝑠
0 0
𝑡𝑛 𝑡𝑛
Notice that lim = 0, and lim = 0, where 𝑠 > 0
𝑡→∞ 𝑠 𝑡→0 𝑠

𝑛
𝐿{𝑡 𝑛 } = ∫ 𝑒 −𝑠𝑡 𝑡 𝑛−1 𝑑𝑡
𝑠
0
𝑛
𝐿{𝑡 𝑛 } = 𝐿{𝑡 𝑛−1 }
𝑠

This module is a property of Technological University of the Philippines Visayas and intended
for EDUCATIONAL PURPOSES ONLY and is NOT FOR SALE NOR FOR REPRODUCTION.
135

The last statement has an important assertion in the property of Laplace transform, i.e.
𝑛
𝐿{𝑡 𝑛 } = 𝐿{𝑡 𝑛−1 }
𝑠
𝑛−1
𝐿{𝑡 𝑛−1 } = 𝐿{𝑡 𝑛−2 }
𝑠
𝑛−2
𝐿{𝑡 𝑛−2 } = 𝐿{𝑡 𝑛−3 }, 𝑒𝑡𝑐.
𝑠
Thus, we have the result
𝑛
𝐿{𝑡 𝑛 } = 𝐿{𝑡 𝑛−1 }
𝑠
𝑛 𝑛−1
= ∙ ∙ 𝐿{𝑡 𝑛−2 }
𝑠 𝑠
𝑛 𝑛−1 𝑛−2
= ∙ ∙ ∙ 𝐿{𝑡 𝑛−3 }
𝑠 𝑠 𝑠
Since the exponent of the power goes down in the numerator and keep on decreasing, then
will eventually terminate to 1, i.e.
𝑛 𝑛−1 𝑛−2 1
𝐿{𝑡 𝑛 } = ∙ ∙ … 𝐿{𝑡 𝑛−3 }
𝑠 𝑠 𝑠 𝑠
1
Knowing that 𝐿{1} = , and 𝑛(𝑛 − 1)(𝑛 − 2) … 2(1) = 𝑛!, then
𝑠
𝒏!
𝑳{𝒕𝒏 } =
𝒔𝒏+𝟏

For example

1
𝐿{𝑡} =
𝑠2
2! 2
𝐿{𝑡 2 } = 3 = 3
𝑠 𝑠
3! 6
𝐿{𝑡 3 } = 4 = 4
𝑠 𝑠
4! 24
𝐿{𝑡 4 } = 5 = 5
𝑠 𝑠
5! 120
𝐿{𝑡 5 } = 6 = 6
𝑠 𝑠

This module is a property of Technological University of the Philippines Visayas and intended
for EDUCATIONAL PURPOSES ONLY and is NOT FOR SALE NOR FOR REPRODUCTION.
136

Example 4. Evaluate 𝐿{5𝑡 3 − 2𝑡 2 − 3𝑡 + 6}

Solution to Example 4.
By property 2 of Laplace transform.
𝐿{5𝑡 3 − 2𝑡 2 − 3𝑡 + 6} = 5𝐿{𝑡 3 } − 2𝐿{𝑡 2 } − 3𝐿{𝑡} + 6𝐿{1}
6 2 1 1
= 5 ( 4) − 2 ( 3) − 3 ( 2) + 6 ( )
𝑠 𝑠 𝑠 𝑠
30 4 3 6
= 4 − 3− 2+
𝑠 𝑠 𝑠 𝑠

Example 5. Evaluate 𝐿{cosh 𝑎𝑡}

Solution to Example 5.
𝑒 𝑥 +𝑒 −𝑥
From Calculus cosh 𝑥 = so that
2
𝑒 𝑎𝑡 + 𝑒 −𝑎𝑡
𝐿{cosh 𝑎𝑡} = 𝐿 { }
2
1
= [𝐿{𝑒 𝑎𝑡 } + 𝐿{𝑒 −𝑎𝑡 }]
2
1 1
= +
2(𝑠 − 𝑎) 2(𝑠 + 𝑎)
𝟐𝒔
𝑳{𝐜𝐨𝐬𝐡 𝒂𝒕} =
𝒔𝟐 − 𝒂𝟐

For example
2𝑠
𝐿{cosh 7𝑡} =
𝑠 2 − 49

This module is a property of Technological University of the Philippines Visayas and intended
for EDUCATIONAL PURPOSES ONLY and is NOT FOR SALE NOR FOR REPRODUCTION.
137

Example 6. Evaluate 𝐿{sinh 𝑎𝑡}

Solution to Example 6.
𝑒 𝑥 −𝑒 −𝑥
From Calculus, sinh 𝑥 = , so that
2
𝑒 𝑎𝑡 − 𝑒 −𝑎𝑡
𝐿{sinh 𝑎𝑡} = 𝐿 { }
2
1
= [𝐿{𝑒 𝑎𝑡 } − 𝐿{𝑒 −𝑎𝑡 }]
2
1 1
= −
2(𝑠 − 𝑎) 2(𝑠 + 𝑎)
𝟐𝒂
𝑳{𝐬𝐢𝐧𝐡 𝒂𝒕} =
𝒔𝟐 − 𝒂𝟐

For example,
2(3) 6
𝐿{sinh 3𝑡} = 2
= 2
𝑠 −9 𝑠 −9

This module is a property of Technological University of the Philippines Visayas and intended
for EDUCATIONAL PURPOSES ONLY and is NOT FOR SALE NOR FOR REPRODUCTION.
138

INVERSE LAPLACE TRANSFORM


From the definition of Laplace transform
𝐿{𝐹(𝑡)} = 𝑓(𝑠)
We then define the inverse Laplace transform, or simply an inverse transform, denoted by
𝐿−1 {𝑓(𝑠)} as
𝐿−1 {𝑓(𝑠)} = 𝐹(𝑡)

Examples

1 1
𝐿−1 { } = 𝑒 𝑘𝑡 , 𝑠𝑖𝑛𝑐𝑒 𝐿{𝑒 𝑘𝑡 } =
𝑠−𝑘 𝑠−𝑘
𝑠 𝑠
𝐿−1 { } = cos 𝑘𝑡 , 𝑠𝑖𝑛𝑐𝑒 𝐿{cos 𝑘𝑡} =
𝑠2 + 𝑘2 𝑠2 + 𝑘2
𝑘 𝑘
𝐿−1 { } = sin 𝑘𝑡 , 𝑠𝑖𝑛𝑐𝑒 𝐿{sin 𝑘𝑡} =
𝑠2 + 𝑘2 𝑠2 + 𝑘2
1 𝑛!
𝐿−1 { } = 𝑡𝑛 , 𝑠𝑖𝑛𝑐𝑒 𝐿{𝑡 𝑛 } =
𝑠 𝑛+1 𝑠 𝑛+1
1 1
𝐿−1 { 2 } = 𝑡, 𝑠𝑖𝑛𝑐𝑒 𝐿{𝑡} =
𝑠 𝑠2
1 𝑡2 2
𝐿−1 { 3 } = , 𝑠𝑖𝑛𝑐𝑒 𝐿{𝑡 2 } =
𝑠 2 𝑠3
1 𝑡3 6
𝐿−1 { 4 } = , 𝑠𝑖𝑛𝑐𝑒 𝐿{𝑡 3 } =
𝑠 6 𝑠4
1 𝑡4 4
𝐿−1 { 5 } = , 𝑠𝑖𝑛𝑐𝑒 𝐿{𝑡 4 } =
𝑠 24 𝑠5
1 1
𝐿−1 { } = 1, 𝑠𝑖𝑛𝑐𝑒 𝐿{1} =
𝑠 𝑠

Properties of Inverse Laplace Transform


1. 𝐿−1 {𝑐𝑓(𝑠)} = 𝑐𝐿−1 {𝑓(𝑠)}

2. 𝐿−1 {𝑐1 𝑓1 (𝑠) + 𝑐2 𝑓2 (𝑠)} = 𝑐1 𝐿−1 {𝑓1 (𝑠)} + 𝑐2 𝐿−1 {𝑓2 (𝑠)}

A very important property of Inverse Laplace transform is embodied in the following


theorem:

This module is a property of Technological University of the Philippines Visayas and intended
for EDUCATIONAL PURPOSES ONLY and is NOT FOR SALE NOR FOR REPRODUCTION.
139

Theorem 11.1

𝐿−1 {𝑓(𝑠)} = 𝑒 −𝑎𝑡 𝐿−1 {𝑓(𝑠 − 𝑎)}

Proof. From the definition of Laplace transform

𝐿{𝐹(𝑡)} = 𝑓(𝑠), or

𝑓(𝑠) = ∫ 𝑒 −𝑠𝑡 𝐹(𝑡)𝑑𝑡


0

Substituting (𝑠 − 𝑎) for 𝑠 in the preceding equation, we get


𝑓(𝑠 − 𝑎) = ∫ 𝑒 −(𝑠−𝑎)𝑡 𝐹(𝑡)𝑑𝑡


0

= ∫ 𝑒 −𝑠𝑡 [𝑒 𝑎𝑡 𝐹(𝑡)] 𝑑𝑡
0

𝑓(𝑠 − 𝑎) = 𝐿{𝑒 𝑎𝑡 𝐹(𝑡)}

It follows that

𝐿−1 {𝑓(𝑠 − 𝑎)} = 𝑒 𝑎𝑡 𝐹(𝑡)

Since 𝐹(𝑡) = 𝐿−1 {𝑓(𝑠)}, then

𝐿−1 {𝑓(𝑠 − 𝑎)} = 𝑒 𝑎𝑡 𝐹 −1 {𝑓(𝑠)}

Example 7. Find

8
𝐿−1 { }
𝑠2 + 4𝑠 + 20
Solution to Example 7.

8 8
𝐿−1 { } = 𝐿−1
{ }
𝑠 2 + 4𝑠 + 20 (𝑠 + 2)2 + 16

This implies that in 𝑠 + 2 = 𝑠 − (−2), we have 𝑎 = −2 , by

Theorem 11.1, we have

8 8
𝐿−1 { 2
} = 𝑒 −2𝑡 𝐿−1 { 2 }
(𝑠 + 2) + 16 𝑠 + 16

This module is a property of Technological University of the Philippines Visayas and intended
for EDUCATIONAL PURPOSES ONLY and is NOT FOR SALE NOR FOR REPRODUCTION.
140

4
= 2𝑒 −2𝑡 𝐿−1 { }
𝑠2 + 16

= 𝟐𝒆−𝟐𝒕 𝐬𝐢𝐧 𝟒𝒕

Example 8. Find
8
𝐿−1 { }
𝑠 2 − 6𝑠 + 13
Solution to Example 8.
𝑠 𝑠
𝐿−1 { 2 } = 𝐿−1 { }
𝑠 − 4𝑠 + 13 (𝑠 − 2)2 + 9
This implies that in 𝑠 − 2, then 𝑎 = 2, and
𝑠 − 2 is transformed into 𝑠
𝑠 is transformed into 𝑠 + 2
Thus, by Theorem 11.1
𝑠 𝑠+2
𝐿−1 { 2 } = 𝑒 2𝑡 𝐿−1 { 2 }
𝑠 − 4𝑠 + 13 𝑠 +9
𝑠 2 3
= 𝑒 2𝑡 [𝐿−1 { 2 } + 𝐿−1 { 2 }]
𝑠 +9 3 𝑠 +9
2
= 𝑒 2𝑡 (cos 3𝑡 + sin 3𝑡)
3

Example 9. Find
1
𝐿−1 { }
𝑠 2 − 4𝑠 + 4
Solution to Example 9.

1 1
𝐿−1 { } = 𝐿−1 { }
𝑠2 − 4𝑠 + 4 (𝑠 − 2)2

with 𝑠 − 2, we have 𝑎 = 2 such that 𝑠 − 2 is transform into 𝑠

then by Theorem 11.1,

1 1
𝐿−1 { } = 𝑒 2𝑡 −1
𝐿 { }
𝑠 2 − 4𝑠 + 4 𝑠2

= 𝒆𝟐𝒕 𝒕

This module is a property of Technological University of the Philippines Visayas and intended
for EDUCATIONAL PURPOSES ONLY and is NOT FOR SALE NOR FOR REPRODUCTION.
141

Example 10. Find


𝑠−3
𝐿−1 { }
𝑠 2 + 4𝑠 + 4
Solution to Example 10.

𝑠−3 𝑠−3
𝐿−1 { } = 𝐿−1 { }
𝑠2 + 4𝑠 + 4 (𝑠 + 2)2

with 𝑠 + 2, 𝑎 = −2 such that

𝑠 + 2 is transformed into 𝑠
𝑠 − 3 is transformed into 𝑠 − 5
Then by Theorem 11.1,
𝑠−3 𝑠−5
𝐿−1 { } = 𝑒 −2𝑡 −1
𝐿 { }
𝑠 2 + 4𝑠 + 4 𝑠2
𝑠 1
= 𝑒 −2𝑡 𝐿−1 { 2 } − 5𝐿−1 { 2 }
𝑠 𝑠
= 𝒆𝟐𝒕 (𝟏 − 𝟓𝒕)

Partial Fractions
Partial Fraction is one useful method for evaluating inverse Laplace transforms. We
need to resolve the proper fraction into its partial forms to obtain easily the inverse Laplace
transform of a function.

Example 11. Find


𝑠+7
𝐿−1 { }
𝑠2 −𝑠−2
Solution to Example 11.
𝑠+7
We first resolve into partial fractions, i.e.
𝑠 2 −𝑠−2
𝑠+7 𝐴 𝐵
= +
(𝑠 − 2)(𝑠 + 1) 𝑠 − 2 𝑠 + 1
𝑠 + 7 = 𝐴(𝑠 + 1) + 𝐵(𝑠 − 2)
Let 𝑠 = −1, then −1 + 7 = −3𝐵, or 𝐵 = −2
𝑠 = 2, then 2 + 7 = 3𝐴, or 𝐴 = 3
Thus,
𝑠+7 3 2
𝐿−1 { } = 𝐿−1
{ − }
𝑠2 − 𝑠 − 4 𝑠−2 𝑠+1

This module is a property of Technological University of the Philippines Visayas and intended
for EDUCATIONAL PURPOSES ONLY and is NOT FOR SALE NOR FOR REPRODUCTION.
142

1 1
= 3𝐿−1 { } − 2𝐿−1 { }
𝑠−2 𝑠+1
with 𝑠 − 2, we take 𝑎 = 2 and for 𝑠 + 1, 𝑎 = −1. Then by Theorem 11.1
𝑠+7 1 1
𝐿−1 { } = 3𝑒 2𝑡 −1
𝐿 { } − 2𝑒 −𝑡 −1
𝐿 { }
𝑠2 − 𝑠 − 2 𝑠 𝑠
= 3𝑒 2𝑡 (1) − 2𝑒 −𝑡 (1)
= 𝟑𝒆𝟐𝒕 − 𝟐𝒆−𝒕

Example 12. Find


𝑠+2
𝐿−1 { }
𝑠 2 (𝑠 − 1)
Solution to Example 12.
By partial fraction
𝑠+2 𝐴 𝐵 𝐶
= + 2+
𝑠 2 (𝑠
− 1) 𝑠 𝑠 𝑠−1
𝑠 + 2 = 𝐴𝑠(𝑠 − 1) + 𝐵𝑠(𝑠 − 1) + 𝐶𝑠 2
Let, 𝑠 = 0, 2 = −𝐵 or 𝐵 = −2
𝑠 = 1, 3=𝐶
𝑠 = −1, 1 = −𝐴(−2) + 𝐵(−2) + 𝐶
1 = 2𝐴 + 4 + 3
−6 = 2𝐴
𝐴 = −3
Thus,
𝑠+2 3 2 3
𝐿−1 { } = 𝐿−1
{− − + }
𝑠 2 (𝑠 − 1) 𝑠 𝑠2 𝑠 − 1
1 1 1
= −3𝐿−1 { } − 2𝐿−1 { 2 } + 3𝐿−1 { }
𝑠 𝑠 𝑠−1
1
= −3(1) − 2𝑡 − 3𝑒 𝑡 𝐿−1 { }
𝑠
= −𝟑 − 𝟐𝒕 − 𝟑𝒆𝒕

This module is a property of Technological University of the Philippines Visayas and intended
for EDUCATIONAL PURPOSES ONLY and is NOT FOR SALE NOR FOR REPRODUCTION.
143

Example 13. Obtain


𝑠+2
𝐿−1 { }
𝑠(𝑠 2 + 1)
Solution to Example 13.
By partial fractions,
𝑠+2 𝐴 𝐵(2𝑠) + 𝐶
2
= +
𝑠(𝑠 + 1) 𝑠 𝑠2 + 1
𝑠 + 2 = 𝐴(𝑠 2 + 1) + (2𝐵𝑠 + 𝐶)𝑠
Let 𝑠 = 0, 2=𝐴
𝑠 = 1, 3 = 2𝐴 + 2𝐵 + 𝐶
3 = 4 + 2𝐵 + 𝐶
𝑠 = −1, 1 = 2𝐴 − (−2𝐵 + 𝐶)
−3 = 2𝐵 − 𝐶
Solving equation (1) and (2), we get
4𝐵 = −4, 𝑜𝑟 𝐵 = −1
−2 + 𝐶 = −1, 𝑜𝑟 𝐶 = 1
Thus,
𝑠+2 2 2𝑠 1
𝐿−1 { 2
} = 𝐿−1 { − 2 + 2 }
𝑠(𝑠 + 1) 𝑠 𝑠 +1 𝑠 +1
1 𝑠 1
= 2𝐿−1 { } − 2𝐿−1 { 2 } + 𝐿−1 { 2 }
𝑠 𝑠 +1 𝑠 +1
= −𝟐 − 𝟐 𝐜𝐨𝐬 𝒕 + 𝐬𝐢𝐧 𝒕

This module is a property of Technological University of the Philippines Visayas and intended
for EDUCATIONAL PURPOSES ONLY and is NOT FOR SALE NOR FOR REPRODUCTION.
144

PROGRESS CHECK
Activity Sheets
EXERCISE 10
Review on Laplace Transforms and Inverse Laplace Transform

Name: _____________________________________________Year & Section: __________


Subject: ____________________________________________Date Submitted: __________
Professor: __________________________________________Score: __________________

For numbers 1 to 10, evaluate the Laplace transforms:


1. 𝐿{2𝑡 2 } 6. 𝐿{6𝑡 4 − 7𝑡 5 }

2. 𝐿{5𝑡 3 + 3 sin 4𝑡} 7. 𝐿{3 cos 2𝑡}

3. 𝐿{𝑡 3 − 𝑡 2 + 5𝑡} 8. 𝐿{𝑡 2 + 4𝑡 − 5}

4. 𝐿{𝑡 7 − 4𝑡 − 6} 9. 𝐿{sinh 7𝑡}

5. 𝐿{7𝑡 − 1} 10. 𝐿{cosh 9𝑡}

For numbers 11 to 20, evaluate the following Inverse Laplace transforms:


1 2
11. 𝐿−1 { } 16. 𝐿−1 { }
𝑠2 +2𝑠+5 𝑠2 +𝑠

1 𝑠+1
12. 𝐿−1 { } 17. 𝐿−1 { }
𝑠2 +6𝑠+13 𝑠2 −3𝑠+2

𝑠+2 𝑠+1
13. 𝐿−1 { } 18. 𝐿−1 { }
𝑠2 +4𝑠+13 𝑠2 −3𝑠+2

𝑠−5 1
14. 𝐿−1 { } 19. 𝐿−1 { }
𝑠2 +6𝑠+13 (𝑠−1)(𝑠2 −1)

1 1
15. 𝐿−1 { } 20. 𝐿−1 { }
𝑠2 +10𝑠+25 𝑠2 (𝑠2 +4)

This module is a property of Technological University of the Philippines Visayas and intended
for EDUCATIONAL PURPOSES ONLY and is NOT FOR SALE NOR FOR REPRODUCTION.
145

REFERENCES
Sergio E. Ymas Jr. Differential Equations (Simplified Approach). Sampaloc, Manila. Pablo.
L. Bustamante III Press, 1997. Chapter 10 & 11.

Ricardo C. Asin. Elementary Differential Equations Reviewer. Manila, Philippines, Merriam


& Webster Bookstore, Inc., 1991.

This module is a property of Technological University of the Philippines Visayas and intended
for EDUCATIONAL PURPOSES ONLY and is NOT FOR SALE NOR FOR REPRODUCTION.
146

LIST OF REFERENCES
Sergio E. Ymas Jr. Differential Equations (Simplified Approach). Sampaloc, Manila. Pablo.
L. Bustamante III Press, 1997.

Louis Leithold. The Calculus with Analytic Geometry. 6th ed. Harper & Row Publishers Inc.,
1990.

Clyde E. Love, Earl D. Rainville. Differential and Integral Calculus. 6th ed. The Macmillan
Co., 1961.

Gilbert Strang & Edwin “Jed” Herman. Applications of Second-Order Differential Equations.
OpenStax CNX., August 2020.

Applications of Second-Order Differential Equations, (2020). Retrieved from


https://math.libretexts.org/Bookshelves/Calculus/Book%3A_Calculus_(OpenStax)/17%3A_S
econd-Order_Differential_Equations/17.3%3A_Applications_of_Second-
Order_Differential_Equations

Ricardo C. Asin. Elementary Differential Equations Reviewer. Manila, Philippines, Merriam


& Webster Bookstore, Inc., 1991.

Ricardo C. Asin. Integral Calculus Reviewer. Revised Edition. Manila, Philippines, Merriam
& Webster Bookstore, Inc., 1991.

This module is a property of Technological University of the Philippines Visayas and intended
for EDUCATIONAL PURPOSES ONLY and is NOT FOR SALE NOR FOR REPRODUCTION.
147

ABOUT THE COMPILER

Genesis Deles Dumaicos is a licensed Mechanical Engineer practicing in the field of academe
since 2015 and a member of Philippine Society of Mechanical Engineers – Negros Occidental
Chapter. He graduated his bachelor’s degree at the Technological University of the Philippines
Visayas in 2014 and is about to complete his master’s degree at Technological University of
the Philippines.

This module is a property of Technological University of the Philippines Visayas and intended
for EDUCATIONAL PURPOSES ONLY and is NOT FOR SALE NOR FOR REPRODUCTION.

You might also like